Sie sind auf Seite 1von 150

Community Health Nursing Practice Questions with Rationales

1. Which is the primary goal of community health nursing?


A. To support and supplement the efforts of the medical profession in the promotion of
health and prevention of illness
B. To enhance the capacity of individuals, families and communities to cope with their
health needs
C. To increase the productivity of the people by providing them with services that will
increase their level of health
D. To contribute to national development through promotion of family welfare, focusing
particularly on mothers and children.
Answer: (B) To enhance the capacity of individuals, families and
communities to cope with their health needs
To contribute to national development through promotion of family welfare, focusing
particularly on mothers and children.
2. CHN is a community-based practice. Which best explains this statement?
A. The service is provided in the natural environment of people.
B. The nurse has to conduct community diagnosis to determine nursing needs and
problems.
C. The services are based on the available resources within the community.
D. Priority setting is based on the magnitude of the health problems identified.
Answer: A. The service is provided in the natural environment of people.
Community-based practice means providing care to people in their own natural
environments: the home, school and workplace, for example.
3. Population-focused nursing practice requires which of the following processes?
A. Community organizing
B. Nursing process
C. Community diagnosis
D. Epidemiologic process
Answer: (C) Community diagnosis
Population-focused nursing care means providing care based on the greater need of the
majority of the population. The greater need is identified through community diagnosis.
4. R.A. 1054 is also known as the Occupational Health Act. Aside from number of
employees, what other factor must be considered in determining the occupational health
privileges to which the workers will be entitled?
A. Type of occupation: agricultural, commercial, industrial
B. Location of the workplace in relation to health facilities
C. Classification of the business enterprise based on net profit
D. Sex and age composition of employees
Answer: (B) Location of the workplace in relation to health facilities

Based on R.A. 1054, an occupational nurse must be employed when there are 30 to 100
employees and the workplace is more than 1 km. away from the nearest health center.
5. A business firm must employ an occupational health nurse when it has at least how
many employees?
A. 21
B. 101
C. 201
D. 301
Answer: (B) 101
Again, this is based on R.A. 1054.
6. When the occupational health nurse employs ergonomic principles, she is performing
which of her roles?
A. Health care provider
B. Health educator
C. Health care coordinator
D. Environmental manager
Answer: (D) Environmental manager
Ergonomics is improving efficiency of workers by improving the workers environment
through appropriately designed furniture, for example.
7. A garment factory does not have an occupational nurse. Who shall provide the
occupational health needs of the factory workers?
A. Occupational health nurse at the Provincial Health Office
B. Physician employed by the factory
C. Public health nurse of the RHU of their municipality
D. Rural sanitary inspector of the RHU of their municipality
Answer: (C) Public health nurse of the RHU of their municipality
Youre right! This question is based on R.A.1054.
8. Public health services are given free of charge. Is this statement true or false?
A. The statement is true; it is the responsibility of government to provide basic services.
B. The statement is false; people pay indirectly for public health services.
C. The statement may be true or false, depending on the specific service required.
D. The statement may be true or false, depending on policies of the government
concerned.
Answer: (B) The statement is false; people pay indirectly for public health
services.
Community health services, including public health services, are pre-paid services,
though taxation, for example.

9. According to C.E.Winslow, which of the following is the goal of Public Health?


A. For people to attain their birthrights of health and longevity
B. For promotion of health and prevention of disease
C. For people to have access to basic health services
D. For people to be organized in their health efforts
Answer: (A) For people to attain their birthrights of health and longevity
According to Winslow, all public health efforts are for people to realize their birthrights
of health and longevity.
10. We say that a Filipino has attained longevity when he is able to reach the average
lifespan of Filipinos. What other statistic may be used to determine attainment of
longevity?
A. Age-specific mortality rate
B. Proportionate mortality rate
C. Swaroops index
D. Case fatality rate
Answer: (C) Swaroops index
Swaroops index is the percentage of the deaths aged 50 years or older. Its inverse
represents the percentage of untimely deaths (those who died younger than 50 years).
11. Which of the following is the most prominent feature of public health nursing?
A. It involves providing home care to sick people who are not confined in the hospital.
B. Services are provided free of charge to people within the catchment area.
C. The public health nurse functions as part of a team providing a public health nursing
services.
D. Public health nursing focuses on preventive, not curative, services.
Answer: (D) Public health nursing focuses on preventive, not curative,
services.
The catchment area in PHN consists of a residential community, many of whom are well
individuals who have greater need for preventive rather than curative services.
12. According to Margaret Shetland, the philosophy of public health nursing is based on
which of the following?
A. Health and longevity as birthrights
B. The mandate of the state to protect the birthrights of its citizens
C. Public health nursing as a specialized field of nursing
D. The worth and dignity of man
Answer: (D) The worth and dignity of man
This is a direct quote from Dr. Margaret Shetlands statements on Public Health
Nursing.
13. Which of the following is the mission of the Department of Health?
A. Health for all Filipinos

B. Ensure the accessibility and quality of health care


C. Improve the general health status of the population
D. Health in the hands of the Filipino people by the year 2020
Answer: (B) Ensure the accessibility and quality of health care
(none)
14. Region IV Hospital is classified as what level of facility?
A. Primary
B. Secondary
C. Intermediate
D. Tertiary
Answer: (D) Tertiary
Regional hospitals are tertiary facilities because they serve as training hospitals for the
region.
15. Which is true of primary facilities?
A. They are usually government-run.
B. Their services are provided on an out-patient basis.
C. They are training facilities for health professionals.
D. A community hospital is an example of this level of health facilities.
Answer: (B) Their services are provided on an out-patient basis.
Primary facilities government and non-government facilities that provide basic outpatient services.
16. Which is an example of the school nurses health care provider functions?
A. Requesting for BCG from the RHU for school entrant immunization
B. Conducting random classroom inspection during a measles epidemic
C. Taking remedial action on an accident hazard in the school playground
D. Observing places in the school where pupils spend their free time
Answer: (B) Conducting random classroom inspection during a measles
epidemic
Random classroom inspection is assessment of pupils/students and teachers for signs of
a health problem prevalent in the community.
17. When the nurse determines whether resources were maximized in implementing
Ligtas Tigdas, she is evaluating
A. Effectiveness
B. Efficiency
C. Adequacy
D. Appropriateness
Answer: (B) Efficiency
Efficiency is determining whether the goals were attained at the least possible cost.

18. You are a new B.S.N. graduate. You want to become a Public Health Nurse. Where
will you apply?
A. Department of Health
B. Provincial Health Office
C. Regional Health Office
D. Rural Health Unit
Answer: (D) Rural Health Unit
R.A. 7160 devolved basic health services to local government units (LGUs ). The public
health nurse is an employee of the LGU.
19. R.A. 7160 mandates devolution of basic services from the national government to
local government units. Which of the following is the major goal of devolution?
A. To strengthen local government units
B. To allow greater autonomy to local government units
C. To empower the people and promote their self-reliance
D. To make basic services more accessible to the people
Answer: (C) To empower the people and promote their self-reliance
People empowerment is the basic motivation behind devolution of basic services to
LGUs.
20. Who is the Chairman of the Municipal Health Board?
A. Mayor
B. Municipal Health Officer
C. Public Health Nurse
D. Any qualified physician
Answer: (A) Mayor
The local executive serves as the chairman of the Municipal Health Board.
21. Which level of health facility is the usual point of entry of a client into the health care
delivery system?
A. Primary
B. Secondary
C. Intermediate
D. Tertiary
Answer: (A) Primary
The entry of a person into the health care delivery system is usually through a
consultation in out-patient services.
22. The public health nurse is the supervisor of rural health midwives. Which of the
following is a supervisory function of the public health nurse?
A. Referring cases or patients to the midwife
B. Providing technical guidance to the midwife

C. Providing nursing care to cases referred by the midwife


D. Formulating and implementing training programs for midwives
Answer: (B) Providing technical guidance to the midwife
The nurse provides technical guidance to the midwife in the care of clients, particularly
in the implementation of management guidelines, as in Integrated Management of
Childhood Illness.
23. One of the participants in a hilot training class asked you to whom she should refer a
patient in labor who develops a complication. You will answer, to the
A. Public Health Nurse
B. Rural Health Midwife
C. Municipal Health Officer
D. Any of these health professionals
Answer: (C) Municipal Health Officer
A public health nurse and rural health midwife can provide care during normal
childbirth. A physician should attend to a woman with a complication during labor.
24. You are the public health nurse in a municipality with a total population of about
20,000. There are 3 rural health midwives among the RHU personnel. How many more
midwife items will the RHU need?
A. 1
B. 2
C. 3
D. The RHU does not need any more midwife item.
Answer: (A) 1
Each rural health midwife is given a population assignment of about 5,000.
25. If the RHU needs additional midwife items, you will submit the request for
additional midwife items for approval to the
A. Rural Health Unit
B. District Health Office
C. Provincial Health Office
D. Municipal Health Board
Answer: (D) Municipal Health Board
As mandated by R.A. 7160, basic health services have been devolved from the national
government to local government units.
26. As an epidemiologist, the nurse is responsible for reporting cases of notifiable
diseases. What law mandates reporting of cases of notifiable diseases?
A. Act 3573
B. R.A. 3753
C. R.A. 1054
D. R.A. 1082

Answer: (A) Act 3573


Act 3573, the Law on Reporting of Communicable Diseases, enacted in 1929, mandated
the reporting of diseases listed in the law to the nearest health station.
27. According to Freeman and Heinrich, community health nursing is a developmental
service. Which of the following best illustrates this statement?
A. The community health nurse continuously develops himself personally and
professionally.
B. Health education and community organizing are necessary in providing community
health services.
C. Community health nursing is intended primarily for health promotion and
prevention and treatment of disease.
D. The goal of community health nursing is to provide nursing services to people in their
own places of residence.
Answer: (B) Health education and community organizing are necessary in
providing community health services.
The community health nurse develops the health capability of people through health
education and community organizing activities.
28. Which disease was declared through Presidential Proclamation No. 4 as a target for
eradication in the Philippines?
A. Poliomyelitis
B. Measles
C. Rabies
D. Neonatal tetanus
Answer: (B) Measles
Presidential Proclamation No. 4 is on the Ligtas Tigdas Program.
29. The public health nurse is responsible for presenting the municipal health statistics
using graphs and tables. To compare the frequency of the leading causes of mortality in
the municipality, which graph will you prepare?
A. Line
B. Bar
C. Pie
D. Scatter diagram
Answer: (B) Bar
A bar graph is used to present comparison of values, a line graph for trends over time or
age, a pie graph for population composition or distribution, and a scatter diagram for
correlation of two variables.
30. Which step in community organizing involves training of potential leaders in the
community?
A. Integration

B. Community organization
C. Community study
D. Core group formation
Answer: (D) Core group formation
In core group formation, the nurse is able to transfer the technology of community
organizing to the potential or informal community leaders through a training program.
31. In which step are plans formulated for solving community problems?
A. Mobilization
B. Community organization
C. Follow-up/extension
D. Core group formation
Answer: (B) Community organization
Community organization is the step when community assemblies take place. During the
community assembly, the people may opt to formalize the community organization and
make plans for community action to resolve a community health problem.
32. The public health nurse takes an active role in community participation. What is the
primary goal of community organizing?
A. To educate the people regarding community health problems
B. To mobilize the people to resolve community health problems
C. To maximize the communitys resources in dealing with health problems
D. To maximize the communitys resources in dealing with health problems
Answer: (D) To maximize the communitys resources in dealing with health
problems
Community organizing is a developmental service, with the goal of developing the
peoples self-reliance in dealing with community health problems. A, B and C are
objectives of contributory objectives to this goal.
33. An indicator of success in community organizing is when people are able to
A. Participate in community activities for the solution of a community problem
B. Implement activities for the solution of the community problem
C. Plan activities for the solution of the community problem
D. Identify the health problem as a common concern
Answer: (A) Participate in community activities for the solution of a
community problem
Participation in community activities in resolving a community problem may be in any
of the processes mentioned in the other choices.
34. Tertiary prevention is needed in which stage of the natural history of disease?
A. Pre-pathogenesis
B. Pathogenesis
C. Prodromal

D. Terminal
Answer: (D) Terminal
Tertiary prevention involves rehabilitation, prevention of permanent disability and
disability limitation appropriate for convalescents, the disabled, complicated cases and
the terminally ill (those in the terminal stage of a disease)
35. Isolation of a child with measles belongs to what level of prevention?
A. Primary
B. Secondary
C. Intermediate
D. Tertiary
Answer: (A) Primary
The purpose of isolating a client with a communicable disease is to protect those who
are not sick (specific disease prevention).
36. On the other hand, Operation Timbang is _____ prevention.
A. Primary
B. Secondary
C. Intermediate
D. Tertiary
Answer: (B) Secondary
Operation Timbang is done to identify members of the susceptible population who are
malnourished. Its purpose is early diagnosis and, subsequently, prompt treatment.
37. Which type of family-nurse contact will provide you with the best opportunity to
observe family dynamics?
A. Clinic consultation
B. Group conference
C. Home visit
D. Written communication
Answer: (C) Home visit
Dynamics of family relationships can best be observed in the familys natural
environment, which is the home.
38. The typology of family nursing problems is used in the statement of nursing
diagnosis in the care of families. The youngest child of the de los Reyes family has been
diagnosed as mentally retarded. This is classified as a
A. Health threat
B. Health deficit
C. Foreseeable crisis
D. Stress point
Answer: (B) Health deficit

Failure of a family member to develop according to what is expected, as in mental


retardation, is a health deficit.
39. The de los Reyes couple have a 6-year old child entering school for the first time. The
de los Reyes family has a
A. Health threat
B. Health deficit
C. Foreseeable crisis
D. Stress point
Answer: (C) Foreseeable crisis
Entry of the 6-year old into school is an anticipated period of unusual demand on the
family.
40. Which of the following is an advantage of a home visit?
A. It allows the nurse to provide nursing care to a greater number of people.
B. It provides an opportunity to do first hand appraisal of the home situation.
C. It allows sharing of experiences among people with similar health problems.
D. It develops the familys initiative in providing for health needs of its members.
Answer: (B) It provides an opportunity to do first hand appraisal of the
home situation.
Choice A is not correct since a home visit requires that the nurse spend so much time
with the family. Choice C is an advantage of a group conference, while choice D is true of
a clinic consultation.
41. Which is CONTRARY to the principles in planning a home visit?
A. A home visit should have a purpose or objective.
B. The plan should revolve around family health needs.
C. A home visit should be conducted in the manner prescribed by the RHU.
D. Planning of continuing care should involve a responsible family member.
Answer: (C) A home visit should be conducted in the manner prescribed by
the RHU.
The home visit plan should be flexible and practical, depending on factors, such as the
familys needs and the resources available to the nurse and the family.
42. The PHN bag is an important tool in providing nursing care during a home visit. The
most important principle of bag technique states that it
A. Should save time and effort.
B. Should minimize if not totally prevent the spread of infection.
C. Should not overshadow concern for the patient and his family.
D. May be done in a variety of ways depending on the home situation, etc.
Answer: (B) Should minimize if not totally prevent the spread of infection.
Bag technique is performed before and after handling a client in the home to prevent
transmission of infection to and from the client.

43. To maintain the cleanliness of the bag and its contents, which of the following must
the nurse do?
A. Wash his/her hands before and after providing nursing care to the family members.
B. In the care of family members, as much as possible, use only articles taken from the
bag.
C. Put on an apron to protect her uniform and fold it with the right side out before
putting it back into the bag.
D. At the end of the visit, fold the lining on which the bag was placed, ensuring that the
contaminated side is on the outside.
Answer: (A) Wash his/her hands before and after providing nursing care to
the family members.
Choice B goes against the idea of utilizing the familys resources, which is encouraged in
CHN. Choices C and D goes against the principle of asepsis of confining the
contaminated surface of objects.
44. The public health nurse conducts a study on the factors contributing to the high
mortality rate due to heart disease in the municipality where she works. Which branch
of epidemiology does the nurse practice in this situation?
A. Descriptive
B. Analytical
C. Therapeutic
D. Evaluation
Answer: (B) Analytical
Analytical epidemiology is the study of factors or determinants affecting the patterns of
occurrence and distribution of disease in a community.
45. Which of the following is a function of epidemiology?
A. Identifying the disease condition based on manifestations presented by a client
B. Determining factors that contributed to the occurrence of pneumonia in a 3 year old
C. Determining the efficacy of the antibiotic used in the treatment of the 3 year old client
with pneumonia
D. Evaluating the effectiveness of the implementation of the Integrated Management of
Childhood Illness
Answer: (D) Evaluating the effectiveness of the implementation of the
Integrated Management of Childhood Illness
Epidemiology is used in the assessment of a community or evaluation of interventions in
community health practice.
46. Which of the following is an epidemiologic function of the nurse during an
epidemic?
A. Conducting assessment of suspected cases to detect the communicable disease
B. Monitoring the condition of the cases affected by the communicable disease
C. Participating in the investigation to determine the source of the epidemic

D. Teaching the community on preventive measures against the disease


Answer: (C) Participating in the investigation to determine the source of the
epidemic
Epidemiology is the study of patterns of occurrence and distribution of disease in the
community, as well as the factors that affect disease patterns. The purpose of an
epidemiologic investigation is to identify the source of an epidemic, i.e., what brought
about the epidemic.
47. The primary purpose of conducting an epidemiologic investigation is to
A. Delineate the etiology of the epidemic
B. Encourage cooperation and support of the community
C. Identify groups who are at risk of contracting the disease
D. Identify geographical location of cases of the disease in the community
Answer: (A) Delineate the etiology of the epidemic
Delineating the etiology of an epidemic is identifying its source.
48. Which is a characteristic of person-to-person propagated epidemics?
A. There are more cases of the disease than expected.
B. The disease must necessarily be transmitted through a vector.
C. The spread of the disease can be attributed to a common vehicle.
D. There is a gradual build up of cases before the epidemic becomes easily noticeable.
Answer: (D) There is a gradual build up of cases before the epidemic
becomes easily noticeable.
A gradual or insidious onset of the epidemic is usually observable in person-to-person
propagated epidemics.
49. In the investigation of an epidemic, you compare the present frequency of the
disease with the usual frequency at this time of the year in this community. This is done
during which stage of the investigation?
A. Establishing the epidemic
B. Testing the hypothesis
C. Formulation of the hypothesis
D. Appraisal of facts
Answer: (A) Establishing the epidemic
Establishing the epidemic is determining whether there is an epidemic or not. This is
done by comparing the present number of cases with the usual number of cases of the
disease at the same time of the year, as well as establishing the relatedness of the cases
of the disease.
50. The number of cases of Dengue fever usually increases towards the end of the rainy
season. This pattern of occurrence of Dengue fever is best described as
A. Epidemic occurrence
B. Cyclical variation

C. Sporadic occurrence
D. Secular variation
Answer: (B) Cyclical variation
A cyclical variation is a periodic fluctuation in the number of cases of a disease in the
community.
51. In the year 1980, the World Health Organization declared the Philippines, together
with some other countries in the Western Pacific Region, free of which disease?
A. Pneumonic plague
B. Poliomyelitis
C. Small pox
D. Anthrax
Answer: (C) Small pox
The last documented case of Small pox was in 1977 at Somalia.
52. In the census of the Philippines in 1995, there were about 35,299,000 males and
about 34,968,000 females. What is the sex ratio?
A. 99.06:100
B. 100.94:100
C. 50.23%
D. 49.76%
Answer: (B) 100.94:100
Sex ratio is the number of males for every 100 females in the population.
53. Primary health care is a total approach to community development. Which of the
following is an indicator of success in the use of the primary health care approach?
A. Health services are provided free of charge to individuals and families.
B. Local officials are empowered as the major decision makers in matters of health.
C. Health workers are able to provide care based on identified health needs of the
people.
D. Health programs are sustained according to the level of development of the
community.
Answer: (D) Health programs are sustained according to the level of
development of the community.
Primary health care is essential health care that can be sustained in all stages of
development of the community.
54. Sputum examination is the major screening tool for pulmonary tuberculosis. Clients
would sometimes get false negative results in this exam. This means that the test is not
perfect in terms of which characteristic of a diagnostic examination?
A. Effectiveness
B. Efficacy
C. Specificity

D. Sensitivity
Answer: (D) Sensitivity
Sensitivity is the capacity of a diagnostic examination to detect cases of the disease. If a
test is 100% sensitive, all the cases tested will have a positive result, i.e., there will be no
false negative results.
55. Use of appropriate technology requires knowledge of indigenous technology. Which
medicinal herb is given for fever, headache and cough?
A. Sambong
B. Tsaang gubat
C. Akapulko
D. Lagundi
Answer: (D) Lagundi
Sambong is used as a diuretic. Tsaang gubat is used to relieve diarrhea. Akapulko is used
for its antifungal property.
56. What law created the Philippine Institute of Traditional and Alternative Health
Care?
A. R.A. 8423
B. R.A. 4823
C. R.A. 2483
D. R.A. 3482
Answer: (A) R.A. 8423
(none)
57. In traditional Chinese medicine, the yielding, negative and feminine force is termed
A. Yin
B. Yang
C. Qi
D. Chai
Answer: (A) Yin
Yang is the male dominating, positive and masculine force.
58. What is the legal basis for Primary Health Care approach in the Philippines?
A. Alma Ata Declaration on PHC
B. Letter of Instruction No. 949
C. Presidential Decree No. 147
D. Presidential Decree 996
Answer: (B) Letter of Instruction No. 949
Letter of Instruction 949 was issued by then President Ferdinand Marcos, directing the
formerly called Ministry of Health, now the Department of Health, to utilize Primary
Health Care approach in planning and implementing health programs.

59. Which of the following demonstrates intersectoral linkages?


A. Two-way referral system
B. Team approach
C. Endorsement done by a midwife to another midwife
D. Cooperation between the PHN and public school teacher
Answer: (D) Cooperation between the PHN and public school teacher
Intersectoral linkages refer to working relationships between the health sector and other
sectors involved in community development.
60. The municipality assigned to you has a population of about 20,000. Estimate the
number of 1-4 year old children who will be given Retinol capsule 200,000 I.U. every 6
months.
A. 1,500
B. 1,800
C. 2,000
D. 2,300
Answer: (D) 2,300
Based on the Philippine population composition, to estimate the number of 1-4 year old
children, multiply total population by 11.5%.
61. Estimate the number of pregnant women who will be given tetanus toxoid during an
immunization outreach activity in a barangay with a population of about 1,500.
A. 265
B. 300
C. 375
D. 400
Answer: (A) 265
To estimate the number of pregnant women, multiply the total population by 3.5%.
62. To describe the sex composition of the population, which demographic tool may be
used?
A. Sex ratio
B. Sex proportion
C. Population pyramid
D. Any of these may be used.
Answer: (D) Any of these may be used.
Sex ratio and sex proportion are used to determine the sex composition of a population.
A population pyramid is used to present the composition of a population by age and sex.
63. Which of the following is a natality rate?
A. Crude birth rate
B. Neonatal mortality rate

C. Infant mortality rate


D. General fertility rate
Answer: (A) Crude birth rate
Natality means birth. A natality rate is a birth rate.
64. You are computing the crude death rate of your municipality, with a total population
of about 18,000, for last year. There were 94 deaths. Among those who died, 20 died
because of diseases of the heart and 32 were aged 50 years or older. What is the crude
death rate?
A. 4.2/1,000
B. 5.2/1,000
C. 6.3/1,000
D. 7.3/1,000
Answer: (B) 5.2/1,000
To compute crude death rate divide total number of deaths (94) by total population
(18,000) and multiply by 1,000.
65. Knowing that malnutrition is a frequent community health problem, you decided to
conduct nutritional assessment. What population is particularly susceptible to protein
energy malnutrition (PEM)?
A. Pregnant women and the elderly
B. Under-5 year old children
C. 1-4 year old children
D. School age children
Answer: (C) 1-4 year old children
Preschoolers are the most susceptible to PEM because they have generally been weaned.
Also, this is the population who, unable to feed themselves, are often the victims of poor
intrafamilial food distribution.
66. Which statistic can give the most accurate reflection of the health status of a
community?
A. 1-4 year old age-specific mortality rate
B. Infant mortality rate
C. Swaroops index
D. Crude death rate
Answer: (C) Swaroops index
Swaroops index is the proportion of deaths aged 50 years and above. The higher the
Swaroops index of a population, the greater the proportion of the deaths who were able
to reach the age of at least 50 years, i.e., more people grew old before they died.
67. In the past year, Barangay A had an average population of 1655. 46 babies were born
in that year, 2 of whom died less than 4 weeks after they were born. There were 4
recorded stillbirths. What is the neonatal mortality rate?
A. 27.8/1,000

B. 43.5/1,000
C. 86.9/1,000
D. 130.4/1,000
Answer: (B) 43.5/1,000
To compute for neonatal mortality rate, divide the number of babies who died before
reaching the age of 28 days by the total number of live births, then multiply by 1,000.
68. Which statistic best reflects the nutritional status of a population?
A. 1-4 year old age-specific mortality rate
B. Proportionate mortality rate
C. Infant mortality rate
D. Swaroops index
Answer: (A) 1-4 year old age-specific mortality rate
Since preschoolers are the most susceptible to the effects of malnutrition, a population
with poor nutritional status will most likely have a high 1-4 year old age-specific
mortality rate, also known as child mortality rate.
69. What numerator is used in computing general fertility rate?
A. Estimated midyear population
B. Number of registered live births
C. Number of pregnancies in the year
D. Number of females of reproductive age
Answer: (B) Number of registered live births
To compute for general or total fertility rate, divide the number of registered live births
by the number of females of reproductive age (15-45 years), then multiply by 1,000.
70. You will gather data for nutritional assessment of a purok. You will gather
information only from families with members who belong to the target population for
PEM. What method of data gathering is best for this purpose?
A. Census
B. Survey
C. Record review
D. Review of civil registry
Answer: (B) Survey
A survey, also called sample survey, is data gathering about a sample of the population.
71. In the conduct of a census, the method of population assignment based on the actual
physical location of the people is termed
A. De jure
B. De locus
C. De facto
D. De novo

Answer: (C) De facto


The other method of population assignment, de jure, is based on the usual place of
residence of the people.
72. The Field Health Services and Information System (FHSIS) is the recording and
reporting system in public health care in the Philippines. The Monthly Field Health
Service Activity Report is a form used in which of the components of the FHSIS?
A. Tally report
B. Output report
C. Target/client list
D. Individual health record
Answer: (A) Tally report
A tally report is prepared monthly or quarterly by the RHU personnel and transmitted
to the Provincial Health Office.
73. To monitor clients registered in long-term regimens, such as the Multi-Drug
Therapy, which component will be most useful?
A. Tally report
B. Output report
C. Target/client list
D. Individual health record
Answer: (C) Target/client list
The MDT Client List is a record of clients enrolled in MDT and other relevant data, such
as dates when clients collected their monthly supply of drugs.
74. Civil registries are important sources of data. Which law requires registration of
births within 30 days from the occurrence of the birth?
A. P.D. 651
B. Act 3573
C. R.A. 3753
D. R.A. 3375
Answer: (A) P.D. 651
P.D. 651 amended R.A. 3753, requiring the registry of births within 30 days from their
occurrence.
75. Which of the following professionals can sign the birth certificate?
A. Public health nurse
B. Rural health midwife
C. Municipal health officer
D. Any of these health professionals
Answer: (D) Any of these health professionals
D. R.A. 3753 states that any birth attendant may sign the certificate of live birth.

76. Which criterion in priority setting of health problems is used only in community
health care?
A. Modifiability of the problem
B. Nature of the problem presented
C. Magnitude of the health problem
D. Preventive potential of the health problem
Answer: (C) Magnitude of the health problem
Magnitude of the problem refers to the percentage of the population affected by a health
problem. The other choices are criteria considered in both family and community health
care.
77. The Sentrong Sigla Movement has been launched to improve health service delivery.
Which of the following is/are true of this movement?
A. This is a project spearheaded by local government units.
B. It is a basis for increasing funding from local government units.
C. It encourages health centers to focus on disease prevention and control.
D. Its main strategy is certification of health centers able to comply with standards.
Answer: (D) Its main strategy is certification of health centers able to
comply with standards.
Sentrong Sigla Movement is a joint project of the DOH and local government units. Its
main strategy is certification of health centers that are able to comply with standards set
by the DOH.
78. Which of the following women should be considered as special targets for family
planning?
A. Those who have two children or more
B. Those with medical conditions such as anemia
C. Those younger than 20 years and older than 35 years
D. Those who just had a delivery within the past 15 months
Answer: (D) Those who just had a delivery within the past 15 months
The ideal birth spacing is at least two years. 15 months plus 9 months of pregnancy = 2
years.
79. Freedom of choice is one of the policies of the Family Planning Program of the
Philippines. Which of the following illustrates this principle?
A. Information dissemination about the need for family planning
B. Support of research and development in family planning methods
C. Adequate information for couples regarding the different methods
D. Encouragement of couples to take family planning as a joint responsibility
Answer: (C) Adequate information for couples regarding the different
methods
To enable the couple to choose freely among different methods of family planning, they

must be given full information regarding the different methods that are available to
them, considering the availability of quality services that can support their choice.
80. A woman, 6 months pregnant, came to the center for consultation. Which of the
following substances is contraindicated?
A. Tetanus toxoid
B. Retinol 200,000 IU
C. Ferrous sulfate 200 mg
D. Potassium iodate 200 mg. capsule
Answer: (B) Retinol 200,000 IU
Retinol 200,000 IU is a form of megadose Vitamin A. This may have a teratogenic
effect.
81. During prenatal consultation, a client asked you if she can have her delivery at home.
After history taking and physical examination, you advised her against a home delivery.
Which of the following findings disqualifies her for a home delivery?
A. Her OB score is G5P3.
B. She has some palmar pallor.
C. Her blood pressure is 130/80.
D. Her baby is in cephalic presentation.
Answer: (A) Her OB score is G5P3.
Only women with less than 5 pregnancies are qualified for a home delivery. It is also
advisable for a primigravida to have delivery at a childbirth facility.
82. Inadequate intake by the pregnant woman of which vitamin may cause neural tube
defects?
A. Niacin
B. Riboflavin
C. Folic acid
D. Thiamine
Answer: (C) Folic acid
It is estimated that the incidence of neural tube defects can be reduced drastically if
pregnant women have an adequate intake of folic acid.
83. You are in a clients home to attend to a delivery. Which of the following will you do
first?
A. Set up the sterile area.
B. Put on a clean gown or apron.
C. Cleanse the clients vulva with soap and water.
D. Note the interval, duration and intensity of labor contractions.
Answer: (D) Note the interval, duration and intensity of labor contractions.
Assessment of the woman should be done first to determine whether she is having true
labor and, if so, what stage of labor she is in.

84. In preparing a primigravida for breastfeeding, which of the following will you do?
A. Tell her that lactation begins within a day after delivery.
B. Teach her nipple stretching exercises if her nipples are everted.
C. Instruct her to wash her nipples before and after each breastfeeding.
D. Explain to her that putting the baby to breast will lessen blood loss after delivery.
Answer: (D) Explain to her that putting the baby to breast will lessen blood
loss after delivery.
Suckling of the nipple stimulates the release of oxytocin by the posterior pituitary gland,
which causes uterine contraction. Lactation begins 1 to 3 days after delivery. Nipple
stretching exercises are done when the nipples are flat or inverted. Frequent washing
dries up the nipples, making them prone to the formation of fissures.
85. A primigravida is instructed to offer her breast to the baby for the first time within
30 minutes after delivery. What is the purpose of offering the breast this early?
A. To initiate the occurrence of milk letdown
B. To stimulate milk production by the mammary acini
C. To make sure that the baby is able to get the colostrum
D. To allow the woman to practice breastfeeding in the presence of the health worker
Answer: (B) To stimulate milk production by the mammary acini
Suckling of the nipple stimulates prolactin reflex (the release of prolactin by the anterior
pituitary gland), which initiates lactation.
86. In a mothers class, you discuss proper breastfeeding technique. Which is of these is
a sign that the baby has latched on to the breast properly?
A. The baby takes shallow, rapid sucks.
B. The mother does not feel nipple pain.
C. The babys mouth is only partly open.
D. Only the mothers nipple is inside the babys mouth.
Answer: (B) The mother does not feel nipple pain.
When the baby has properly latched on to the breast, he takes deep, slow sucks; his
mouth is wide open; and much of the areola is inside his mouth. And, youre right! The
mother does not feel nipple pain.
87. You explain to a breastfeeding mother that breast milk is sufficient for all of the
babys nutrient needs only up to ____.
A. 3 months
B. 6 months
C. 1 year
D. 2 years
Answer: (B) 6 months
After 6 months, the babys nutrient needs, especially the babys iron requirement, can no
longer be provided by mothers milk alone.

88. What is given to a woman within a month after the delivery of a baby?
A. Malunggay capsule
B. Ferrous sulfate 100 mg. OD
C. Retinol 200,000 I.U., 1 capsule
D. Potassium iodate 200 mg, 1 capsule
Answer: (C) Retinol 200,000 I.U., 1 capsule
A capsule of Retinol 200,000 IU is given within 1 month after delivery. Potassium
iodate is given during pregnancy; malunggay capsule is not routinely administered after
delivery; and ferrous sulfate is taken for two months after delivery.
89. Which biological used in Expanded Program on Immunization (EPI) is stored in the
freezer?
A. DPT
B. Tetanus toxoid
C. Measles vaccine
D. Hepatitis B vaccine
Answer: (C) Measles vaccine
Among the biologicals used in the Expanded Program on Immunization, measles
vaccine and OPV are highly sensitive to heat, requiring storage in the freezer.
90. Unused BCG should be discarded how many hours after reconstitution?
A. 2
B. 4
C. 6
D. At the end of the day
Answer: (B) 4
While the unused portion of other biologicals in EPI may be given until the end of the
day, only BCG is discarded 4 hours after reconstitution. This is why BCG immunization
is scheduled only in the morning.
91. In immunizing school entrants with BCG, you are not obliged to secure parental
consent. This is because of which legal document?
A. P.D. 996
B. R.A. 7846
C. Presidential Proclamation No. 6
D. Presidential Proclamation No. 46
Answer: (A) P.D. 996
Presidential Decree 996, enacted in 1976, made immunization in the EPI compulsory for
children under 8 years of age. Hepatitis B vaccination was made compulsory for the
same age group by R.A. 7846.

92. Which immunization produces a permanent scar?


A. DPT
B. BCG
C. Measles vaccination
D. Hepatitis B vaccination
Answer: (B) BCG
BCG causes the formation of a superficial abscess, which begins 2 weeks after
immunization. The abscess heals without treatment, with the formation of a permanent
scar.
93. A 4-week old baby was brought to the health center for his first immunization.
Which can be given to him?
A. DPT1
B. OPV1
C. Infant BCG
D. Hepatitis B vaccine 1
Answer: (C) Infant BCG
Infant BCG may be given at birth. All the other immunizations mentioned can be given
at 6 weeks of age.
94. You will not give DPT 2 if the mother says that the infant had
A. Seizures a day after DPT 1.
B. Fever for 3 days after DPT 1.
C. Abscess formation after DPT 1.
D. Local tenderness for 3 days after DPT 1.
Answer: (A) Seizures a day after DPT 1.
Seizures within 3 days after administration of DPT is an indication of hypersensitivity to
pertussis vaccine, a component of DPT. This is considered a specific contraindication to
subsequent doses of DPT.
95. A 2-month old infant was brought to the health center for immunization. During
assessment, the infants temperature registered at 38.1C. Which is the best course of
action that you will take?
A. Go on with the infants immunizations.
B. Give Paracetamol and wait for his fever to subside.
C. Refer the infant to the physician for further assessment.
D. Advise the infants mother to bring him back for immunization when he is well.
Answer: (A) Go on with the infants immunizations.
In the EPI, fever up to 38.5C is not a contraindication to immunization. Mild acute
respiratory tract infection, simple diarrhea and malnutrition are not contraindications
either.

96. A pregnant woman had just received her 4th dose of tetanus toxoid. Subsequently,
her baby will have protection against tetanus for how long?
A. 1 year
B. 3 years
C. 10 years
D. Lifetime
Answer: (A) 1 year
The baby will have passive natural immunity by placental transfer of antibodies. The
mother will have active artificial immunity lasting for about 10 years. 5 doses will give
the mother lifetime protection.
97. A 4-month old infant was brought to the health center because of cough. Her
respiratory rate is 42/minute. Using the Integrated Management of Child Illness (IMCI)
guidelines of assessment, her breathing is considered
A. Fast
B. Slow
C. Normal
D. Insignificant
Answer: (C) Normal
In IMCI, a respiratory rate of 50/minute or more is fast breathing for an infant aged 2 to
12 months.
98. Which of the following signs will indicate that a young child is suffering from severe
pneumonia?
A. Dyspnea
B. Wheezing
C. Fast breathing
D. Chest indrawing
Answer: (D) Chest indrawing
In IMCI, chest indrawing is used as the positive sign of dyspnea, indicating severe
pneumonia.
99. Using IMCI guidelines, you classify a child as having severe pneumonia. What is the
best management for the child?
A. Prescribe an antibiotic.
B. Refer him urgently to the hospital.
C. Instruct the mother to increase fluid intake.
D. Instruct the mother to continue breastfeeding.
Answer: (B) Refer him urgently to the hospital.
Severe pneumonia requires urgent referral to a hospital. Answers A, C and D are done
for a client classified as having pneumonia.

100. A 5-month old infant was brought by his mother to the health center because of
diarrhea occurring 4 to 5 times a day. His skin goes back slowly after a skin pinch and
his eyes are sunken. Using the IMCI guidelines, you will classify this infant in which
category?
A. No signs of dehydration
B. Some dehydration
C. Severe dehydration
D. The data is insufficient.
Answer: (B) Some dehydration
Using the assessment guidelines of IMCI, a child (2 months to 5 years old) with diarrhea
is classified as having SOME DEHYDRATION if he shows 2 or more of the following
signs: restless or irritable, sunken eyes, the skin goes back slow after a skin pinch.
101. Based on assessment, you classified a 3-month old infant with the chief complaint
of diarrhea in the category of SOME DEHYDRATION. Based on IMCI management
guidelines, which of the following will you do?
A. Bring the infant to the nearest facility where IV fluids can be given.
B. Supervise the mother in giving 200 to 400 ml. of Oresol in 4 hours.
C. Give the infants mother instructions on home management.
D. Keep the infant in your health center for close observation.
Answer: (B) Supervise the mother in giving 200 to 400 ml. of Oresol in 4
hours.
In the IMCI management guidelines, SOME DEHYDRATION is treated with the
administration of Oresol within a period of 4 hours. The amount of Oresol is best
computed on the basis of the childs weight (75 ml/kg body weight). If the weight is
unknown, the amount of Oresol is based on the childs age.
102. A mother is using Oresol in the management of diarrhea of her 3-year old child. She
asked you what to do if her child vomits. You will tell her to
A. Bring the child to the nearest hospital for further assessment.
B. Bring the child to the health center for intravenous fluid therapy.
C. Bring the child to the health center for assessment by the physician.
D. Let the child rest for 10 minutes then continue giving Oresol more slowly.
Answer: (D) Let the child rest for 10 minutes then continue giving Oresol
more slowly.
If the child vomits persistently, that is, he vomits everything that he takes in, he has to
be referred urgently to a hospital. Otherwise, vomiting is managed by letting the child
rest for 10 minutes and then continuing with Oresol administration. Teach the mother
to give Oresol more slowly.
103. A 1 year old child was classified as having 3rd degree protein energy
malnutrition, kwashiorkor. Which of the following signs will be most apparent in this
child?
A. Voracious appetite
B. Wasting

C. Apathy
D. Edema
Answer: (D) Edema
Edema, a major sign of kwashiorkor, is caused by decreased colloidal osmotic pressure
of the blood brought about by hypoalbuminemia. Decreased blood albumin level is due a
protein-deficient diet.
104. Assessment of a 2-year old child revealed baggy pants. Using the IMCI guidelines,
how will you manage this child?
A. Refer the child urgently to a hospital for confinement.
B. Coordinate with the social worker to enroll the child in a feeding program.
C. Make a teaching plan for the mother, focusing on menu planning for her child.
D. Assess and treat the child for health problems like infections and intestinal
parasitism.
Answer: (A) Refer the child urgently to a hospital for confinement.
Baggy pants is a sign of severe marasmus. The best management is urgent referral to a
hospital.
105. During the physical examination of a young child, what is the earliest sign of
xerophthalmia that you may observe?
A. Keratomalacia
B. Corneal opacity
C. Night blindness
D. Conjunctival xerosis
Answer: (D) Conjunctival xerosis
The earliest sign of Vitamin A deficiency (xerophthalmia) is night blindness. However,
this is a functional change, which is not observable during physical examination.The
earliest visible lesion is conjunctival xerosis or dullness of the conjunctiva due to
inadequate tear production.
106. To prevent xerophthalmia, young children are given Retinol capsule every 6
months. What is the dose given to preschoolers?
A. 10,000 IU
B. 20,000 IU
C. 100,000 IU
D. 200,000 IU
Answer: (D) 200,000 IU
Preschoolers are given Retinol 200,000 IU every 6 months. 100,000 IU is given once to
infants aged 6 to 12 months. The dose for pregnant women is 10,000 IU.
107. The major sign of iron deficiency anemia is pallor. What part is best examined for
pallor?
A. Palms

B. Nailbeds
C. Around the lips
D. Lower conjunctival sac
Answer: (A) Palms
The anatomic characteristics of the palms allow a reliable and convenient basis for
examination for pallor.
108. Food fortification is one of the strategies to prevent micronutrient deficiency
conditions. R.A. 8976 mandates fortification of certain food items. Which of the
following is among these food items?
A. Sugar
B. Bread
C. Margarine
D. Filled milk
Answer: (A) Sugar
R.A. 8976 mandates fortification of rice, wheat flour, sugar and cooking oil with Vitamin
A, iron and/or iodine.
109. What is the best course of action when there is a measles epidemic in a nearby
municipality?
A. Give measles vaccine to babies aged 6 to 8 months.
B. Give babies aged 6 to 11 months one dose of 100,000 I.U. of Retinol
C. Instruct mothers to keep their babies at home to prevent disease transmission.
D. Instruct mothers to feed their babies adequately to enhance their babies resistance.
Answer: (A) Give measles vaccine to babies aged 6 to 8 months.
Ordinarily, measles vaccine is given at 9 months of age. During an impending epidemic,
however, one dose may be given to babies aged 6 to 8 months. The mother is instructed
that the baby needs another dose when the baby is 9 months old.
110. A mother brought her daughter, 4 years old, to the RHU because of cough and
colds. Following the IMCI assessment guide, which of the following is a danger sign that
indicates the need for urgent referral to a hospital?
A. Inability to drink
B. High grade fever
C. Signs of severe dehydration
D. Cough for more than 30 days
Answer: (A) Inability to drink
A sick child aged 2 months to 5 years must be referred urgently to a hospital if he/she
has one or more of the following signs: not able to feed or drink, vomits everything,
convulsions, abnormally sleepy or difficult to awaken.
111. Management of a child with measles includes the administration of which of the
following?
A. Gentian violet on mouth lesions

B. Antibiotics to prevent pneumonia


C. Tetracycline eye ointment for corneal opacity
D. Retinol capsule regardless of when the last dose was given
Answer: (D) Retinol capsule regardless of when the last dose was given
An infant 6 to 12 months classified as a case of measles is given Retinol 100,000 IU; a
child is given 200,000 IU regardless of when the last dose was given.
112. A mother brought her 10 month old infant for consultation because of fever, which
started 4 days prior to consultation. To determine malaria risk, what will you do?
A. Do a tourniquet test.
B. Ask where the family resides.
C. Get a specimen for blood smear.
D. Ask if the fever is present everyday.
Answer: (B) Ask where the family resides.
Because malaria is endemic, the first question to determine malaria risk is where the
clients family resides. If the area of residence is not a known endemic area, ask if the
child had traveled within the past 6 months, where he/she was brought and whether
he/she stayed overnight in that area.
113. The following are strategies implemented by the Department of Health to prevent
mosquito-borne diseases. Which of these is most effective in the control of Dengue
fever?
A. Stream seeding with larva-eating fish
B. Destroying breeding places of mosquitoes
C. Chemoprophylaxis of non-immune persons going to endemic areas
D. Teaching people in endemic areas to use chemically treated mosquito nets
Answer: (B) Destroying breeding places of mosquitoes
Aedes aegypti, the vector of Dengue fever, breeds in stagnant, clear water. Its feeding
time is usually during the daytime. It has a cyclical pattern of occurrence, unlike malaria
which is endemic in certain parts of the country.
114. Secondary prevention for malaria includes
A. Planting of neem or eucalyptus trees
B. Residual spraying of insecticides at night
C. Determining whether a place is endemic or not
D. Growing larva-eating fish in mosquito breeding places
Answer: (C) Determining whether a place is endemic or not
This is diagnostic and therefore secondary level prevention. The other choices are for
primary prevention.
115. Scotch tape swab is done to check for which intestinal parasite?
A. Ascaris
B. Pinworm

C. Hookworm
D. Schistosoma
Answer: (B) Pinworm
Pinworm ova are deposited around the anal orifice.
116. Which of the following signs indicates the need for sputum examination for AFB?
A. Hematemesis
B. Fever for 1 week
C. Cough for 3 weeks
D. Chest pain for 1 week
Answer: (C) Cough for 3 weeks
A client is considered a PTB suspect when he has cough for 2 weeks or more, plus one or
more of the following signs: fever for 1 month or more; chest pain lasting for 2 weeks or
more not attributed to other conditions; progressive, unexplained weight loss; night
sweats; and hemoptysis.
117. Which clients are considered targets for DOTS Category I?
A. Sputum negative cavitary cases
B. Clients returning after a default
C. Relapses and failures of previous PTB treatment regimens
D. Clients diagnosed for the first time through a positive sputum exam
Answer: (D) Clients diagnosed for the first time through a positive sputum
exam
Category I is for new clients diagnosed by sputum examination and clients diagnosed to
have a serious form of extrapulmonary tuberculosis, such as TB osteomyelitis.
118. To improve compliance to treatment, what innovation is being implemented in
DOTS?
A. Having the health worker follow up the client at home
B. Having the health worker or a responsible family member monitor drug intake
C. Having the patient come to the health center every month to get his medications
D. Having a target list to check on whether the patient has collected his monthly supply
of drugs
Answer: (B) Having the health worker or a responsible family member
monitor drug intake
Directly Observed Treatment Short Course is so-called because a treatment partner,
preferably a health worker accessible to the client, monitors the clients compliance to
the treatment.
119. Diagnosis of leprosy is highly dependent on recognition of symptoms. Which of the
following is an early sign of leprosy?
A. Macular lesions
B. Inability to close eyelids

C. Thickened painful nerves


D. Sinking of the nosebridge
Answer: (C) Thickened painful nerves
The lesion of leprosy is not macular. It is characterized by a change in skin color (either
reddish or whitish) and loss of sensation, sweating and hair growth over the lesion.
Inability to close the eyelids (lagophthalmos) and sinking of the nosebridge are late
symptoms.
120. Which of the following clients should be classified as a case of multibacillary
leprosy?
A. 3 skin lesions, negative slit skin smear
B. 3 skin lesions, positive slit skin smear
C. 5 skin lesions, negative slit skin smear
D. 5 skin lesions, positive slit skin smear
Answer: (D) 5 skin lesions, positive slit skin smear
A multibacillary leprosy case is one who has a positive slit skin smear and at least 5 skin
lesions.
121. In the Philippines, which condition is the most frequent cause of death associated
with schistosomiasis?
A. Liver cancer
B. Liver cirrhosis
C. Bladder cancer
D. Intestinal perforation
Answer: (B) Liver cirrhosis
The etiologic agent of schistosomiasis in the Philippines is Schistosoma japonicum,
which affects the small intestine and the liver. Liver damage is a consequence of fibrotic
reactions to schistosoma eggs in the liver.
122. What is the most effective way of controlling schistosomiasis in an endemic area?
A. Use of molluscicides
B. Building of foot bridges
C. Proper use of sanitary toilets
D. Use of protective footwear, such as rubber boots
Answer: (C) Proper use of sanitary toilets
The ova of the parasite get out of the human body together with feces. Cutting the cycle
at this stage is the most effective way of preventing the spread of the disease to
susceptible hosts.
123. When residents obtain water from an artesian well in the neighborhood, the level of
this approved type of water facility is
A. I
B. II

C. III
D. IV
Answer: (B) II
A communal faucet or water standpost is classified as Level II.
124. For prevention of hepatitis A, you decided to conduct health education activities.
Which of the following is IRRELEVANT?
A. Use of sterile syringes and needles
B. Safe food preparation and food handling by vendors
C. Proper disposal of human excreta and personal hygiene
D. Immediate reporting of water pipe leaks and illegal water connections
Answer: (A) Use of sterile syringes and needles
Hepatitis A is transmitted through the fecal oral route. Hepatitis B is transmitted
through infected body secretions like blood and semen.
126. Which biological used in Expanded Program on Immunization (EPI) should NOT
be stored in the freezer?
A. DPT
B. Oral polio vaccine
C. Measles vaccine
D. MMR
Answer: (A) DPT
DPT is sensitive to freezing. The appropriate storage temperature of DPT is 2 to 8 C
only. OPV and measles vaccine are highly sensitive to heat and require freezing. MMR is
not an immunization in the Expanded Program on Immunization.
127. You will conduct outreach immunization in a barangay with a population of about
1500. Estimate the number of infants in the barangay.
A. 45
B. 50
C. 55
D. 60
Answer: (A) 45
To estimate the number of infants, multiply total population by 3%.
128. In Integrated Management of Childhood Illness, severe conditions generally
require urgent referral to a hospital. Which of the following severe conditions DOES
NOT always require urgent referral to a hospital?
A. Mastoiditis
B. Severe dehydration
C. Severe pneumonia
D. Severe febrile disease

Answer: (B) Severe dehydration


The order of priority in the management of severe dehydration is as follows: intravenous
fluid therapy, referral to a facility where IV fluids can be initiated within 30 minutes,
Oresol/nasogastric tube, Oresol/orem. When the foregoing measures are not possible or
effective, tehn urgent referral to the hospital is done.
129. A client was diagnosed as having Dengue fever. You will say that there is slow
capillary refill when the color of the nailbed that you pressed does not return within how
many seconds?
A. 3
B. 5
C. 8
D. 10
Answer: (A) 3
Adequate blood supply to the area allows the return of the color of the nailbed within 3
seconds.
130. A 3-year old child was brought by his mother to the health center because of fever
of 4-day duration. The child had a positive tourniquet test result. In the absence of other
signs, which is the most appropriate measure that the PHN may carry out to prevent
Dengue shock syndrome?
A. Insert an NGT and give fluids per NGT.
B. Instruct the mother to give the child Oresol.
C. Start the patient on intravenous fluids STAT.
D. Refer the client to the physician for appropriate management.
Answer: (B) Instruct the mother to give the child Oresol.
Since the child does not manifest any other danger sign, maintenance of fluid balance
and replacement of fluid loss may be done by giving the client Oresol.
131. The pathognomonic sign of measles is Kopliks spot. You may see Kopliks spot by
inspecting the _____.
A. Nasal mucosa
B. Buccal mucosa
C. Skin on the abdomen
D. Skin on the antecubital surface
Answer: (B) Buccal mucosa
Kopliks spot may be seen on the mucosa of the mouth or the throat.
132. Among the following diseases, which is airborne?
A. Viral conjunctivitis
B. Acute poliomyelitis
C. Diphtheria
D. Measles

Answer: (D) Measles


Viral conjunctivitis is transmitted by direct or indirect contact with discharges from
infected eyes. Acute poliomyelitis is spread through the fecal-oral route and contact with
throat secretions, whereas diphtheria is through direct and indirect contact with
respiratory secretions.
133. Among children aged 2 months to 3 years, the most prevalent form of meningitis is
caused by which microorganism?
A. Hemophilus influenzae
B. Morbillivirus
C. Steptococcus pneumoniae
D. Neisseria meningitidis
Answer: (A) Hemophilus influenzae
Hemophilus meningitis is unusual over the age of 5 years. In developing countries, the
peak incidence is in children less than 6 months of age. Morbillivirus is the etiology of
measles. Streptococcus pneumoniae and Neisseria meningitidis may cause meningitis,
but age distribution is not specific in young children.
134. Human beings are the major reservoir of malaria. Which of the following strategies
in malaria control is based on this fact?
A. Stream seeding
B. Stream clearing
C. Destruction of breeding places
D. Zooprophylaxis
Answer: (D) Zooprophylaxis
Zooprophylaxis is done by putting animals like cattle or dogs close to windows or
doorways just before nightfall. The Anopheles mosquito takes his blood meal from the
animal and goes back to its breeding place, thereby preventing infection of humans.
135. The use of larvivorous fish in malaria control is the basis for which strategy of
malaria control?
A. Stream seeding
B. Stream clearing
C. Destruction of breeding places
D. Zooprophylaxis
Answer: (A) Stream seeding
Stream seeding is done by putting tilapia fry in streams or other bodies of water
identified as breeding places of the Anopheles mosquito
136. Mosquito-borne diseases are prevented mostly with the use of mosquito control
measures. Which of the following is NOT appropriate for malaria control?
A. Use of chemically treated mosquito nets
B. Seeding of breeding places with larva-eating fish
C. Destruction of breeding places of the mosquito vector

D. Use of mosquito-repelling soaps, such as those with basil or citronella


Answer: (C) Destruction of breeding places of the mosquito vector
Anopheles mosquitoes breed in slow-moving, clear water, such as mountain streams.
137. A 4-year old client was brought to the health center with the chief complaint of
severe diarrhea and the passage of rice water stools. The client is most probably
suffering from which condition?
A. Giardiasis
B. Cholera
C. Amebiasis
D. Dysentery
Answer: (B) Cholera
Passage of profuse watery stools is the major symptom of cholera. Both amebic and
bacillary dysentery are characterized by the presence of blood and/or mucus in the
stools. Giardiasis is characterized by fat malabsorption and, therefore, steatorrhea.
138. In the Philippines, which specie of schistosoma is endemic in certain regions?
A. S. mansoni
B. S. japonicum
C. S. malayensis
D. S. haematobium
Answer: (B) S. japonicum
S. mansoni is found mostly in Africa and South America; S. haematobium in Africa and
the Middle East; and S. malayensis only in peninsular Malaysia.
139. A 32-year old client came for consultation at the health center with the chief
complaint of fever for a week. Accompanying symptoms were muscle pains and body
malaise. A week after the start of fever, the client noted yellowish discoloration of his
sclera. History showed that he waded in flood waters about 2 weeks before the onset of
symptoms. Based on his history, which disease condition will you suspect?
A. Hepatitis A
B. Hepatitis B
C. Tetanus
D. Leptospirosis
Answer: (D) Leptospirosis
Leptospirosis is transmitted through contact with the skin or mucous membrane with
water or moist soil contaminated with urine of infected animals, like rats.
140. MWSS provides water to Manila and other cities in Metro Manila. This is an
example of which level of water facility?
A. I
B. II
C. III

D. IV
Answer: (C) III
Waterworks systems, such as MWSS, are classified as level III.
141. You are the PHN in the city health center. A client underwent screening for AIDS
using ELISA. His result was positive. What is the best course of action that you may
take?
A. Get a thorough history of the client, focusing on the practice of high risk behaviors.
B. Ask the client to be accompanied by a significant person before revealing the result.
C. Refer the client to the physician since he is the best person to reveal the result to the
client.
D. Refer the client for a supplementary test, such as Western blot, since the ELISA result
may be false.
Answer: (D) Refer the client for a supplementary test, such as Western blot,
since the ELISA result may be false.
A client having a reactive ELISA result must undergo a more specific test, such as
Western blot. A negative supplementary test result means that the ELISA result was
false and that, most probably, the client is not infected.
142. Which is the BEST control measure for AIDS?
A. Being faithful to a single sexual partner
B. Using a condom during each sexual contact
C. Avoiding sexual contact with commercial sex workers
D. Making sure that ones sexual partner does not have signs of AIDS
Answer: (A) Being faithful to a single sexual partner
Sexual fidelity rules out the possibility of getting the disease by sexual contact with
another infected person. Transmission occurs mostly through sexual intercourse and
exposure to blood or tissues.
143. The most frequent causes of death among clients with AIDS are opportunistic
diseases. Which of the following opportunistic infections is characterized by
tonsillopharyngitis?
A. Respiratory candidiasis
B. Infectious mononucleosis
C. Cytomegalovirus disease
D. Pneumocystis carinii pneumonia
Answer: (B) Infectious mononucleosis
Cytomegalovirus disease is an acute viral disease characterized by fever, sore throat and
lymphadenopathy.
144. To determine possible sources of sexually transmitted infections, which is the BEST
method that may be undertaken by the public health nurse?
A. Contact tracing

B. Community survey
C. Mass screening tests
D. Interview of suspects
Answer: (A) Contact tracing
Contact tracing is the most practical and reliable method of finding possible sources of
person-to-person transmitted infections, such as sexually transmitted diseases.
145. Antiretroviral agents, such as AZT, are used in the management of AIDS. Which of
the following is NOT an action expected of these drugs.
A. They prolong the life of the client with AIDS.
B. They reduce the risk of opportunistic infections
C. They shorten the period of communicability of the disease.
D. They are able to bring about a cure of the disease condition.
Answer: (D) They are able to bring about a cure of the disease condition.
There is no known treatment for AIDS. Antiretroviral agents reduce the risk of
opportunistic infections and prolong life, but does not cure the underlying
immunodeficiency.
146. A barangay had an outbreak of German measles. To prevent congenital rubella,
what is the BEST advice that you can give to women in the first trimester of pregnancy
in the barangay?
A. Advice them on the signs of German measles.
B. Avoid crowded places, such as markets and moviehouses.
C. Consult at the health center where rubella vaccine may be given.
D. Consult a physician who may give them rubella immunoglobulin.
Answer: (D) Consult a physician who may give them rubella
immunoglobulin.
Rubella vaccine is made up of attenuated German measles viruses. This is
contraindicated in pregnancy. Immune globulin, a specific prophylactic against German
measles, may be given to pregnant women.
147. You were invited to be the resource person in a training class for food handlers.
Which of the following would you emphasize regarding prevention of staphylococcal
food poisoning?
A. All cooking and eating utensils must be thoroughly washed.
B. Food must be cooked properly to destroy staphylococcal microorganisms.
C. Food handlers and food servers must have a negative stool examination result.
D. Proper handwashing during food preparation is the best way of preventing the
condition.
Answer: (D) Proper handwashing during food preparation is the best way
of preventing the condition.
Symptoms of this food poisoning are due to staphylococcal enterotoxin, not the

microorganisms themselves. Contamination is by food handling by persons with


staphylococcal skin or eye infections.
148. In a mothers class, you discussed childhood diseases such as chicken pox. Which of
the following statements about chicken pox is correct?
A. The older one gets, the more susceptible he becomes to the complications of chicken
pox.
B. A single attack of chicken pox will prevent future episodes, including conditions such
as shingles.
C. To prevent an outbreak in the community, quarantine may be imposed by health
authorities.
D. Chicken pox vaccine is best given when there is an impending outbreak in the
community.
Answer: (A) The older one gets, the more susceptible he becomes to the
complications of chicken pox.
Chicken pox is usually more severe in adults than in children. Complications, such as
pneumonia, are higher in incidence in adults.
149. Complications to infectious parotitis (mumps) may be serious in which type of
clients?
A. Pregnant women
B. Elderly clients
C. Young adult males
D. Young infants
Answer: (C) Young adult males
Epididymitis and orchitis are possible complications of mumps. In post-adolescent
males, bilateral inflammation of the testes and epididymis may cause sterility.

FUNDAMENTALS OF NURSING QUESTIONS WITH RATIONALES

1. The most appropriate nursing order for a patient who develops dyspnea and
shortness of breath would be
a.
Maintain the patient on strict bed rest at all times
b.
Maintain the patient in an orthopneic position as needed
c.
Administer oxygen by Venturi mask at 24%, as needed
d.
Allow a 1 hour rest period between activities
2.The nurse observes that Mr. Adams begins to have increased difficulty breathing. She
elevates the head of the bed to the high Fowler position, which decreases his respiratory
distress. The nurse documents this breathing as:
a.
Tachypnea
b.
Eupnca
c.
Orthopnea

d.
Hyperventilation
3. The physician orders a platelet count to be performed on Mrs. Smith after breakfast. The
nurse is responsible for:
a.
Instructing the patient about this diagnostic test
b.
Writing the order for this test
c.
Giving the patient breakfast
d.
All of the above
4. Mrs. Mitchell has been given a copy of her diet. The nurse discusses the foods allowed on
a 500-mg low sodium diet. These include:
a.
A ham and Swiss cheese sandwich on whole wheat bread
b.
Mashed potatoes and broiled chicken
c.
A tossed salad with oil and vinegar and olives
d.
Chicken bouillon
5.
The physician orders a maintenance dose of 5,000 units of subcutaneous heparin (an
anticoagulant) daily. Nursing responsibilities for Mrs. Mitchell now include:
a.
Reviewing daily activated partial thromboplastin time (APTT) and prothrombin time.
b.
Reporting an APTT above 45 seconds to the physician
c.
Assessing the patient for signs and symptoms of frank and occult bleeding
d.
All of the above
6. The four main concepts common to nursing that appear in each of the current
conceptual models are:
a.
Person, nursing, environment, medicine
b.
Person, health, nursing, support systems
c.
Person, health, psychology, nursing
d.
Person, environment, health, nursing
7. In Maslows hierarchy of physiologic needs, the human need of greatest priority is:
a.
Love
b.
Elimination
c.
Nutrition
d.
Oxygen
8. The family of an accident victim who has been declared brain-dead seems amenable to
organ donation. What should the nurse do?
a.
Discourage them from making a decision until their grief has eased
b.
Listen to their concerns and answer their questions honestly
c.
Encourage them to sign the consent form right away
d.
Tell them the body will not be available for a wake or funeral
9. A new head nurse on a unit is distressed about the poor staffing on the 11 p.m. to 7 a.m.
shift. What should she do?
a.
Complain to her fellow nurses
b.
Wait until she knows more about the unit
c.
Discuss the problem with her supervisor
d.
Inform the staff that they must volunteer to rotate
10. Which of the following principles of primary nursing has proven the most satisfying to
the patient and nurse?
a.
Continuity of patient care promotes efficient, cost-effective nursing care
b.
Autonomy and authority for planning are best delegated to a nurse who knows the
patient well

c.

Accountability is clearest when one nurse is responsible for the overall plan and its
implementation.
d.
The holistic approach provides for a therapeutic relationship, continuity, and efficient
nursing care.
11. If nurse administers an injection to a patient who refuses that injection, she has
committed:
a.
Assault and battery
b.
Negligence
c.
Malpractice
d.
None of the above
12. If patient asks the nurse her opinion about a particular physicians and the nurse
replies that the physician is incompetent, the nurse could be held liable for:
a.
Slander
b.
Libel
c.
Assault
d.
Respondent superior
13. A registered nurse reaches to answer the telephone on a busy pediatric unit,
momentarily turning away from a 3 month-old infant she has been weighing. The infant
falls off the scale, suffering a skull fracture. The nurse could be charged with:
a.
Defamation
b.
Assault
c.
Battery
d.
Malpractice
14. Which of the following is an example of nursing malpractice?
a.
The nurse administers penicillin to a patient with a documented history of allergy to
the drug. The patient experiences an allergic reaction and has cerebral damage resulting
from anoxia.
b.
The nurse applies a hot water bottle or a heating pad to the abdomen of a patient with
abdominal cramping.
c.
The nurse assists a patient out of bed with the bed locked in position; the patient slips
and fractures his right humerus.
d.
The nurse administers the wrong medication to a patient and the patient vomits. This
information is documented and reported to the physician and the nursing supervisor.
15. Which of the following signs and symptoms would the nurse expect to find when
assessing an Asian patient for postoperative pain following abdominal surgery?
a.
Decreased blood pressure and heart rate and shallow respirations
b.
Quiet crying
c.
Immobility, diaphoresis, and avoidance of deep breathing or coughing
d.
Changing position every 2 hours
16. A patient is admitted to the hospital with complaints of nausea, vomiting, diarrhea,
and severe abdominal pain. Which of the following would immediately alert the nurse
that the patient has bleeding from the GI tract?
a.
Complete blood count
b.
Guaiac test
c.
Vital signs
d.
Abdominal girth
17. The correct sequence for assessing the abdomen is:

a.
b.
c.
d.
18.
a.
b.
c.
d.
19.

Tympanic percussion, measurement of abdominal girth, and inspection


Assessment for distention, tenderness, and discoloration around the umbilicus.
Percussions, palpation, and auscultation
Auscultation, percussion, and palpation
High-pitched gurgles head over the right lower quadrant are:
A sign of increased bowel motility
A sign of decreased bowel motility
Normal bowel sounds
A sign of abdominal cramping
A patient about to undergo abdominal inspection is best placed in which of the
following positions?
a.
Prone
b.
Trendelenburg
c.
Supine
d.
Side-lying
20. For a rectal examination, the patient can be directed to assume which of the following
positions?
a.
Genupecterol
b.
Sims
c.
Horizontal recumbent
d.
All of the above
21. During a Romberg test, the nurse asks the patient to assume which position?
a.
Sitting
b.
Standing
c.
Genupectoral
d.
Trendelenburg
22. If a patients blood pressure is 150/96, his pulse pressure is:
a.
54
b.
96
c.
150
d.
246
23. A patient is kept off food and fluids for 10 hours before surgery. His oral temperature
at 8 a.m. is 99.8 F (37.7 C) This temperature reading probably indicates:
a.
Infection
b.
Hypothermia
c.
Anxiety
d.
Dehydration
24. Which of the following parameters should be checked when assessing respirations?
a.
Rate
b.
Rhythm
c.
Symmetry
d.
All of the above
25. A 38-year old patients vital signs at 8 a.m. are axillary temperature 99.6 F (37.6 C);
pulse rate, 88; respiratory rate, 30. Which findings should be reported?
a.
Respiratory rate only
b.
Temperature only
c.
Pulse rate and temperature

d.
Temperature and respiratory rate
26. All of the following can cause tachycardia except:
a.
Fever
b.
Exercise
c.
Sympathetic nervous system stimulation
d.
Parasympathetic nervous system stimulation
27. Palpating the midclavicular line is the correct technique for assessing
a.
Baseline vital signs
b.
Systolic blood pressure
c.
Respiratory rate
d.
Apical pulse
28. The absence of which pulse may not be a significant finding when a patient is
admitted to the hospital?
a.
Apical
b.
Radial
c.
Pedal
d.
Femoral
29. Which of the following patients is at greatest risk for developing pressure ulcers?
a.
An alert, chronic arthritic patient treated with steroids and aspirin
b.
An 88-year old incontinent patient with gastric cancer who is confined to his bed at
home
c.
An apathetic 63-year old COPD patient receiving nasal oxygen via cannula
d.
A confused 78-year old patient with congestive heart failure (CHF) who requires
assistance to get out of bed.
30. The physician orders the administration of high-humidity oxygen by face mask and
placement of the patient in a high Fowlers position. After assessing Mrs. Paul, the nurse
writes the following nursing diagnosis: Impaired gas exchange related to increased
secretions. Which of the following nursing interventions has the greatest potential for
improving this situation?
a.
Encourage the patient to increase her fluid intake to 200 ml every 2 hours
b.
Place a humidifier in the patients room.
c.
Continue administering oxygen by high humidity face mask
d.
Perform chest physiotheraphy on a regular schedule
31. The most common deficiency seen in alcoholics is:
a.
Thiamine
b.
Riboflavin
c.
Pyridoxine
d.
Pantothenic acid
32. Which of the following statement is incorrect about a patient with dysphagia?
a.
The patient will find pureed or soft foods, such as custards, easier to swallow than
water
b.
Fowlers or semi Fowlers position reduces the risk of aspiration during swallowing
c.
The patient should always feed himself
d.
The nurse should perform oral hygiene before assisting with feeding.
33. To assess the kidney function of a patient with an indwelling urinary (Foley) catheter,
the nurse measures his hourly urine output. She should notify the physician if the urine
output is:

a.
b.
c.
d.
34.
a.
b.
c.
d.
35.

Less than 30 ml/hour


64 ml in 2 hours
90 ml in 3 hours
125 ml in 4 hours
Certain substances increase the amount of urine produced. These include:
Caffeine-containing drinks, such as coffee and cola.
Beets
Urinary analgesics
Kaolin with pectin (Kaopectate)
A male patient who had surgery 2 days ago for head and neck cancer is about to make
his first attempt to ambulate outside his room. The nurse notes that he is steady on his
feet and that his vision was unaffected by the surgery. Which of the following nursing
interventions would be appropriate?
a.
Encourage the patient to walk in the hall alone
b.
Discourage the patient from walking in the hall for a few more days
c.
Accompany the patient for his walk.
d.
Consuit a physical therapist before allowing the patient to ambulate
36. A patient has exacerbation of chronic obstructive pulmonary disease (COPD)
manifested by shortness of breath; orthopnea: thick, tenacious secretions; and a dry
hacking cough. An appropriate nursing diagnosis would be:
a.
Ineffective airway clearance related to thick, tenacious secretions.
b.
Ineffective airway clearance related to dry, hacking cough.
c.
Ineffective individual coping to COPD.
d.
Pain related to immobilization of affected leg.
37. Mrs. Lim begins to cry as the nurse discusses hair loss. The best response would be:
a.
Dont worry. Its only temporary
b.
Why are you crying? I didnt get to the bad news yet
c.
Your hair is really pretty
d.
I know this will be difficult for you, but your hair will grow back after the completion
of chemotheraphy
38. An additional Vitamin C is required during all of the following periods except:
a.
Infancy
b.
Young adulthood
c.
Childhood
d.
Pregnancy
39. A prescribed amount of oxygen s needed for a patient with COPD to prevent:
a.
Cardiac arrest related to increased partial pressure of carbon dioxide in arterial blood
(PaCO2)
b.
Circulatory overload due to hypervolemia
c.
Respiratory excitement
d.
Inhibition of the respiratory hypoxic stimulus
40. After 1 week of hospitalization, Mr. Gray develops hypokalemia. Which of the
following is the most significant symptom of his disorder?
a.
Lethargy
b.
Increased pulse rate and blood pressure
c.
Muscle weakness
d.
Muscle irritability

41.
a.
b.
c.
d.
42.

Which of the following nursing interventions promotes patient safety?


Asses the patients ability to ambulate and transfer from a bed to a chair
Demonstrate the signal system to the patient
Check to see that the patient is wearing his identification band
All of the above
Studies have shown that about 40% of patients fall out of bed despite the use of side
rails; this has led to which of the following conclusions?
a.
Side rails are ineffective
b.
Side rails should not be used
c.
Side rails are a deterrent that prevent a patient from falling out of bed.
d.
Side rails are a reminder to a patient not to get out of bed
43. Examples of patients suffering from impaired awareness include all of the following
except:
a.
A semiconscious or over fatigued patient
b.
A disoriented or confused patient
c.
A patient who cannot care for himself at home
d.
A patient demonstrating symptoms of drugs or alcohol withdrawal
44. The most common injury among elderly persons is:
a.
Atheroscleotic changes in the blood vessels
b.
Increased incidence of gallbladder disease
c.
Urinary Tract Infection
d.
Hip fracture
45. The most common psychogenic disorder among elderly person is:
a.
Depression
b.
Sleep disturbances (such as bizarre dreams)
c.
Inability to concentrate
d.
Decreased appetite
46. Which of the following vascular system changes results from aging?
a.
Increased peripheral resistance of the blood vessels
b.
Decreased blood flow
c.
Increased work load of the left ventricle
d.
All of the above
47. Which of the following is the most common cause of dementia among elderly
persons?
a.
Parkinsons disease
b.
Multiple sclerosis
c.
Amyotrophic lateral sclerosis (Lou Gerhigs disease)
d.
Alzheimers disease
48. The nurses most important legal responsibility after a patients death in a hospital is:
a.
Obtaining a consent of an autopsy
b.
Notifying the coroner or medical examiner
c.
Labeling the corpse appropriately
d.
Ensuring that the attending physician issues the death certification
49. Before rigor mortis occurs, the nurse is responsible for:
a.
Providing a complete bath and dressing change
b.
Placing one pillow under the bodys head and shoulders
c.
Removing the bodys clothing and wrapping the body in a shroud

d.
50.
a.
b.
c.
d.

Allowing the body to relax normally


When a patient in the terminal stages of lung cancer begins to exhibit loss of
consciousness, a major nursing priority is to:
Protect the patient from injury
Insert an airway
Elevate the head of the bed
Withdraw all pain medications
ANSWERS and RATIONALES for FUNDAMENTALS OF NURSING
PRACTICE QUESTIONS

1.

B. When a patient develops dyspnea and shortness of breath, the orthopneic position
encourages maximum chest expansion and keeps the abdominal organs from pressing
against the diaphragm, thus improving ventilation. Bed rest and oxygen by Venturi
mask at 24% would improve oxygenation of the tissues and cells but must be ordered by
a physician. Allowing for rest periods decreases the possibility of hypoxia.
2.
C. Orthopnea is difficulty of breathing except in the upright position. Tachypnea is
rapid respiration characterized by quick, shallow breaths. Eupnea is normal respiration
quiet, rhythmic, and without effort.
3.
C. A platelet count evaluates the number of platelets in the circulating blood volume.
The nurse is responsible for giving the patient breakfast at the scheduled time. The
physician is responsible for instructing the patient about the test and for writing the
order for the test.
4.
B. Mashed potatoes and broiled chicken are low in natural sodium chloride. Ham,
olives, and chicken bouillon contain large amounts of sodium and are contraindicated
on a low sodium diet.
5.
D. All of the identified nursing responsibilities are pertinent when a patient is
receiving heparin. The normal activated partial thromboplastin time is 16 to 25 seconds
and the normal prothrombin time is 12 to 15 seconds; these levels must remain within
two to two and one half the normal levels. All patients receiving anticoagulant therapy
must be observed for signs and symptoms of frank and occult bleeding (including
hemorrhage, hypotension, tachycardia, tachypnea, restlessness, pallor, cold and clammy
skin, thirst and confusion); blood pressure should be measured every 4 hours and the
patient should be instructed to report promptly any bleeding that occurs with tooth
brushing, bowel movements, urination or heavy prolonged menstruation.
6.
D. The focus concepts that have been accepted by all theorists as the focus of nursing
practice from the time of Florence Nightingale include the person receiving nursing
care, his environment, his health on the health illness continuum, and the nursing
actions necessary to meet his needs.
7.
D. Maslow, who defined a need as a satisfaction whose absence causes illness,
considered oxygen to be the most important physiologic need; without it, human life
could not exist. According to this theory, other physiologic needs (including food, water,
elimination, shelter, rest and sleep, activity and temperature regulation) must be met
before proceeding to the next hierarchical levels on psychosocial needs.
8.
B. The brain-dead patients family needs support and reassurance in making a
decision about organ donation. Because transplants are done within hours of death,
decisions about organ donation must be made as soon as possible. However, the familys
concerns must be addressed before members are asked to sign a consent form. The body
of an organ donor is available for burial.
9.
C. Although a new head nurse should initially spend time observing the unit for its
strengths and weakness, she should take action if a problem threatens patient safety. In
this case, the supervisor is the resource person to approach.
10. D. Studies have shown that patients and nurses both respond well to primary nursing
care units. Patients feel less anxious and isolated and more secure because they are
allowed to participate in planning their own care. Nurses feel personal satisfaction,
much of it related to positive feedback from the patients. They also seem to gain a
greater sense of achievement and esprit de corps.

11.

A. Assault is the unjustifiable attempt or threat to touch or injure another person.


Battery is the unlawful touching of another person or the carrying out of threatened
physical harm. Thus, any act that a nurse performs on the patient against his will is
considered assault and battery.
12. A. Oral communication that injures an individuals reputation is considered slander.
Written communication that does the same is considered libel.
13. D. Malpractice is defined as injurious or unprofessional actions that harm another. It
involves professional misconduct, such as omission or commission of an act that a
reasonable and prudent nurse would or would not do. In this example, the standard of
care was breached; a 3-month-old infant should never be left unattended on a scale.
14. A. The three elements necessary to establish a nursing malpractice are nursing error
(administering penicillin to a patient with a documented allergy to the drug), injury
(cerebral damage), and proximal cause (administering the penicillin caused the cerebral
damage). Applying a hot water bottle or heating pad to a patient without a physicians
order does not include the three required components. Assisting a patient out of bed
with the bed locked in position is the correct nursing practice; therefore, the fracture
was not the result of malpractice. Administering an incorrect medication is a nursing
error; however, if such action resulted in a serious illness or chronic problem, the nurse
could be sued for malpractice.
15. C. An Asian patient is likely to hide his pain. Consequently, the nurse must observe
for objective signs. In an abdominal surgery patient, these might include immobility,
diaphoresis, and avoidance of deep breathing or coughing, as well as increased heart
rate, shallow respirations (stemming from pain upon moving the diaphragm and
respiratory muscles), and guarding or rigidity of the abdominal wall. Such a patient is
unlikely to display emotion, such as crying.
16. B. To assess for GI tract bleeding when frank blood is absent, the nurse has two
options: She can test for occult blood in vomitus, if present, or in stool through guaiac
(Hemoccult) test. A complete blood count does not provide immediate results and does
not always immediately reflect blood loss. Changes in vital signs may be cause by factors
other than blood loss. Abdominal girth is unrelated to blood loss.
17. D. Because percussion and palpation can affect bowel motility and thus bowel sounds,
they should follow auscultation in abdominal assessment. Tympanic percussion,
measurement of abdominal girth, and inspection are methods of assessing the
abdomen. Assessing for distention, tenderness and discoloration around the umbilicus
can indicate various bowel-related conditions, such as cholecystitis, appendicitis and
peritonitis.
18. C. Hyperactive sounds indicate increased bowel motility; two or three sounds per
minute indicate decreased bowel motility. Abdominal cramping with hyperactive, high
pitched tinkling bowel sounds can indicate a bowel obstruction.
19. C. The supine position (also called the dorsal position), in which the patient lies on
his back with his face upward, allows for easy access to the abdomen. In the prone
position, the patient lies on his abdomen with his face turned to the side. In the
Trendelenburg position, the head of the bed is tilted downward to 30 to 40 degrees so
that the upper body is lower than the legs. In the lateral position, the patient lies on his
side.
20. D. All of these positions are appropriate for a rectal examination. In the genupectoral
(knee-chest) position, the patient kneels and rests his chest on the table, forming a 90

degree angle between the torso and upper legs. In Sims position, the patient lies on his
left side with the left arm behind the body and his right leg flexed. In the horizontal
recumbent position, the patient lies on his back with legs extended and hips rotated
outward.
21. B. During a Romberg test, which evaluates for sensory or cerebellar ataxia, the patient
must stand with feet together and arms resting at the sidesfirst with eyes open, then
with eyes closed. The need to move the feet apart to maintain this stance is an abnormal
finding.
22. A. The pulse pressure is the difference between the systolic and diastolic blood
pressure readings in this case, 54.
23. D. A slightly elevated temperature in the immediate preoperative or post operative
period may result from the lack of fluids before surgery rather than from infection.
Anxiety will not cause an elevated temperature. Hypothermia is an abnormally low body
temperature.
24. D. The quality and efficiency of the respiratory process can be determined by
appraising the rate, rhythm, depth, ease, sound, and symmetry of respirations.
25. D. Under normal conditions, a healthy adult breathes in a smooth uninterrupted
pattern 12 to 20 times a minute. Thus, a respiratory rate of 30 would be abnormal. A
normal adult body temperature, as measured on an oral thermometer, ranges between
97 and 100F (36.1 and 37.8C); an axillary temperature is approximately one degree
lower and a rectal temperature, one degree higher. Thus, an axillary temperature of
99.6F (37.6C) would be considered abnormal. The resting pulse rate in an adult ranges
from 60 to 100 beats/minute, so a rate of 88 is normal.
26. D. Parasympathetic nervous system stimulation of the heart decreases the heart rate
as well as the force of contraction, rate of impulse conduction and blood flow through
the coronary vessels. Fever, exercise, and sympathetic stimulation all increase the heart
rate.
27. D. The apical pulse (the pulse at the apex of the heart) is located on the midclavicular
line at the fourth, fifth, or sixth intercostal space. Base line vital signs include pulse rate,
temperature, respiratory rate, and blood pressure. Blood pressure is typically assessed
at the antecubital fossa, and respiratory rate is assessed best by observing chest
movement with each inspiration and expiration.
28. C. Because the pedal pulse cannot be detected in 10% to 20% of the population, its
absence is not necessarily a significant finding. However, the presence or absence of the
pedal pulse should be documented upon admission so that changes can be identified
during the hospital stay. Absence of the apical, radial, or femoral pulse is abnormal and
should be investigated.
29. B. Pressure ulcers are most likely to develop in patients with impaired mental status,
mobility, activity level, nutrition, circulation and bladder or bowel control. Age is also a
factor. Thus, the 88-year old incontinent patient who has impaired nutrition (from
gastric cancer) and is confined to bed is at greater risk.
30. A. Adequate hydration thins and loosens pulmonary secretions and also helps to
replace fluids lost from elevated temperature, diaphoresis, dehydration and dyspnea.
High- humidity air and chest physiotherapy help liquefy and mobilize secretions.
31. A. Chronic alcoholism commonly results in thiamine deficiency and other symptoms
of malnutrition.

32.

C. A patient with dysphagia (difficulty swallowing) requires assistance with feeding.


Feeding himself is a long-range expected outcome. Soft foods, Fowlers or semi-Fowlers
position, and oral hygiene before eating should be part of the feeding regimen.
33. A. A urine output of less than 30ml/hour indicates hypovolemia or oliguria, which is
related to kidney function and inadequate fluid intake.
34. A. Fluids containing caffeine have a diuretic effect. Beets and urinary analgesics, such
as pyridium, can color urine red. Kaopectate is an anti diarrheal medication.
35. C. A hospitalized surgical patient leaving his room for the first time fears rejection
and others staring at him, so he should not walk alone. Accompanying him will offer
moral support, enabling him to face the rest of the world. Patients should begin
ambulation as soon as possible after surgery to decrease complications and to regain
strength and confidence. Waiting to consult a physical therapist is unnecessary.
36. A. Thick, tenacious secretions, a dry, hacking cough, orthopnea, and shortness of
breath are signs of ineffective airway clearance. Ineffective airway clearance related to
dry, hacking cough is incorrect because the cough is not the reason for the ineffective
airway clearance. Ineffective individual coping related to COPD is wrong because the
etiology for a nursing diagnosis should not be a medical diagnosis (COPD) and because
no data indicate that the patient is coping ineffectively. Pain related to immobilization of
affected leg would be an appropriate nursing diagnosis for a patient with a leg fracture.
37. D. I know this will be difficult acknowledges the problem and suggests a resolution
to it. Dont worry.. offers some relief but doesnt recognize the patients feelings. ..I
didnt get to the bad news yet would be inappropriate at any time. Your hair is really
pretty offers no consolation or alternatives to the patient.
38. B. Additional Vitamin C is needed in growth periods, such as infancy and childhood,
and during pregnancy to supply demands for fetal growth and maternal tissues. Other
conditions requiring extra vitamin C include wound healing, fever, infection and stress.
39. D. Delivery of more than 2 liters of oxygen per minute to a patient with chronic
obstructive pulmonary disease (COPD), who is usually in a state of compensated
respiratory acidosis (retaining carbon dioxide (CO2)), can inhibit the hypoxic stimulus
for respiration. An increased partial pressure of carbon dioxide in arterial blood
(PACO2) would not initially result in cardiac arrest. Circulatory overload and
respiratory excitement have no relevance to the question.
40. C. Presenting symptoms of hypokalemia ( a serum potassium level below 3.5
mEq/liter) include muscle weakness, chronic fatigue, and cardiac dysrhythmias. The
combined effects of inadequate food intake and prolonged diarrhea can deplete the
potassium stores of a patient with GI problems.
41. D. Assisting a patient with ambulation and transfer from a bed to a chair allows the
nurse to evaluate the patients ability to carry out these functions safely. Demonstrating
the signal system and providing an opportunity for a return demonstration ensures that
the patient knows how to operate the equipment and encourages him to call for
assistance when needed. Checking the patients identification band verifies the patients
identity and prevents identification mistakes in drug administration.
42. D. Since about 40% of patients fall out of bed despite the use of side rails, side rails
cannot be said to prevent falls; however, they do serve as a reminder that the patient
should not get out of bed. The other answers are incorrect interpretations of the
statistical data.

43.

C. A patient who cannot care for himself at home does not necessarily have impaired
awareness; he may simply have some degree of immobility.
44. D. Hip fracture, the most common injury among elderly persons, usually results from
osteoporosis. The other answers are diseases that can occur in the elderly from
physiologic changes.
45. A. Sleep disturbances, inability to concentrate and decreased appetite are symptoms
of depression, the most common psychogenic disorder among elderly persons. Other
symptoms include diminished memory, apathy, disinterest in appearance, withdrawal,
and irritability. Depression typically begins before the onset of old age and usually is
caused by psychosocial, genetic, or biochemical factors
46. D. Aging decreases elasticity of the blood vessels, which leads to increased peripheral
resistance and decreased blood flow. These changes, in turn, increase the work load of
the left ventricle.
47. D. Alzheimer;s disease, sometimes known as senile dementia of the Alzheimers type
or primary degenerative dementia, is an insidious; progressive, irreversible, and
degenerative disease of the brain whose etiology is still unknown. Parkinsons disease is
a neurologic disorder caused by lesions in the extrapyramidial system and manifested by
tremors, muscle rigidity, hypokinesis, dysphagia, and dysphonia. Multiple sclerosis, a
progressive, degenerative disease involving demyelination of the nerve fibers, usually
begins in young adulthood and is marked by periods of remission and
exacerbation. Amyotrophic lateral sclerosis, a disease marked by progressive
degeneration of the neurons, eventually results in atrophy of all the muscles; including
those necessary for respiration.
48. C. The nurse is legally responsible for labeling the corpse when death occurs in the
hospital. She may be involved in obtaining consent for an autopsy or notifying the
coroner or medical examiner of a patients death; however, she is not legally responsible
for performing these functions. The attending physician may need information from the
nurse to complete the death certificate, but he is responsible for issuing it.
49. B. The nurse must place a pillow under the decreased persons head and shoulders to
prevent blood from settling in the face and discoloring it. She is required to bathe only
soiled areas of the body since the mortician will wash the entire body. Before wrapping
the body in a shroud, the nurse places a clean gown on the body and closes the eyes and
mouth.
50. A. Ensuring the patients safety is the most essential action at this time. The other
nursing actions may be necessary but are not a major priority.

MATERNAL AND CHILD HEALTH NURSING PRACTICE


QUESTIONS WITH RATIONALE
When assessing the adequacy of sperm for conception to occur, which of the
following is the most useful criterion?
1.
Sperm count
2.
Sperm motility
3.
Sperm maturity
1.

4.
2.

1.
2.
3.
4.
2
1.
2.
3.
4.
2
1.
2.
3.
4.
2
1.
2.
3.
4.
2
1.
2.
3.
4.
2
1.
2.
3.
4.
2
1.
2.
3.
4.
2

Semen volume
A couple who wants to conceive but has been unsuccessful during the last 2
years has undergone many diagnostic procedures. When discussing the situation with
the nurse, one partner states, We know several friends in our age group and all of
them have their own child already, Why cant we have one?. Which of the following
would be the most pertinent nursing diagnosis for this couple?
Fear related to the unknown
Pain related to numerous procedures.
Ineffective family coping related to infertility.
Self-esteem disturbance related to infertility.
Which of the following urinary symptoms does the pregnant woman most frequently
experience during the first trimester?
Dysuria
Frequency
Incontinence
Burning
Heartburn and flatulence, common in the second trimester, are most likely the result
of which of the following?
Increased plasma HCG levels
Decreased intestinal motility
Decreased gastric acidity
Elevated estrogen levels
On which of the following areas would the nurse expect to observe chloasma?
Breast, areola, and nipples
Chest, neck, arms, and legs
Abdomen, breast, and thighs
Cheeks, forehead, and nose
A pregnant client states that she waddles when she walks. The nurses explanation is
based on which of the following as the cause?
The large size of the newborn
Pressure on the pelvic muscles
Relaxation of the pelvic joints
Excessive weight gain
Which of the following represents the average amount of weight gained during
pregnancy?
12 to 22 lb
15 to 25 lb
24 to 30 lb
25 to 40 lb
When talking with a pregnant client who is experiencing aching swollen, leg veins, the
nurse would explain that this is most probably the result of which of the following?
Thrombophlebitis
Pregnancy-induced hypertension
Pressure on blood vessels from the enlarging uterus
The force of gravity pulling down on the uterus
Cervical softening and uterine souffle are classified as which of the following?

1.
Diagnostic signs
2.
Presumptive signs
3.
Probable signs
4.
Positive signs
2 Which of the following would the nurse identify as a presumptive sign of pregnancy?
1.
Hegar sign
2.
Nausea and vomiting
3.
Skin pigmentation changes
4.
Positive serum pregnancy test
2 Which of the following common emotional reactions to pregnancy would the nurse
expect to occur during the first trimester?
1.
Introversion, egocentrism, narcissism
2.
Awkwardness, clumsiness, and unattractiveness
3.
Anxiety, passivity, extroversion
4.
Ambivalence, fear, fantasies
2 During which of the following would the focus of classes be mainly on physiologic
changes, fetal development, sexuality, during pregnancy, and nutrition?
1.
Prepregnant period
2.
First trimester
3.
Second trimester
4.
Third trimester
2 Which of the following would be disadvantage of breast feeding?
1.
Involution occurs more rapidly
2.
The incidence of allergies increases due to maternal antibodies
3.
The father may resent the infants demands on the mothers body
4.
There is a greater chance for error during preparation
2 Which of the following would cause a false-positive result on a pregnancy test?
1.
The test was performed less than 10 days after an abortion
2.
The test was performed too early or too late in the pregnancy
3.
The urine sample was stored too long at room temperature
4.
A spontaneous abortion or a missed abortion is impending
2 FHR can be auscultated with a fetoscope as early as which of the following?
1.
5 weeks gestation
2.
10 weeks gestation
3.
15 weeks gestation
4.
20 weeks gestation
2 A client LMP began July 5. Her EDD should be which of the following?
1.
January 2
2.
March 28
3.
April 12
4.
October 12
2 Which of the following fundal heights indicates less than 12 weeks gestation when the
date of the LMP is unknown?
1.
Uterus in the pelvis
2.
Uterus at the xiphoid
3.
Uterus in the abdomen

4.
Uterus at the umbilicus
2 Which of the following danger signs should be reported promptly during the
antepartum period?
1.
Constipation
2.
Breast tenderness
3.
Nasal stuffiness
4.
Leaking amniotic fluid
2 Which of the following prenatal laboratory test values would the nurse consider as
significant?
1.
Hematocrit 33.5%
2.
Rubella titer less than 1:8
3.
White blood cells 8,000/mm3
4.
One hour glucose challenge test 110 g/dL
2 Which of the following characteristics of contractions would the nurse expect to find
in a client experiencing true labor?
1.
Occurring at irregular intervals
2.
Starting mainly in the abdomen
3.
Gradually increasing intervals
4.
Increasing intensity with walking
2 During which of the following stages of labor would the nurse assess crowning?
1.
First stage
2.
Second stage
3.
Third stage
4.
Fourth stage
2 Barbiturates are usually not given for pain relief during active labor for which of the
following reasons?
1.
The neonatal effects include hypotonia, hypothermia, generalized
drowsiness, and reluctance to feed for the first few days.
2.
These drugs readily cross the placental barrier, causing depressive
effects in the newborn 2 to 3 hours after intramuscular injection.
3.
They rapidly transfer across the placenta, and lack of an antagonist
make them generally inappropriate during labor.
4.
Adverse reactions may include maternal hypotension, allergic or toxic
reaction or partial or total respiratory failure
2 Which of the following nursing interventions would the nurse perform during the
third stage of labor?
1.
Obtain a urine specimen and other laboratory tests.
2.
Assess uterine contractions every 30 minutes.
3.
Coach for effective client pushing
4.
Promote parent-newborn interaction.
2 Which of the following actions demonstrates the nurses understanding about the
newborns thermoregulatory ability?
1.
Placing the newborn under a radiant warmer.
2.
Suctioning with a bulb syringe
3.
Obtaining an Apgar score
4.
Inspecting the newborns umbilical cord

2 Immediately before expulsion, which of the following cardinal movements occur?


1.
Descent
2.
Flexion
3.
Extension
4.
External rotation
2 Before birth, which of the following structures connects the right and left auricles of
the heart?
1.
Umbilical vein
2.
Foramen ovale
3.
Ductus arteriosus
4.
Ductus venosus
2 Which of the following when present in the urine may cause a reddish stain on the
diaper of a newborn?
1.
Mucus
2.
Uric acid crystals
3.
Bilirubin
4.
Excess iron
2 When assessing the newborns heart rate, which of the following ranges would be
considered normal if the newborn were sleeping?
1.
80 beats per minute
2.
100 beats per minute
3.
120 beats per minute
4.
140 beats per minute
2 Which of the following is true regarding the fontanels of the newborn?
1.
The anterior is triangular shaped; the posterior is diamond shaped.
2.
The posterior closes at 18 months; the anterior closes at 8 to 12
weeks.
3.
The anterior is large in size when compared to the posterior fontanel.
4.
The anterior is bulging; the posterior appears sunken.
2 Which of the following groups of newborn reflexes below are present at birth and
remain unchanged through adulthood?
1.
Blink, cough, rooting, and gag
2.
Blink, cough, sneeze, gag
3.
Rooting, sneeze, swallowing, and cough
4.
Stepping, blink, cough, and sneeze
2 Which of the following describes the Babinski reflex?
1.
The newborns toes will hyperextend and fan apart from dorsiflexion
of the big toe when one side of foot is stroked upward from the ball of the heel and
across the ball of the foot.
2.
The newborn abducts and flexes all extremities and may begin to cry
when exposed to sudden movement or loud noise.
3.
The newborn turns the head in the direction of stimulus, opens the
mouth, and begins to suck when cheek, lip, or corner of mouth is touched.
4.
The newborn will attempt to crawl forward with both arms and legs
when he is placed on his abdomen on a flat surface
2 Which of the following statements best describes hyperemesis gravidarum?

1.
2.
3.
4.
2
1.
2.
3.
4.
2
1.
2.
3.
4.
2
1.
2.
3.
4.
2
1.
2.
3.
4.
2
1.
2.
3.
4.
2
1.
2.
3.
4.
2

Severe anemia leading to electrolyte, metabolic, and nutritional


imbalances in the absence of other medical problems.
Severe nausea and vomiting leading to electrolyte, metabolic, and
nutritional imbalances in the absence of other medical problems.
Loss of appetite and continuous vomiting that commonly results in
dehydration and ultimately decreasing maternal nutrients
Severe nausea and diarrhea that can cause gastrointestinal irritation
and possibly internal bleeding
Which of the following would the nurse identify as a classic sign of PIH?
Edema of the feet and ankles
Edema of the hands and face
Weight gain of 1 lb/week
Early morning headache
In which of the following types of spontaneous abortions would the nurse assess dark
brown vaginal discharge and a negative pregnancy tests?
Threatened
Imminent
Missed
Incomplete
Which of the following factors would the nurse suspect as predisposing a client to
placenta previa?
Multiple gestation
Uterine anomalies
Abdominal trauma
Renal or vascular disease
Which of the following would the nurse assess in a client experiencing abruptio
placenta?
Bright red, painless vaginal bleeding
Concealed or external dark red bleeding
Palpable fetal outline
Soft and nontender abdomen
Which of the following is described as premature separation of a normally implanted
placenta during the second half of pregnancy, usually with severe hemorrhage?
Placenta previa
Ectopic pregnancy
Incompetent cervix
Abruptio placentae
Which of the following may happen if the uterus becomes overstimulated by oxytocin
during the induction of labor?
Weak contraction prolonged to more than 70 seconds
Tetanic contractions prolonged to more than 90 seconds
Increased pain with bright red vaginal bleeding
Increased restlessness and anxiety
When preparing a client for cesarean delivery, which of the following key concepts
should be considered when implementing nursing care?

1.
2.
3.
4.
2
1.
2.
3.
4.
2
1.
2.
3.
4.
2
1.
2.
3.
4.
2
1.
2.
3.
4.
2
1.
2.
3.
4.
2
1.
2.
3.
4.
2

Instruct the mothers support person to remain in the family lounge


until after the delivery
Arrange for a staff member of the anesthesia department to explain
what to expect postoperatively
Modify preoperative teaching to meet the needs of either a planned or
emergency cesarean birth
Explain the surgery, expected outcome, and kind of anesthetics
Which of the following best describes preterm labor?
Labor that begins after 20 weeks gestation and before 37 weeks
gestation
Labor that begins after 15 weeks gestation and before 37 weeks
gestation
Labor that begins after 24 weeks gestation and before 28 weeks
gestation
Labor that begins after 28 weeks gestation and before 40 weeks
gestation
When PROM occurs, which of the following provides evidence of the nurses
understanding of the clients immediate needs?
The chorion and amnion rupture 4 hours before the onset of labor.
PROM removes the fetus most effective defense against infection
Nursing care is based on fetal viability and gestational age.
PROM is associated with malpresentation and possibly incompetent
cervix
Which of the following factors is the underlying cause of dystocia?
Nurtional
Mechanical
Environmental
Medical
When uterine rupture occurs, which of the following would be the priority?
Limiting hypovolemic shock
Obtaining blood specimens
Instituting complete bed rest
Inserting a urinary catheter
Which of the following is the nurses initial action when umbilical cord prolapse
occurs?
Begin monitoring maternal vital signs and FHR
Place the client in a knee-chest position in bed
Notify the physician and prepare the client for delivery
Apply a sterile warm saline dressing to the exposed cord
Which of the following amounts of blood loss following birth marks the criterion for
describing postpartum hemorrhage?
More than 200 ml
More than 300 ml
More than 400 ml
More than 500 ml
Which of the following is the primary predisposing factor related to mastitis?

1.
2.
3.
4.
2
1.
2.
3.
4.
2
1.
2.
3.
4.
2
1.
2.
3.
4.
2
1.
2.
3.
4.
2
1.
2.
3.
4.

Epidemic infection from nosocomial sources localizing in the


lactiferous glands and ducts
Endemic infection occurring randomly and localizing in the
periglandular connective tissue
Temporary urinary retention due to decreased perception of the urge
to avoid
Breast injury caused by overdistention, stasis, and cracking of the
nipples
Which of the following best describes thrombophlebitis?
Inflammation and clot formation that result when blood components
combine to form an aggregate body
Inflammation and blood clots that eventually become lodged within
the pulmonary blood vessels
Inflammation and blood clots that eventually become lodged within
the femoral vein
Inflammation of the vascular endothelium with clot formation on the
vessel wall
Which of the following assessment findings would the nurse expect if the client
develops DVT?
Midcalf pain, tenderness and redness along the vein
Chills, fever, malaise, occurring 2 weeks after delivery
Muscle pain the presence of Homans sign, and swelling in the
affected limb
Chills, fever, stiffness, and pain occurring 10 to 14 days after delivery
Which of the following are the most commonly assessed findings in cystitis?
Frequency, urgency, dehydration, nausea, chills, and flank pain
Nocturia, frequency, urgency dysuria, hematuria, fever and
suprapubic pain
Dehydration, hypertension, dysuria, suprapubic pain, chills, and fever
High fever, chills, flank pain nausea, vomiting, dysuria, and frequency
Which of the following best reflects the frequency of reported postpartum blues?
Between 10% and 40% of all new mothers report some form of
postpartum blues
Between 30% and 50% of all new mothers report some form of
postpartum blues
Between 50% and 80% of all new mothers report some form of
postpartum blues
Between 25% and 70% of all new mothers report some form of
postpartum blues
For the client who is using oral contraceptives, the nurse informs the client about the
need to take the pill at the same time each day to accomplish which of the following?
Decrease the incidence of nausea
Maintain hormonal levels
Reduce side effects
Prevent drug interactions

When teaching a client about contraception. Which of the following would the
nurse include as the most effective method for preventing sexually transmitted
infections?
1.
Spermicides
2.
Diaphragm
3.
Condoms
4.
Vasectomy
53.
When preparing a woman who is 2 days postpartum for discharge,
recommendations for which of the following contraceptive methods would be
avoided?
1.
Diaphragm
2.
Female condom
3.
Oral contraceptives
4.
Rhythm method
53 For which of the following clients would the nurse expect that an intrauterine device
would not be recommended?
1.
Woman over age 35
2.
Nulliparous woman
3.
Promiscuous young adult
4.
Postpartum client
53 A client in her third trimester tells the nurse, Im constipated all the time! Which of
the following should the nurse recommend?
1.
Daily enemas
2.
Laxatives
3.
Increased fiber intake
4.
Decreased fluid intake
53 Which of the following would the nurse use as the basis for the teaching plan when
caring for a pregnant teenager concerned about gaining too much weight during
pregnancy?
1.
10 pounds per trimester
2.
1 pound per week for 40 weeks
3.
pound per week for 40 weeks
4.
A total gain of 25 to 30 pounds
53 The client tells the nurse that her last menstrual period started on January 14 and
ended on January 20. Using Nageles rule, the nurse determines her EDD to be which
of the following?
1.
September 27
2.
October 21
3.
November 7
4.
December 27
53 When taking an obstetrical history on a pregnant client who states, I had a son born
at 38 weeks gestation, a daughter born at 30 weeks gestation and I lost a baby at
about 8 weeks, the nurse should record her obstetrical history as which of the
following?
1.
G2 T2 P0 A0 L2
2.
G3 T1 P1 A0 L2
52.

3.
G3 T2 P0 A0 L2
4.
G4 T2 P1 A1 L2
53 When preparing to listen to the fetal heart rate at 12 weeks gestation, the nurse would
use which of the following?
1.
Stethoscope placed midline at the umbilicus
2.
Doppler placed midline at the suprapubic region
3.
Fetoscope placed midway between the umbilicus and the xiphoid
process
4.
External electronic fetal monitor placed at the umbilicus
53 When developing a plan of care for a client newly diagnosed with gestational diabetes,
which of the following instructions would be the priority?
1.
Dietary intake
2.
Medication
3.
Exercise
4.
Glucose monitoring
53 A client at 24 weeks gestation has gained 6 pounds in 4 weeks. Which of the following
would be the priority when assessing the client?
1.
Glucosuria
2.
Depression
3.
Hand/face edema
4.
Dietary intake
53 A client 12 weeks pregnant come to the emergency department with abdominal
cramping and moderate vaginal bleeding. Speculum examination reveals 2 to 3 cms
cervical dilation. The nurse would document these findings as which of the following?
1.
Threatened abortion
2.
Imminent abortion
3.
Complete abortion
4.
Missed abortion
53 Which of the following would be the priority nursing diagnosis for a client with an
ectopic pregnancy?
1.
Risk for infection
2.
Pain
3.
Knowledge Deficit
4.
Anticipatory Grieving
53 Before assessing the postpartum clients uterus for firmness and position in relation
to the umbilicus and midline, which of the following should the nurse do first?
1.
Assess the vital signs
2.
Administer analgesia
3.
Ambulate her in the hall
4.
Assist her to urinate
53 Which of the following should the nurse do when a primipara who is lactating tells the
nurse that she has sore nipples?
1.
Tell her to breast feed more frequently
2.
Administer a narcotic before breast feeding
3.
Encourage her to wear a nursing brassiere
4.
Use soap and water to clean the nipples

53 The nurse assesses the vital signs of a client, 4 hours postpartum that are as follows:
BP 90/60; temperature 100.4F; pulse 100 weak, thready; R 20 per minute. Which of
the following should the nurse do first?
1.
Report the temperature to the physician
2.
Recheck the blood pressure with another cuff
3.
Assess the uterus for firmness and position
4.
Determine the amount of lochia
53 The nurse assesses the postpartum vaginal discharge (lochia) on four clients. Which
of the following assessments would warrant notification of the physician?
1.
A dark red discharge on a 2-day postpartum client
2.
A pink to brownish discharge on a client who is 5 days postpartum
3.
Almost colorless to creamy discharge on a client 2 weeks after
delivery
4.
A bright red discharge 5 days after delivery
53 A postpartum client has a temperature of 101.4F, with a uterus that is tender when
palpated, remains unusually large, and not descending as normally expected. Which
of the following should the nurse assess next?
1.
Lochia
2.
Breasts
3.
Incision
4.
Urine
53 Which of the following is the priority focus of nursing practice with the current early
postpartum discharge?
1.
Promoting comfort and restoration of health
2.
Exploring the emotional status of the family
3.
Facilitating safe and effective self-and newborn care
4.
Teaching about the importance of family planning
53 Which of the following actions would be least effective in maintaining a neutral
thermal environment for the newborn?
1.
Placing infant under radiant warmer after bathing
2.
Covering the scale with a warmed blanket prior to weighing
3.
Placing crib close to nursery window for family viewing
4.
Covering the infants head with a knit stockinette
53 A newborn who has an asymmetrical Moro reflex response should be further assessed
for which of the following?
1.
Talipes equinovarus
2.
Fractured clavicle
3.
Congenital hypothyroidism
4.
Increased intracranial pressure
53 During the first 4 hours after a male circumcision, assessing for which of the following
is the priority?
1.
Infection
2.
Hemorrhage
3.
Discomfort
4.
Dehydration

53 The mother asks the nurse. Whats wrong with my sons breasts? Why are they so
enlarged? Whish of the following would be the best response by the nurse?
1.
The breast tissue is inflamed from the trauma experienced with
birth
2.
A decrease in material hormones present before birth causes
enlargement,
3.
You should discuss this with your doctor. It could be a malignancy
4.
The tissue has hypertrophied while the baby was in the uterus
53 Immediately after birth the nurse notes the following on a male newborn: respirations
78; apical hearth rate 160 BPM, nostril flaring; mild intercostal retractions; and
grunting at the end of expiration. Which of the following should the nurse do?
1.
Call the assessment data to the physicians attention
2.
Start oxygen per nasal cannula at 2 L/min.
3.
Suction the infants mouth and nares
4.
Recognize this as normal first period of reactivity
53 The nurse hears a mother telling a friend on the telephone about umbilical cord care.
Which of the following statements by the mother indicates effective teaching?
1.
Daily soap and water cleansing is best
2.
Alcohol helps it dry and kills germs
3.
An antibiotic ointment applied daily prevents infection
4.
He can have a tub bath each day
53 A newborn weighing 3000 grams and feeding every 4 hours needs 120 calories/kg of
body weight every 24 hours for proper growth and development. How many ounces of
20 cal/oz formula should this newborn receive at each feeding to meet nutritional
needs?
1.
2 ounces
2.
3 ounces
3.
4 ounces
4.
6 ounces
53 The postterm neonate with meconium-stained amniotic fluid needs care designed to
especially monitor for which of the following?
1.
Respiratory problems
2.
Gastrointestinal problems
3.
Integumentary problems
4.
Elimination problems
53 When measuring a clients fundal height, which of the following techniques denotes
the correct method of measurement used by the nurse?
1.
From the xiphoid process to the umbilicus
2.
From the symphysis pubis to the xiphoid process
3.
From the symphysis pubis to the fundus
4.
From the fundus to the umbilicus
53 A client with severe preeclampsia is admitted with of BP 160/110, proteinuria, and
severe pitting edema. Which of the following would be most important to include in
the clients plan of care?
1.
Daily weights
2.
Seizure precautions

3.
Right lateral positioning
4.
Stress reduction
53 A postpartum primipara asks the nurse, When can we have sexual intercourse
again? Which of the following would be the nurses best response?
1.
Anytime you both want to.
2.
As soon as choose a contraceptive method.
3.
When the discharge has stopped and the incision is healed.
4.
After your 6 weeks examination.
53 When preparing to administer the vitamin K injection to a neonate, the nurse would
select which of the following sites as appropriate for the injection?
1.
Deltoid muscle
2.
Anterior femoris muscle
3.
Vastus lateralis muscle
4.
Gluteus maximus muscle
53 When performing a pelvic examination, the nurse observes a red swollen area on the
right side of the vaginal orifice. The nurse would document this as enlargement of
which of the following?
1.
Clitoris
2.
Parotid gland
3.
Skenes gland
4.
Bartholins gland
53 To differentiate as a female, the hormonal stimulation of the embryo that must occur
involves which of the following?
1.
Increase in maternal estrogen secretion
2.
Decrease in maternal androgen secretion
3.
Secretion of androgen by the fetal gonad
4.
Secretion of estrogen by the fetal gonad
53 A client at 8 weeks gestation calls complaining of slight nausea in the morning hours.
Which of the following client interventions should the nurse question?
1.
Taking 1 teaspoon of bicarbonate of soda in an 8-ounce glass of water
2.
Eating a few low-sodium crackers before getting out of bed
3.
Avoiding the intake of liquids in the morning hours
4.
Eating six small meals a day instead of thee large meals
53 The nurse documents positive ballottement in the clients prenatal record. The nurse
understands that this indicates which of the following?
1.
Palpable contractions on the abdomen
2.
Passive movement of the unengaged fetus
3.
Fetal kicking felt by the client
4.
Enlargement and softening of the uterus
53 During a pelvic exam the nurse notes a purple-blue tinge of the cervix. The nurse
documents this as which of the following?
1.
Braxton-Hicks sign
2.
Chadwicks sign
3.
Goodells sign
4.
McDonalds sign

53 During a prenatal class, the nurse explains the rationale for breathing techniques
during preparation for labor based on the understanding that breathing techniques
are most important in achieving which of the following?
1.
Eliminate pain and give the expectant parents something to do
2.
Reduce the risk of fetal distress by increasing uteroplacental
perfusion
3.
Facilitate relaxation, possibly reducing the perception of pain
4.
Eliminate pain so that less analgesia and anesthesia are needed
53 After 4 hours of active labor, the nurse notes that the contractions of a primigravida
client are not strong enough to dilate the cervix. Which of the following would the
nurse anticipate doing?
1.
Obtaining an order to begin IV oxytocin infusion
2.
Administering a light sedative to allow the patient to rest for several
hour
3.
Preparing for a cesarean section for failure to progress
4.
Increasing the encouragement to the patient when pushing begins
53 A multigravida at 38 weeks gestation is admitted with painless, bright red bleeding
and mild contractions every 7 to 10 minutes. Which of the following assessments
should be avoided?
1.
Maternal vital sign
2.
Fetal heart rate
3.
Contraction monitoring
4.
Cervical dilation
53 Which of the following would be the nurses most appropriate response to a client
who asks why she must have a cesarean delivery if she has a complete placenta
previa?
1.
You will have to ask your physician when he returns.
2.
You need a cesarean to prevent hemorrhage.
3.
The placenta is covering most of your cervix.
4.
The placenta is covering the opening of the uterus and blocking your
baby.
53 The nurse understands that the fetal head is in which of the following positions with a
face presentation?
1.
Completely flexed
2.
Completely extended
3.
Partially extended
4.
Partially flexed
53 With a fetus in the left-anterior breech presentation, the nurse would expect the fetal
heart rate would be most audible in which of the following areas?
1.
Above the maternal umbilicus and to the right of midline
2.
In the lower-left maternal abdominal quadrant
3.
In the lower-right maternal abdominal quadrant
4.
Above the maternal umbilicus and to the left of midline
53 The amniotic fluid of a client has a greenish tint. The nurse interprets this to be the
result of which of the following?
1.
Lanugo

2.
Hydramnio
3.
Meconium
4.
Vernix
53 A patient is in labor and has just been told she has a breech presentation. The nurse
should be particularly alert for which of the following?
1.
Quickening
2.
Ophthalmia neonatorum
3.
Pica
4.
Prolapsed umbilical cord
53 When describing dizygotic twins to a couple, on which of the following would the
nurse base the explanation?
1.
Two ova fertilized by separate sperm
2.
Sharing of a common placenta
3.
Each ova with the same genotype
4.
Sharing of a common chorion
53 Which of the following refers to the single cell that reproduces itself after conception?
1.
Chromosome
2.
Blastocyst
3.
Zygote
4.
Trophoblast
53 In the late 1950s, consumers and health care professionals began challenging the
routine use of analgesics and anesthetics during childbirth. Which of the following
was an outgrowth of this concept?
1.
Labor, delivery, recovery, postpartum (LDRP)
2.
Nurse-midwifery
3.
Clinical nurse specialist
4.
Prepared childbirth
53 A client has a midpelvic contracture from a previous pelvic injury due to a motor
vehicle accident as a teenager. The nurse is aware that this could prevent a fetus from
passing through or around which structure during childbirth?
1.
Symphysis pubis
2.
Sacral promontory
3.
Ischial spines
4.
Pubic arch
53 When teaching a group of adolescents about variations in the length of the menstrual
cycle, the nurse understands that the underlying mechanism is due to variations in
which of the following phases?
1.
Menstrual phase
2.
Proliferative phase
3.
Secretory phase
4.
Ischemic phase
53 When teaching a group of adolescents about male hormone production, which of the
following would the nurse include as being produced by the Leydig cells?
1.
Follicle-stimulating hormone
2.
Testosterone
3.
Leuteinizing hormone

Gonadotropin releasing hormone


101.
While performing physical assessment of a 12 month-old, the nurse notes that
the infants anterior fontanelle is still slightly open. Which of the following is the
nurses most appropriate action?
1.
Notify the physician immediately because there is a problem.
2.
Perform an intensive neurologic examination.
3.
Perform an intensive developmental examination.
4.
Do nothing because this is a normal finding for the age.
102.
When teaching a mother about introducing solid foods to her child, which of
the following indicates the earliest age at which this should be done?
1.
1 month
2.
2 months
3.
3 months
4.
4 months
102
The infant of a substance-abusing mother is at risk for developing a sense of
which of the following?
1.
Mistrust
2.
Shame
3.
Guilt
4.
Inferiority
102
Which of the following toys should the nurse recommend for a 5-month-old?
1.
A big red balloon
2.
A teddy bear with button eyes
3.
A push-pull wooden truck
4.
A colorful busy box
102
The mother of a 2-month-old is concerned that she may be spoiling her baby
by picking her up when she cries. Which of the following would be the nurses best
response?
1.
Let her cry for a while before picking her up, so you dont spoil her
2.
Babies need to be held and cuddled; you wont spoil her this way
3.
Crying at this age means the baby is hungry; give her a bottle
4.
If you leave her alone she will learn how to cry herself to sleep
102
When assessing an 18-month-old, the nurse notes a characteristic protruding
abdomen. Which of the following would explain the rationale for this finding?
1.
Increased food intake owing to age
2.
Underdeveloped abdominal muscles
3.
Bowlegged posture
4.
Linear growth curve
102
If parents keep a toddler dependent in areas where he is capable of using skills,
the toddle will develop a sense of which of the following?
1.
Mistrust
2.
Shame
3.
Guilt
4.
Inferiority
102
Which of the following is an appropriate toy for an 18-month-old?
1.
Multiple-piece puzzle

Miniature cars
Finger paints
Comic book
When teaching parents about the childs readiness for toilet training, which of
the following signs should the nurse instruct them to watch for in the toddler?
1.
Demonstrates dryness for 4 hours
2.
Demonstrates ability to sit and walk
3.
Has a new sibling for stimulation
4.
Verbalizes desire to go to the bathroom
102
When teaching parents about typical toddler eating patterns, which of the
following should be included?
1.
Food jags
2.
Preference to eat alone
3.
Consistent table manners
4.
Increase in appetite
102
Which of the following suggestions should the nurse offer the parents of a 4year-old boy who resists going to bed at night?
1.
Allow him to fall asleep in your room, then move him to his own
bed.
2.
Tell him that you will lock him in his room if he gets out of bed one
more time.
3.
Encourage active play at bedtime to tire him out so he will fall asleep
faster.
4.
Read him a story and allow him to play quietly in his bed until he
falls asleep.
102
When providing therapeutic play, which of the following toys would best
promote imaginative play in a 4-year-old?
1.
Large blocks
2.
Dress-up clothes
3.
Wooden puzzle
4.
Big wheels
102
Which of the following activities, when voiced by the parents following a
teaching session about the characteristics of school-age cognitive development would
indicate the need for additional teaching?
1.
Collecting baseball cards and marbles
2.
Ordering dolls according to size
3.
Considering simple problem-solving options
4.
Developing plans for the future
102
A hospitalized schoolager states: Im not afraid of this place, Im not afraid of
anything. This statement is most likely an example of which of the following?
1.
Regression
2.
Repression
3.
Reaction formation
4.
Rationalization
2.
3.
4.
102

After teaching a group of parents about accident prevention for schoolagers,


which of the following statements by the group would indicate the need for more
teaching?
1.
Schoolagers are more active and adventurous than are younger
children.
2.
Schoolagers are more susceptible to home hazards than are younger
children.
3.
Schoolagers are unable to understand potential dangers around
them.
4.
Schoolargers are less subject to parental control than are younger
children.
102
Which of the following skills is the most significant one learned during the
schoolage period?
1.
Collecting
2.
Ordering
3.
Reading
4.
Sorting
102
A child age 7 was unable to receive the measles, mumps, and rubella (MMR)
vaccine at the recommended scheduled time. When would the nurse expect to
administer MMR vaccine?
1.
In a month from now
2.
In a year from now
3.
At age 10
4.
At age 13
102
The adolescents inability to develop a sense of who he is and what he can
become results in a sense of which of the following?
1.
Shame
2.
Guilt
3.
Inferiority
4.
Role diffusion
102
Which of the following would be most appropriate for a nurse to use when
describing menarche to a 13-year-old?
1.
A females first menstruation or menstrual periods
2.
The first year of menstruation or period
3.
The entire menstrual cycle or from one period to another
4.
The onset of uterine maturation or peak growth
102
A 14-year-old boy has acne and according to his parents, dominates the
bathroom by using the mirror all the time. Which of the following remarks by the
nurse would be least helpful in talking to the boy and his parents?
1.
This is probably the only concern he has about his body. So dont
worry about it or the time he spends on it.
2.
Teenagers are anxious about how their peers perceive them. So they
spend a lot of time grooming.
3.
A teen may develop a poor self-image when experiencing acne. Do
you feel this way sometimes?
4.
You appear to be keeping your face well washed. Would you feel
comfortable discussing your cleansing method?
102

Which of the following should the nurse suspect when noting that a 3-year-old
is engaging in explicit sexual behavior during doll play?
1.
The child is exhibiting normal pre-school curiosity
2.
The child is acting out personal experiences
3.
The child does not know how to play with dolls
4.
The child is probably developmentally delayed.
102
Which of the following statements by the parents of a child with school phobia
would indicate the need for further teaching?
1.
Well keep him at home until phobia subsides.
2.
Well work with his teachers and counselors at school.
3.
Well try to encourage him to talk about his problem.
4.
Well discuss possible solutions with him and his counselor.
102
When developing a teaching plan for a group of high school students about
teenage pregnancy, the nurse would keep in mind which of the following?
1.
The incidence of teenage pregnancies is increasing.
2.
Most teenage pregnancies are planned.
3.
Denial of the pregnancy is common early on.
4.
The risk for complications during pregnancy is rare.
102
When assessing a child with a cleft palate, the nurse is aware that the child is at
risk for more frequent episodes of otitis media due to which of the following?
1.
Lowered resistance from malnutrition
2.
Ineffective functioning of the Eustachian tubes
3.
Plugging of the Eustachian tubes with food particles
4.
Associated congenital defects of the middle ear.
102
While performing a neurodevelopmental assessment on a 3-month-old infant,
which of the following characteristics would be expected?
1.
A strong Moro reflex
2.
A strong parachute reflex
3.
Rolling from front to back
4.
Lifting of head and chest when prone
102
By the end of which of the following would the nurse most commonly expect a
childs birth weight to triple?
1.
4 months
2.
7 months
3.
9 months
4.
12 months
102
Which of the following best describes parallel play between two toddlers?
1.
Sharing crayons to color separate pictures
2.
Playing a board game with a nurse
3.
Sitting near each other while playing with separate dolls
4.
Sharing their dolls with two different nurses
102
Which of the following would the nurse identify as the initial priority for a
child with acute lymphocytic leukemia?
1.
Instituting infection control precautions
2.
Encouraging adequate intake of iron-rich foods
3.
Assisting with coping with chronic illness
102

Administering medications via IM injections


Which of the following information, when voiced by the mother, would
indicate to the nurse that she understands home care instructions following the
administration of a diphtheria, tetanus, and pertussis injection?
1.
Measures to reduce fever
2.
Need for dietary restrictions
3.
Reasons for subsequent rash
4.
Measures to control subsequent diarrhea
102
Which of the following actions by a community health nurse is most
appropriate when noting multiple bruises and burns on the posterior trunk of an 18month-old child during a home visit?
1.
Report the childs condition to Protective Services immediately.
2.
Schedule a follow-up visit to check for more bruises.
3.
Notify the childs physician immediately.
4.
Don nothing because this is a normal finding in a toddler.
102
Which of the following is being used when the mother of a hospitalized child
calls the student nurse and states, You idiot, you have no idea how to care for my sick
child?
1.
Displacement
2.
Projection
3.
Repression
4.
Psychosis
102
Which of the following should the nurse expect to note as a frequent
complication for a child with congenital heart disease?
1.
Susceptibility to respiratory infection
2.
Bleeding tendencies
3.
Frequent vomiting and diarrhea
4.
Seizure disorder
102
Which of the following would the nurse do first for a 3-year-old boy who
arrives in the emergency room with a temperature of 105 degrees, inspiratory stridor,
and restlessness, who is learning forward and drooling?
1.
Auscultate his lungs and place him in a mist tent.
2.
Have him lie down and rest after encouraging fluids.
3.
Examine his throat and perform a throat culture
4.
Notify the physician immediately and prepare for intubation.
102
Which of the following would the nurse need to keep in mind as a predisposing
factor when formulating a teaching plan for child with a urinary tract infection?
1.
A shorter urethra in females
2.
Frequent emptying of the bladder
3.
Increased fluid intake
4.
Ingestion of acidic juices
102
Which of the following should the nurse do first for a 15-year-old boy with a
full leg cast who is screaming in unrelenting pain and exhibiting right foot pallor
signifying compartment syndrome?
1.
Medicate him with acetaminophen.
2.
Notify the physician immediately
4.
102

Release the traction


Monitor him every 5 minutes
At which of the following ages would the nurse expect to administer the
varicella zoster vaccine to child?
1.
At birth
2.
2 months
3.
6 months
4.
12 months
102
When discussing normal infant growth and development with parents, which
of the following toys would the nurse suggest as most appropriate for an 8-monthold?
1.
Push-pull toys
2.
Rattle
3.
Large blocks
4.
Mobile
102
Which of the following aspects of psychosocial development is necessary for
the nurse to keep in mind when providing care for the preschool child?
1.
The child can use complex reasoning to think out situations.
2.
Fear of body mutilation is a common preschool fear
3.
The child engages in competitive types of play
4.
Immediate gratification is necessary to develop initiative.
102
Which of the following is characteristic of a preschooler with mid mental
retardation?
1.
Slow to feed self
2.
Lack of speech
3.
Marked motor delays
4.
Gait disability
102
Which of the following assessment findings would lead the nurse to suspect
Down syndrome in an infant?
1.
Small tongue
2.
Transverse palmar crease
3.
Large nose
4.
Restricted joint movement
102
While assessing a newborn with cleft lip, the nurse would be alert that which of
the following will most likely be compromised?
1.
Sucking ability
2.
Respiratory status
3.
Locomotion
4.
GI function
102
When providing postoperative care for the child with a cleft palate, the nurse
should position the child in which of the following positions?
1.
Supine
2.
Prone
3.
In an infant seat
4.
On the side
3.
4.
102

While assessing a child with pyloric stenosis, the nurse is likely to note which
of the following?
1.
Regurgitation
2.
Steatorrhea
3.
Projectile vomiting
4.
Currant jelly stools
102
Which of the following nursing diagnoses would be inappropriate for the infant
with gastroesophageal reflux (GER)?
1.
Fluid volume deficit
2.
Risk for aspiration
3.
Altered nutrition: less than body requirements
4.
Altered oral mucous membranes
102
Which of the following parameters would the nurse monitor to evaluate the
effectiveness of thickened feedings for an infant with gastroesophageal reflux (GER)?
1.
Vomiting
2.
Stools
3.
Uterine
4.
Weight
102
Discharge teaching for a child with celiac disease would include instructions
about avoiding which of the following?
1.
Rice
2.
Milk
3.
Wheat
4.
Chicken
102
Which of the following would the nurse expect to assess in a child with celiac
disease having a celiac crisis secondary to an upper respiratory infection?
1.
Respiratory distress
2.
Lethargy
3.
Watery diarrhea
4.
Weight gain
102
Which of the following should the nurse do first after noting that a child with
Hirschsprung disease has a fever and watery explosive diarrhea?
1.
Notify the physician immediately
2.
Administer antidiarrheal medications
3.
Monitor child ever 30 minutes
4.
Nothing, this is characteristic of Hirschsprung disease
102
A newborns failure to pass meconium within the first 24 hours after birth may
indicate which of the following?
1.
Hirschsprung disease
2.
Celiac disease
3.
Intussusception
4.
Abdominal wall defect
102
When assessing a child for possible intussusception, which of the following
would be least likely to provide valuable information?
1.
Stool inspection
2.
Pain pattern
102

3.
4.

Family history
Abdominal palpation

ANSWER AND RATIONALE


1.
B. Although all of the factors listed are important, sperm motility is the most
significant criterion when assessing male infertility. Sperm count, sperm maturity,
and semen volume are all significant, but they are not as significant sperm motility.
2.
D. Based on the partners statement, the couple is verbalizing feelings of
inadequacy and negative feelings about themselves and their capabilities. Thus, the
nursing diagnosis of self-esteem disturbance is most appropriate. Fear, pain, and
ineffective family coping also may be present but as secondary nursing diagnoses.
3.
B. Pressure and irritation of the bladder by the growing uterus during the first
trimester is responsible for causing urinary frequency. Dysuria, incontinence, and
burning are symptoms associated with urinary tract infections.
4.
C. During the second trimester, the reduction in gastric acidity in conjunction
with pressure from the growing uterus and smooth muscle relaxation, can cause
heartburn and flatulence. HCG levels increase in the first, not the second, trimester.
Decrease intestinal motility would most likely be the cause of constipation and
bloating. Estrogen levels decrease in the second trimester.
5.
D. Chloasma, also called the mask of pregnancy, is an irregular
hyperpigmented area found on the face. It is not seen on the breasts, areola, nipples,
chest, neck, arms, legs, abdomen, or thighs.
6.
C. During pregnancy, hormonal changes cause relaxation of the pelvic joints,
resulting in the typical waddling gait. Changes in posture are related to the growing
fetus. Pressure on the surrounding muscles causing discomfort is due to the growing
uterus. Weight gain has no effect on gait.
7.
C. The average amount of weight gained during pregnancy is 24 to 30 lb. This
weight gain consists of the following: fetus 7.5 lb; placenta and membrane 1.5 lb;
amniotic fluid 2 lb; uterus 2.5 lb; breasts 3 lb; and increased blood volume 2
to 4 lb; extravascular fluid and fat 4 to 9 lb. A gain of 12 to 22 lb is insufficient,
whereas a weight gain of 15 to 25 lb is marginal. A weight gain of 25 to 40 lb is
considered excessive.
8.
C. Pressure of the growing uterus on blood vessels results in an increased risk
for venous stasis in the lower extremities. Subsequently, edema and varicose vein
formation may occur. Thrombophlebitis is an inflammation of the veins due to
thrombus formation. Pregnancy-induced hypertension is not associated with these
symptoms. Gravity plays only a minor role with these symptoms.
9.
C. Cervical softening (Goodell sign) and uterine souffl are two probable signs
of pregnancy. Probable signs are objective findings that strongly suggest pregnancy.
Other probable signs include Hegar sign, which is softening of the lower uterine
segment; Piskacek sign, which is enlargement and softening of the uterus; serum
laboratory tests; changes in skin pigmentation; and ultrasonic evidence of a
gestational sac. Presumptive signs are subjective signs and include amenorrhea;
nausea and vomiting; urinary frequency; breast tenderness and changes; excessive
fatigue; uterine enlargement; and quickening.
10.
B. Presumptive signs of pregnancy are subjective signs. Of the signs listed,
only nausea and vomiting are presumptive signs. Hegar sign, skin pigmentation
changes, and a positive serum pregnancy test are considered probably signs, which
are strongly suggestive of pregnancy.

D. During the first trimester, common emotional reactions include


ambivalence, fear, fantasies, or anxiety. The second trimester is a period of well-being
accompanied by the increased need to learn about fetal growth and development.
Common emotional reactions during this trimester include narcissism, passivity, or
introversion. At times the woman may seem egocentric and self-centered. During the
third trimester, the woman typically feels awkward, clumsy, and unattractive, often
becoming more introverted or reflective of her own childhood.
12.
B. First-trimester classes commonly focus on such issues as early physiologic
changes, fetal development, sexuality during pregnancy, and nutrition. Some early
classes may include pregnant couples. Second and third trimester classes may focus
on preparation for birth, parenting, and newborn care.
13.
C. With breast feeding, the fathers body is not capable of providing the milk
for the newborn, which may interfere with feeding the newborn, providing fewer
chances for bonding, or he may be jealous of the infants demands on his wifes time
and body. Breast feeding is advantageous because uterine involution occurs more
rapidly, thus minimizing blood loss. The presence of maternal antibodies in breast
milk helps decrease the incidence of allergies in the newborn. A greater chance for
error is associated with bottle feeding. No preparation is required for breast feeding.
14.
A. A false-positive reaction can occur if the pregnancy test is performed less
than 10 days after an abortion. Performing the tests too early or too late in the
pregnancy, storing the urine sample too long at room temperature, or having a
spontaneous or missed abortion impending can all produce false-negative results.
15.
D. The FHR can be auscultated with a fetoscope at about 20 weeks gestation.
FHR usually is ausculatated at the midline suprapubic region with Doppler
ultrasound transducer at 10 to 12 weeks gestation. FHR, cannot be heard any earlier
than 10 weeks gestation.
16.
C. To determine the EDD when the date of the clients LMP is known use
Nagele rule. To the first day of the LMP, add 7 days, subtract 3 months, and add 1 year
(if applicable) to arrive at the EDD as follows: 5 + 7 = 12 (July) minus 3 = 4 (April).
Therefore, the clients EDD is April 12.
17.
A. When the LMP is unknown, the gestational age of the fetus is estimated by
uterine size or position (fundal height). The presence of the uterus in the pelvis
indicates less than 12 weeks gestation. At approximately 12 to 14 weeks, the fundus is
out of the pelvis above the symphysis pubis. The fundus is at the level of the umbilicus
at approximately 20 weeks gestation and reaches the xiphoid at term or 40 weeks.
18.
D. Danger signs that require prompt reporting leaking of amniotic fluid,
vaginal bleeding, blurred vision, rapid weight gain, and elevated blood pressure.
Constipation, breast tenderness, and nasal stuffiness are common discomforts
associated with pregnancy.
19.
B. A rubella titer should be 1:8 or greater. Thurs, a finding of a titer less than
1:8 is significant, indicating that the client may not possess immunity to rubella. A
hematocrit of 33.5% a white blood cell count of 8,000/mm3, and a 1 hour glucose
challenge test of 110 g/dl are with normal parameters.
20.
D. With true labor, contractions increase in intensity with walking. In addition,
true labor contractions occur at regular intervals, usually starting in the back and
sweeping around to the abdomen. The interval of true labor contractions gradually
shortens.
11.

B. Crowing, which occurs when the newborns head or presenting part appears
at the vaginal opening, occurs during the second stage of labor. During the first stage
of labor, cervical dilation and effacement occur. During the third stage of labor, the
newborn and placenta are delivered. The fourth stage of labor lasts from 1 to 4 hours
after birth, during which time the mother and newborn recover from the physical
process of birth and the mothers organs undergo the initial readjustment to the
nonpregnant state.
22.
C. Barbiturates are rapidly transferred across the placental barrier, and lack of
an antagonist makes them generally inappropriate during active labor. Neonatal side
effects of barbiturates include central nervous system depression, prolonged
drowsiness, delayed establishment of feeding (e.g. due to poor sucking reflex or poor
sucking pressure). Tranquilizers are associated with neonatal effects such as
hypotonia, hypothermia, generalized drowsiness, and reluctance to feed for the first
few days. Narcotic analgesic readily cross the placental barrier, causing depressive
effects in the newborn 2 to 3 hours after intramuscular injection. Regional anesthesia
is associated with adverse reactions such as maternal hypotension, allergic or toxic
reaction, or partial or total respiratory failure.
23.
D. During the third stage of labor, which begins with the delivery of the
newborn, the nurse would promote parent-newborn interaction by placing the
newborn on the mothers abdomen and encouraging the parents to touch the
newborn. Collecting a urine specimen and other laboratory tests is done on admission
during the first stage of labor. Assessing uterine contractions every 30 minutes is
performed during the latent phase of the first stage of labor. Coaching the client to
push effectively is appropriate during the second stage of labor.
24.
A. The newborns ability to regulate body temperature is poor. Therefore,
placing the newborn under a radiant warmer aids in maintaining his or her body
temperature. Suctioning with a bulb syringe helps maintain a patent airway.
Obtaining an Apgar score measures the newborns immediate adjustment to
extrauterine life. Inspecting the umbilical cord aids in detecting cord anomalies.
25.
D. Immediately before expulsion or birth of the rest of the body, the cardinal
movement of external rotation occurs. Descent flexion, internal rotation, extension,
and restitution (in this order) occur before external rotation.
26.
B. The foramen ovale is an opening between the right and left auricles (atria)
that should close shortly after birth so the newborn will not have a murmur or mixed
blood traveling through the vascular system. The umbilical vein, ductus arteriosus,
and ductus venosus are obliterated at birth.
27.
B. Uric acid crystals in the urine may produce the reddish brick dust stain on
the diaper. Mucus would not produce a stain. Bilirubin and iron are from hepatic
adaptation.
28.
B. The normal heart rate for a newborn that is sleeping is approximately 100
beats per minute. If the newborn was awake, the normal heart rate would range from
120 to 160 beats per minute.
29.
C. The anterior fontanel is larger in size than the posterior fontanel.
Additionally, the anterior fontanel, which is diamond shaped, closes at 18 months,
whereas the posterior fontanel, which is triangular shaped, closes at 8 to 12 weeks.
Neither fontanel should appear bulging, which may indicate increased intracranial
pressure, or sunken, which may indicate dehydration.
21.

B. Blink, cough, sneeze, swallowing and gag reflexes are all present at birth and
remain unchanged through adulthood. Reflexes such as rooting and stepping subside
within the first year.
31.
A. With the babinski reflex, the newborns toes hyperextend and fan apart
from dorsiflexion of the big toe when one side of foot is stroked upward form the heel
and across the ball of the foot. With the startle reflex, the newborn abducts and flexes
all extremities and may begin to cry when exposed to sudden movement of loud noise.
With the rooting and sucking reflex, the newborn turns his head in the direction of
stimulus, opens the mouth, and begins to suck when the cheeks, lip, or corner of
mouth is touched. With the crawl reflex, the newborn will attempt to crawl forward
with both arms and legs when he is placed on his abdomen on a flat surface.
32.
B. The description of hyperemesis gravidarum includes severe nausea and
vomiting, leading to electrolyte, metabolic, and nutritional imbalances in the absence
of other medical problems. Hyperemesis is not a form of anemia. Loss of appetite may
occur secondary to the nausea and vomiting of hyperemesis, which, if it continues,
can deplete the nutrients transported to the fetus. Diarrhea does not occur with
hyperemesis.
33.
B. Edema of the hands and face is a classic sign of PIH. Many healthy pregnant
woman experience foot and ankle edema. A weight gain of 2 lb or more per week
indicates a problem. Early morning headache is not a classic sign of PIH.
34.
C. In a missed abortion, there is early fetal intrauterine death, and products of
conception are not expelled. The cervix remains closed; there may be a dark brown
vaginal discharge, negative pregnancy test, and cessation of uterine growth and breast
tenderness. A threatened abortion is evidenced with cramping and vaginal bleeding in
early pregnancy, with no cervical dilation. An incomplete abortion presents with
bleeding, cramping, and cervical dilation. An incomplete abortion involves only
expulsion of part of the products of conception and bleeding occurs with cervical
dilation.
35.
A. Multiple gestation is one of the predisposing factors that may cause
placenta previa. Uterine anomalies abdominal trauma, and renal or vascular disease
may predispose a client to abruptio placentae.
36.
B. A client with abruptio placentae may exhibit concealed or dark red bleeding,
possibly reporting sudden intense localized uterine pain. The uterus is typically firm
to boardlike, and the fetal presenting part may be engaged. Bright red, painless
vaginal bleeding, a palpable fetal outline and a soft nontender abdomen are
manifestations of placenta previa.
37.
D. Abruptio placentae is described as premature separation of a normally
implanted placenta during the second half of pregnancy, usually with severe
hemorrhage. Placenta previa refers to implantation of the placenta in the lower
uterine segment, causing painless bleeding in the third trimester of pregnancy.
Ectopic pregnancy refers to the implantation of the products of conception in a site
other than the endometrium. Incompetent cervix is a conduction characterized by
painful dilation of the cervical os without uterine contractions.
38.
B. Hyperstimulation of the uterus such as with oxytocin during the induction
of labor may result in tetanic contractions prolonged to more than 90seconds, which
could lead to such complications as fetal distress, abruptio placentae, amniotic fluid
embolism, laceration of the cervix, and uterine rupture. Weak contractions would not
30.

occur. Pain, bright red vaginal bleeding, and increased restlessness and anxiety are
not associated with hyperstimulation.
39.
C. A key point to consider when preparing the client for a cesarean delivery is
to modify the preoperative teaching to meet the needs of either a planned or
emergency cesarean birth, the depth and breadth of instruction will depend on
circumstances and time available. Allowing the mothers support person to remain
with her as much as possible is an important concept, although doing so depends on
many variables. Arranging for necessary explanations by various staff members to be
involved with the clients care is a nursing responsibility. The nurse is responsible for
reinforcing the explanations about the surgery, expected outcome, and type of
anesthetic to be used. The obstetrician is responsible for explaining about the surgery
and outcome and the anesthesiology staff is responsible for explanations about the
type of anesthesia to be used.
40.
A. Preterm labor is best described as labor that begins after 20 weeks
gestation and before 37 weeks gestation. The other time periods are inaccurate.
41.
B. PROM can precipitate many potential and actual problems; one of the most
serious is the fetus loss of an effective defense against infection. This is the clients
most immediate need at this time. Typically, PROM occurs about 1 hour, not 4 hours,
before labor begins. Fetal viability and gestational age are less immediate
considerations that affect the plan of care. Malpresentation and an incompetent
cervix may be causes of PROM.
42.
B. Dystocia is difficult, painful, prolonged labor due to mechanical factors
involving the fetus (passenger), uterus (powers), pelvis (passage), or psyche.
Nutritional, environment, and medical factors may contribute to the mechanical
factors that cause dystocia.
43.
A. With uterine rupture, the client is at risk for hypovolemic shock. Therefore,
the priority is to prevent and limit hypovolemic shock. Immediate steps should
include giving oxygen, replacing lost fluids, providing drug therapy as needed,
evaluating fetal responses and preparing for surgery. Obtaining blood specimens,
instituting complete bed rest, and inserting a urinary catheter are necessary in
preparation for surgery to remedy the rupture.
44.
B. The immediate priority is to minimize pressure on the cord. Thus the
nurses initial action involves placing the client on bed rest and then placing the client
in a knee-chest position or lowering the head of the bed, and elevating the maternal
hips on a pillow to minimize the pressure on the cord. Monitoring maternal vital signs
and FHR, notifying the physician and preparing the client for delivery, and wrapping
the cord with sterile saline soaked warm gauze are important. But these actions have
no effect on minimizing the pressure on the cord.
45.
D. Postpartum hemorrhage is defined as blood loss of more than 500 ml
following birth. Any amount less than this not considered postpartum hemorrhage.
46.
D. With mastitis, injury to the breast, such as overdistention, stasis, and
cracking of the nipples, is the primary predisposing factor. Epidemic and endemic
infections are probable sources of infection for mastitis. Temporary urinary retention
due to decreased perception of the urge to void is a contributory factor to the
development of urinary tract infection, not mastitis.
47.
D. Thrombophlebitis refers to an inflammation of the vascular endothelium
with clot formation on the wall of the vessel. Blood components combining to form an

aggregate body describe a thrombus or thrombosis. Clots lodging in the pulmonary


vasculature refers to pulmonary embolism; in the femoral vein, femoral
thrombophlebitis.
48.
C. Classic symptoms of DVT include muscle pain, the presence of Homans
sign, and swelling of the affected limb. Midcalf pain, tenderness, and redness, along
the vein reflect superficial thrombophlebitis. Chills, fever and malaise occurring 2
weeks after delivery reflect pelvic thrombophlebitis. Chills, fever, stiffness and pain
occurring 10 to 14 days after delivery suggest femoral thrombophlebitis.
49.
B. Manifestations of cystitis include, frequency, urgency, dysuria, hematuria
nocturia, fever, and suprapubic pain. Dehydration, hypertension, and chills are not
typically associated with cystitis. High fever chills, flank pain, nausea, vomiting,
dysuria, and frequency are associated with pvelonephritis.
50.
C. According to statistical reports, between 50% and 80% of all new mothers
report some form of postpartum blues. The ranges of 10% to 40%, 30% to 50%, and
25% to 70% are incorrect.
51.
B. Regular timely ingestion of oral contraceptives is necessary to maintain
hormonal levels of the drugs to suppress the action of the hypothalamus and anterior
pituitary leading to inappropriate secretion of FSH and LH. Therefore, follicles do not
mature, ovulation is inhibited, and pregnancy is prevented. The estrogen content of
the oral site contraceptive may cause the nausea, regardless of when the pill is taken.
Side effects and drug interactions may occur with oral contraceptives regardless of the
time the pill is taken.
52.
C. Condoms, when used correctly and consistently, are the most effective
contraceptive method or barrier against bacterial and viral sexually transmitted
infections. Although spermicides kill sperm, they do not provide reliable protection
against the spread of sexually transmitted infections, especially intracellular
organisms such as HIV. Insertion and removal of the diaphragm along with the use of
the spermicides may cause vaginal irritations, which could place the client at risk for
infection transmission. Male sterilization eliminates spermatozoa from the ejaculate,
but it does not eliminate bacterial and/or viral microorganisms that can cause
sexually transmitted infections.
53.
A. The diaphragm must be fitted individually to ensure effectiveness. Because
of the changes to the reproductive structures during pregnancy and following
delivery, the diaphragm must be refitted, usually at the 6 weeks examination
following childbirth or after a weight loss of 15 lbs or more. In addition, for maximum
effectiveness, spermicidal jelly should be placed in the dome and around the rim.
However, spermicidal jelly should not be inserted into the vagina until involution is
completed at approximately 6 weeks. Use of a female condom protects the
reproductive system from the introduction of semen or spermicides into the vagina
and may be used after childbirth. Oral contraceptives may be started within the first
postpartum week to ensure suppression of ovulation. For the couple who has
determined the females fertile period, using the rhythm method, avoidance of
intercourse during this period, is safe and effective.
54.
C. An IUD may increase the risk of pelvic inflammatory disease, especially in
women with more than one sexual partner, because of the increased risk of sexually
transmitted infections. An UID should not be used if the woman has an active or
chronic pelvic infection, postpartum infection, endometrial hyperplasia or carcinoma,

or uterine abnormalities. Age is not a factor in determining the risks associated with
IUD use. Most IUD users are over the age of 30. Although there is a slightly higher
risk for infertility in women who have never been pregnant, the IUD is an acceptable
option as long as the risk-benefit ratio is discussed. IUDs may be inserted
immediately after delivery, but this is not recommended because of the increased risk
and rate of expulsion at this time.
55.
C. During the third trimester, the enlarging uterus places pressure on the
intestines. This coupled with the effect of hormones on smooth muscle relaxation
causes decreased intestinal motility (peristalsis). Increasing fiber in the diet will help
fecal matter pass more quickly through the intestinal tract, thus decreasing the
amount of water that is absorbed. As a result, stool is softer and easier to pass.
Enemas could precipitate preterm labor and/or electrolyte loss and should be
avoided. Laxatives may cause preterm labor by stimulating peristalsis and may
interfere with the absorption of nutrients. Use for more than 1 week can also lead to
laxative dependency. Liquid in the diet helps provide a semisolid, soft consistency to
the stool. Eight to ten glasses of fluid per day are essential to maintain hydration and
promote stool evacuation.
56.
D. To ensure adequate fetal growth and development during the 40 weeks of a
pregnancy, a total weight gain 25 to 30 pounds is recommended: 1.5 pounds in the
first 10 weeks; 9 pounds by 30 weeks; and 27.5 pounds by 40 weeks. The pregnant
woman should gain less weight in the first and second trimester than in the third.
During the first trimester, the client should only gain 1.5 pounds in the first 10 weeks,
not 1 pound per week. A weight gain of pound per week would be 20 pounds for
the total pregnancy, less than the recommended amount.
57.
B. To calculate the EDD by Nageles rule, add 7 days to the first day of the last
menstrual period and count back 3 months, changing the year appropriately. To
obtain a date of September 27, 7 days have been added to the last day of the LMP
(rather than the first day of the LMP), plus 4 months (instead of 3 months) were
counted back. To obtain the date of November 7, 7 days have been subtracted (instead
of added) from the first day of LMP plus November indicates counting back 2 months
(instead of 3 months) from January. To obtain the date of December 27, 7 days were
added to the last day of the LMP (rather than the first day of the LMP) and December
indicates counting back only 1 month (instead of 3 months) from January.
58.
D. The client has been pregnant four times, including current pregnancy (G).
Birth at 38 weeks gestation is considered full term (T), while birth form 20 weeks to
38 weeks is considered preterm (P). A spontaneous abortion occurred at 8 weeks (A).
She has two living children (L).
59.
B. At 12 weeks gestation, the uterus rises out of the pelvis and is palpable
above the symphysis pubis. The Doppler intensifies the sound of the fetal pulse rate
so it is audible. The uterus has merely risen out of the pelvis into the abdominal cavity
and is not at the level of the umbilicus. The fetal heart rate at this age is not audible
with a stethoscope. The uterus at 12 weeks is just above the symphysis pubis in the
abdominal cavity, not midway between the umbilicus and the xiphoid process. At 12
weeks the FHR would be difficult to auscultate with a fetoscope. Although the
external electronic fetal monitor would project the FHR, the uterus has not risen to
the umbilicus at 12 weeks.

A. Although all of the choices are important in the management of diabetes,


diet therapy is the mainstay of the treatment plan and should always be the priority.
Women diagnosed with gestational diabetes generally need only diet therapy without
medication to control their blood sugar levels. Exercise, is important for all pregnant
women and especially for diabetic women, because it burns up glucose, thus
decreasing blood sugar. However, dietary intake, not exercise, is the priority. All
pregnant women with diabetes should have periodic monitoring of serum glucose.
However, those with gestational diabetes generally do not need daily glucose
monitoring. The standard of care recommends a fasting and 2-hour postprandial
blood sugar level every 2 weeks.
61.
C. After 20 weeks gestation, when there is a rapid weight gain, preeclampsia
should be suspected, which may be caused by fluid retention manifested by edema,
especially of the hands and face. The three classic signs of preeclampsia
are hypertension, edema, and proteinuria. Although urine is checked for glucose at
each clinic visit, this is not the priority. Depression may cause either anorexia or
excessive food intake, leading to excessive weight gain or loss. This is not, however,
the priority consideration at this time. Weight gain thought to be caused by excessive
food intake would require a 24-hour diet recall. However, excessive intake would not
be the primary consideration for this client at this time.
62.
B. Cramping and vaginal bleeding coupled with cervical dilation signifies that
termination of the pregnancy is inevitable and cannot be prevented. Thus, the nurse
would document an imminent abortion. In a threatened abortion, cramping and
vaginal bleeding are present, but there is no cervical dilation. The symptoms may
subside or progress to abortion. In a complete abortion all the products of conception
are expelled. A missed abortion is early fetal intrauterine death without expulsion of
the products of conception.
63.
B. For the client with an ectopic pregnancy, lower abdominal pain, usually
unilateral, is the primary symptom. Thus, pain is the priority. Although the potential
for infection is always present, the risk is low in ectopic pregnancy because
pathogenic microorganisms have not been introduced from external sources. The
client may have a limited knowledge of the pathology and treatment of the condition
and will most likely experience grieving, but this is not the priority at this time.
64.
D. Before uterine assessment is performed, it is essential that the woman
empty her bladder. A full bladder will interfere with the accuracy of the assessment by
elevating the uterus and displacing to the side of the midline. Vital sign assessment is
not necessary unless an abnormality in uterine assessment is identified. Uterine
assessment should not cause acute pain that requires administration of analgesia.
Ambulating the client is an essential component of postpartum care, but is not
necessary prior to assessment of the uterus.
65.
A. Feeding more frequently, about every 2 hours, will decrease the infants
frantic, vigorous sucking from hunger and will decrease breast engorgement, soften
the breast, and promote ease of correct latching-on for feeding. Narcotics
administered prior to breast feeding are passed through the breast milk to the infant,
causing excessive sleepiness. Nipple soreness is not severe enough to warrant narcotic
analgesia. All postpartum clients, especially lactating mothers, should wear a
supportive brassiere with wide cotton straps. This does not, however, prevent or
reduce nipple soreness. Soaps are drying to the skin of the nipples and should not be
60.

used on the breasts of lactating mothers. Dry nipple skin predisposes to cracks and
fissures, which can become sore and painful.
66.
D. A weak, thready pulse elevated to 100 BPM may indicate impending
hemorrhagic shock. An increased pulse is a compensatory mechanism of the body in
response to decreased fluid volume. Thus, the nurse should check the amount of
lochia present. Temperatures up to 100.48F in the first 24 hours after birth are
related to the dehydrating effects of labor and are considered normal. Although
rechecking the blood pressure may be a correct choice of action, it is not the first
action that should be implemented in light of the other data. The data indicate a
potential impending hemorrhage. Assessing the uterus for firmness and position in
relation to the umbilicus and midline is important, but the nurse should check the
extent of vaginal bleeding first. Then it would be appropriate to check the uterus,
which may be a possible cause of the hemorrhage.
67.
D. Any bright red vaginal discharge would be considered abnormal, but
especially 5 days after delivery, when the lochia is typically pink to brownish. Lochia
rubra, a dark red discharge, is present for 2 to 3 days after delivery. Bright red vaginal
bleeding at this time suggests late postpartum hemorrhage, which occurs after the
first 24 hours following delivery and is generally caused by retained placental
fragments or bleeding disorders. Lochia rubra is the normal dark red discharge
occurring in the first 2 to 3 days after delivery, containing epithelial cells, erythrocyes,
leukocytes and decidua. Lochia serosa is a pink to brownish serosanguineous
discharge occurring from 3 to 10 days after delivery that contains decidua,
erythrocytes, leukocytes, cervical mucus, and microorganisms. Lochia alba is an
almost colorless to yellowish discharge occurring from 10 days to 3 weeks after
delivery and containing leukocytes, decidua, epithelial cells, fat, cervical mucus,
cholesterol crystals, and bacteria.
68.
A. The data suggests an infection of the endometrial lining of the uterus. The
lochia may be decreased or copious, dark brown in appearance, and foul smelling,
providing further evidence of a possible infection. All the clients data indicate a
uterine problem, not a breast problem. Typically, transient fever, usually 101F, may
be present with breast engorgement. Symptoms of mastitis include influenza-like
manifestations. Localized infection of an episiotomy or C-section incision rarely
causes systemic symptoms, and uterine involution would not be affected. The client
data do not include dysuria, frequency, or urgency, symptoms of urinary tract
infections, which would necessitate assessing the clients urine.
69.
C. Because of early postpartum discharge and limited time for teaching, the
nurses priority is to facilitate the safe and effective care of the client and newborn.
Although promoting comfort and restoration of health, exploring the familys
emotional status, and teaching about family planning are important in
postpartum/newborn nursing care, they are not the priority focus in the limited time
presented by early post-partum discharge.
70.
C. Heat loss by radiation occurs when the infants crib is placed too near cold
walls or windows. Thus placing the newborns crib close to the viewing window would
be least effective. Body heat is lost through evaporation during bathing. Placing the
infant under the radiant warmer after bathing will assist the infant to be rewarmed.
Covering the scale with a warmed blanket prior to weighing prevents heat loss

through conduction. A knit cap prevents heat loss from the head a large head, a large
body surface area of the newborns body.
71.
B. A fractured clavicle would prevent the normal Moro response of
symmetrical sequential extension and abduction of the arms followed by flexion and
adduction. In talipes equinovarus (clubfoot) the foot is turned medially, and in
plantar flexion, with the heel elevated. The feet are not involved with the Moro reflex.
Hypothyroiddism has no effect on the primitive reflexes. Absence of the Moror reflex
is the most significant single indicator of central nervous system status, but it is not a
sign of increased intracranial pressure.
72.
B. Hemorrhage is a potential risk following any surgical procedure. Although
the infant has been given vitamin K to facilitate clotting, the prophylactic dose is often
not sufficient to prevent bleeding. Although infection is a possibility, signs will not
appear within 4 hours after the surgical procedure. The primary discomfort of
circumcision occurs during the surgical procedure, not afterward. Although feedings
are withheld prior to the circumcision, the chances of dehydration are minimal.
73.
B. The presence of excessive estrogen and progesterone in the maternal-fetal
blood followed by prompt withdrawal at birth precipitates breast engorgement, which
will spontaneously resolve in 4 to 5 days after birth. The trauma of the birth process
does not cause inflammation of the newborns breast tissue. Newborns do not have
breast malignancy. This reply by the nurse would cause the mother to have undue
anxiety. Breast tissue does not hypertrophy in the fetus or newborns.
74.
D. The first 15 minutes to 1 hour after birth is the first period of reactivity
involving respiratory and circulatory adaptation to extrauterine life. The data given
reflect the normal changes during this time period. The infants assessment data
reflect normal adaptation. Thus, the physician does not need to be notified and
oxygen is not needed. The data do not indicate the presence of choking, gagging or
coughing, which are signs of excessive secretions. Suctioning is not necessary.
75.
B. Application of 70% isopropyl alcohol to the cord minimizes microorganisms
(germicidal) and promotes drying. The cord should be kept dry until it falls off and
the stump has healed. Antibiotic ointment should only be used to treat an infection,
not as a prophylaxis. Infants should not be submerged in a tub of water until the cord
falls off and the stump has completely healed.
76.
B. To determine the amount of formula needed, do the following mathematical
calculation. 3 kg x 120 cal/kg per day = 360 calories/day feeding q 4 hours = 6
feedings per day = 60 calories per feeding: 60 calories per feeding; 60 calories per
feeding with formula 20 cal/oz = 3 ounces per feeding. Based on the calculation. 2, 4
or 6 ounces are incorrect.
77.
A. Intrauterine anoxia may cause relaxation of the anal sphincter and
emptying of meconium into the amniotic fluid. At birth some of the meconium fluid
may be aspirated, causing mechanical obstruction or chemical pneumonitis. The
infant is not at increased risk for gastrointestinal problems. Even though the skin is
stained with meconium, it is noninfectious (sterile) and nonirritating. The postterm
meconium-stained infant is not at additional risk for bowel or urinary problems.
78.
C. The nurse should use a nonelastic, flexible, paper measuring tape, placing
the zero point on the superior border of the symphysis pubis and stretching the tape
across the abdomen at the midline to the top of the fundus. The xiphoid and

umbilicus are not appropriate landmarks to use when measuring the height of the
fundus (McDonalds measurement).
79.
B. Women hospitalized with severe preeclampsia need decreased CNS
stimulation to prevent a seizure. Seizure precautions provide environmental safety
should a seizure occur. Because of edema, daily weight is important but not the
priority. Preclampsia causes vasospasm and therefore can reduce utero-placental
perfusion. The client should be placed on her left side to maximize blood flow, reduce
blood pressure, and promote diuresis. Interventions to reduce stress and anxiety are
very important to facilitate coping and a sense of control, but seizure precautions are
the priority.
80.
C. Cessation of the lochial discharge signifies healing of the endometrium. Risk
of hemorrhage and infection are minimal 3 weeks after a normal vaginal delivery.
Telling the client anytime is inappropriate because this response does not provide the
client with the specific information she is requesting. Choice of a contraceptive
method is important, but not the specific criteria for safe resumption of sexual
activity. Culturally, the 6-weeks examination has been used as the time frame for
resuming sexual activity, but it may be resumed earlier.
81.
C. The middle third of the vastus lateralis is the preferred injection site for
vitamin K administration because it is free of blood vessels and nerves and is large
enough to absorb the medication. The deltoid muscle of a newborn is not large
enough for a newborn IM injection. Injections into this muscle in a small child might
cause damage to the radial nerve. The anterior femoris muscle is the next safest
muscle to use in a newborn but is not the safest. Because of the proximity of the
sciatic nerve, the gluteus maximus muscle should not be until the child has been
walking 2 years.
82.
D. Bartholins glands are the glands on either side of the vaginal orifice. The
clitoris is female erectile tissue found in the perineal area above the urethra. The
parotid glands are open into the mouth. Skenes glands open into the posterior wall of
the female urinary meatus.
83.
D. The fetal gonad must secrete estrogen for the embryo to differentiate as a
female. An increase in maternal estrogen secretion does not effect differentiation of
the embryo, and maternal estrogen secretion occurs in every pregnancy. Maternal
androgen secretion remains the same as before pregnancy and does not effect
differentiation. Secretion of androgen by the fetal gonad would produce a male fetus.
84.
A. Using bicarbonate would increase the amount of sodium ingested, which
can cause complications. Eating low-sodium crackers would be appropriate. Since
liquids can increase nausea avoiding them in the morning hours when nausea is
usually the strongest is appropriate. Eating six small meals a day would keep the
stomach full, which often decrease nausea.
85.
B. Ballottement indicates passive movement of the unengaged fetus.
Ballottement is not a contraction. Fetal kicking felt by the client represents
quickening. Enlargement and softening of the uterus is known as Piskaceks sign.
86.
B. Chadwicks sign refers to the purple-blue tinge of the cervix. Braxton Hicks
contractions are painless contractions beginning around the 4 th month. Goodells sign
indicates softening of the cervix. Flexibility of the uterus against the cervix is known
as McDonalds sign.

C. Breathing techniques can raise the pain threshold and reduce the perception
of pain. They also promote relaxation. Breathing techniques do not eliminate pain,
but they can reduce it. Positioning, not breathing, increases uteroplacental perfusion.
88.
A. The clients labor is hypotonic. The nurse should call the physical and obtain
an order for an infusion of oxytocin, which will assist the uterus to contact more
forcefully in an attempt to dilate the cervix. Administering light sedative would be
done for hypertonic uterine contractions. Preparing for cesarean section is
unnecessary at this time. Oxytocin would increase the uterine contractions and
hopefully progress labor before a cesarean would be necessary. It is too early to
anticipate client pushing with contractions.
89.
D. The signs indicate placenta previa and vaginal exam to determine cervical
dilation would not be done because it could cause hemorrhage. Assessing maternal
vital signs can help determine maternal physiologic status. Fetal heart rate is
important to assess fetal well-being and should be done. Monitoring the contractions
will help evaluate the progress of labor.
90.
D. A complete placenta previa occurs when the placenta covers the opening of
the uterus, thus blocking the passageway for the baby. This response explains what a
complete previa is and the reason the baby cannot come out except by cesarean
delivery. Telling the client to ask the physician is a poor response and would increase
the patients anxiety. Although a cesarean would help to prevent hemorrhage, the
statement does not explain why the hemorrhage could occur. With a complete previa,
the placenta is covering all the cervix, not just most of it.
91.
B. With a face presentation, the head is completely extended. With a vertex
presentation, the head is completely or partially flexed. With a brow (forehead)
presentation, the head would be partially extended.
92.
D. With this presentation, the fetal upper torso and back face the left upper
maternal abdominal wall. The fetal heart rate would be most audible above the
maternal umbilicus and to the left of the middle. The other positions would be
incorrect.
93.
C. The greenish tint is due to the presence of meconium. Lanugo is the soft,
downy hair on the shoulders and back of the fetus. Hydramnios represents excessive
amniotic fluid. Vernix is the white, cheesy substance covering the fetus.
94.
D. In a breech position, because of the space between the presenting part and
the cervix, prolapse of the umbilical cord is common. Quickening is the womans first
perception of fetal movement. Ophthalmia neonatorum usually results from maternal
gonorrhea and is conjunctivitis. Pica refers to the oral intake of nonfood substances.
95.
A. Dizygotic (fraternal) twins involve two ova fertilized by separate sperm.
Monozygotic (identical) twins involve a common placenta, same genotype, and
common chorion.
96.
C. The zygote is the single cell that reproduces itself after conception. The
chromosome is the material that makes up the cell and is gained from each parent.
Blastocyst and trophoblast are later terms for the embryo after zygote.
97.
D. Prepared childbirth was the direct result of the 1950s challenging of the
routine use of analgesic and anesthetics during childbirth. The LDRP was a much
later concept and was not a direct result of the challenging of routine use of analgesics
and anesthetics during childbirth. Roles for nurse midwives and clinical nurse
specialists did not develop from this challenge.
87.

C. The ischial spines are located in the mid-pelvic region and could be
narrowed due to the previous pelvic injury. The symphysis pubis, sacral promontory,
and pubic arch are not part of the mid-pelvis.
99.
B. Variations in the length of the menstrual cycle are due to variations in the
proliferative phase. The menstrual, secretory and ischemic phases do not contribute
to this variation.
100.
B. Testosterone is produced by the Leyding cells in the seminiferous tubules.
Follicle-stimulating hormone and leuteinzing hormone are released by the anterior
pituitary gland. The hypothalamus is responsible for releasing gonadotropin-releasing
hormone.
101.
D. The anterior fontanelle typically closes anywhere between 12 to 18 months
of age. Thus, assessing the anterior fontanelle as still being slightly open is a normal
finding requiring no further action. Because it is normal finding for this age, notifying
he physician or performing additional examinations are inappropriate.
102.
D. Solid foods are not recommended before age 4 to 6 months because of the
sucking reflex and the immaturity of the gastrointestinal tract and immune system.
Therefore, the earliest age at which to introduce foods is 4 months. Any time earlier
would be inappropriate.
103.
A. According to Erikson, infants need to have their needs met consistently and
effectively to develop a sense of trust. An infant whose needs are consistently unmet
or who experiences significant delays in having them met, such as in the case of the
infant of a substance-abusing mother, will develop a sense of uncertainty, leading to
mistrust of caregivers and the environment. Toddlers develop a sense of shame when
their autonomy needs are not met consistently. Preschoolers develop a sense of guilt
when their sense of initiative is thwarted. Schoolagers develop a sense of inferiority
when they do not develop a sense of industry.
104.
D. A busy box facilitates the fine motor development that occurs between 4
and 6 months. Balloons are contraindicated because small children may aspirate
balloons. Because the button eyes of a teddy bear may detach and be aspirated, this
toy is unsafe for children younger than 3 years. A 5-month-old is too young to use a
push-pull toy.
105.
B. Infants need to have their security needs met by being held and cuddled. At
2 months of age, they are unable to make the connection between crying and
attention. This association does not occur until late infancy or early toddlerhood.
Letting the infant cry for a time before picking up the infant or leaving the infant
alone to cry herself to sleep interferes with meeting the infants need for security at
this very young age. Infants cry for many reasons. Assuming that the child s hungry
may cause overfeeding problems such as obesity.
106.
B. Underdeveloped abdominal musculature gives the toddler a
characteristically protruding abdomen. During toddlerhood, food intake decreases,
not increases. Toddlers are characteristically bowlegged because the leg muscles must
bear the weight of the relatively large trunk. Toddler growth patterns occur in a
steplike, not linear pattern.
107.
B. According to Erikson, toddlers experience a sense of shame when they are
not allowed to develop appropriate independence and autonomy. Infants develop
mistrust when their needs are not consistently gratified. Preschoolers develop guilt
98.

when their initiative needs are not met while schoolagers develop a sense of
inferiority when their industry needs are not met.
108.
C. Young toddlers are still sensorimotor learners and they enjoy the experience
of feeling different textures. Thus, finger paints would be an appropriate toy choice.
Multiple-piece toys, such as puzzle, are too difficult to manipulate and may be
hazardous if the pieces are small enough to be aspirated. Miniature cars also have a
high potential for aspiration. Comic books are on too high a level for toddlers.
Although they may enjoy looking at some of the pictures, toddlers are more likely to
rip a comic book apart.
109.
D. The child must be able to sate the need to go to the bathroom to initiate
toilet training. Usually, a child needs to be dry for only 2 hours, not 4 hours. The child
also must be able to sit, walk, and squat. A new sibling would most likely hinder toilet
training.
110.
A. Toddlers become picky eaters, experiencing food jags and eating large
amounts one day and very little the next. A toddlers food gags express a preference
for the ritualism of eating one type of food for several days at a time. Toddlers
typically enjoy socialization and limiting others at meal time. Toddlers prefer to feed
themselves and thus are too young to have table manners. A toddlers appetite and
need for calories, protein, and fluid decrease due to the dramatic slowing of growth
rate.
111.
D. Preschoolers commonly have fears of the dark, being left alone especially at
bedtime, and ghosts, which may affect the childs going to bed at night. Quiet play and
time with parents is a positive bedtime routine that provides security and also readies
the child for sleep. The child should sleep in his own bed. Telling the child about
locking him in his room will viewed by the child as a threat. Additionally, a locked
door is frightening and potentially hazardous. Vigorous activity at bedtime stirs up
the child and makes more difficult to fall asleep.
112.
B. Dress-up clothes enhance imaginative play and imagination, allowing
preschoolers to engage in rich fantasy play. Building blocks and wooden puzzles are
appropriate for encouraging fine motor development. Big wheels and tricycles
encourage gross motor development.
113.
D. The school-aged child is in the stage of concrete operations, marked by
inductive reasoning, logical operations, and reversible concrete thought. The ability to
consider the future requires formal thought operations, which are not developed until
adolescence. Collecting baseball cards and marbles, ordering dolls by size, and simple
problem-solving options are examples of the concrete operational thinking of the
schoolager.
114.
C. Reaction formation is the schoolagers typical defensive response when
hospitalized. In reaction formation, expression of unacceptable thoughts or behaviors
is prevented (or overridden) by the exaggerated expression of opposite thoughts or
types of behaviors. Regression is seen in toddlers and preshcoolers when they retreat
or return to an earlier level of development. Repression refers to the involuntary
blocking of unpleasant feelings and experiences from ones awareness.
Rationalization is the attempt to make excuses to justify unacceptable feelings or
behaviors.
115.
C. The schoolagers cognitive level is sufficiently developed to enable good
understanding of and adherence to rules. Thus, schoolagers should be able to

understand the potential dangers around them. With growth comes greater freedom
and children become more adventurous and daring. The school-aged child is also still
prone to accidents and home hazards, especially because of increased motor abilities
and independence. Plus the home hazards differ from other age groups. These
hazards, which are potentially lethal but tempting, may include firearms, alcohol, and
medications. School-age children begin to internalize their own controls and need less
outside direction. Plus the child is away from home more often. Some parental or
caregiver assistance is still needed to answer questions and provide guidance for
decisions and responsibilities.
116.
C. The most significant skill learned during the school-age period is reading.
During this time the child develops formal adult articulation patterns and learns that
words can be arranged in structure. Collective, ordering, and sorting, although
important, are not most significant skills learned.
117.
C. Based on the recommendations of the American Academy of Family
Physicians and the American Academy of Pediatrics, the MMR vaccine should be
given at the age of 10 if the child did not receive it between the ages of 4 to 6 years as
recommended. Immunization for diphtheria and tetanus is required at age 13.
118.
D. According to Erikson, role diffusion develops when the adolescent does not
develop a sense of identity and a sense or where he fits in. Toddlers develop a sense of
shame when they do not achieve autonomy. Preschoolers develop a sense of guilt
when they do not develop a sense of initiative. School-age children develop a sense of
inferiority when they do not develop a sense of industry.
119.
A. Menarche refers to the onset of the first menstruation or menstrual period
and refers only to the first cycle. Uterine growth and broadening of the pelvic girdle
occurs before menarche.
120.
A. Stating that this is probably the only concern the adolescent has and telling
the parents not to worry about it or the time her spends on it shuts off further
investigation and is likely to make the adolescent and his parents feel defensive. The
statement about peer acceptance and time spent in front of the mirror for the
development of self image provides information about the adolescents needs to the
parents and may help to gain trust with the adolescent. Asking the adolescent how he
feels about the acne will encourage the adolescent to share his feelings. Discussing the
cleansing method shows interest and concern for the adolescent and also can help to
identify any patient-teaching needs for the adolescent regarding cleansing.
121.
B. Preschoolers should be developmentally incapable of demonstrating explicit
sexual behavior. If a child does so, the child has been exposed to such behavior, and
sexual abuse should be suspected. Explicit sexual behavior during doll play is not a
characteristic of preschool development nor symptomatic of developmental delay.
Whether or nor the child knows how to play with dolls is irrelevant.
122.
A. The parents need more teaching if they state that they will keep the child
home until the phobia subsides. Doing so reinforces the childs feelings of
worthlessness and dependency. The child should attend school even during resolution
of the problem. Allowing the child to verbalize helps the child to ventilate feelings and
may help to uncover causes and solutions. Collaboration with the teachers and
counselors at school may lead to uncovering the cause of the phobia and to the
development of solutions. The child should participate and play an active role in
developing possible solutions.

123.
C. The adolescent who becomes pregnant typically denies the pregnancy early
on. Early recognition by a parent or health care provider may be crucial to timely
initiation of prenatal care. The incidence of adolescent pregnancy has declined since
1991, yet morbidity remains high. Most teenage pregnancies are unplanned and occur
out of wedlock. The pregnant adolescent is at high risk for physical complications
including premature labor and low-birth-weight infants, high neonatal mortality, iron
deficiency anemia, prolonged labor, and fetopelvic disproportion as well as numerous
psychological crises.
124.
B. Because of the structural defect, children with cleft palate may have
ineffective functioning of their Eustachian tubes creating frequent bouts of otitis
media. Most children with cleft palate remain well-nourished and maintain adequate
nutrition through the use of proper feeding techniques. Food particles do not pass
through the cleft and into the Eustachian tubes. There is no association between cleft
palate and congenial ear deformities.
125.
D. A 3-month-old infant should be able to lift the head and chest when prone.
The Moro reflex typically diminishes or subsides by 3 months. The parachute reflex
appears at 9 months. Rolling from front to back usually is accomplished at about 5
months.
126.
D. A childs birth weight usually triples by 12 months and doubles by 4
months. No specific birth weight parameters are established for 7 or 9 months.
127.
C. Toddlers engaging in parallel play will play near each other, but not with
each other. Thus, when two toddlers sit near each other but play with separate dolls,
they are exhibiting parallel play. Sharing crayons, playing a board game with a nurse,
or sharing dolls with two different nurses are all examples of cooperative play.
128.
A. Acute lymphocytic leukemia (ALL) causes leukopenia, resulting in
immunosuppression and increasing the risk of infection, a leading cause of death in
children with ALL. Therefore, the initial priority nursing intervention would be to
institute infection control precautions to decrease the risk of infection. Iron-rich foods
help with anemia, but dietary iron is not an initial intervention. The prognosis of ALL
usually is good. However, later on, the nurse may need to assist the child and family
with coping since death and dying may still be an issue in need of discussion.
Injections should be discouraged, owing to increased risk from bleeding due to
thrombocytopenia.
129.
A. The pertusis component may result in fever and the tetanus component
may result in injection soreness. Therefore, the mothers verbalization of information
about measures to reduce fever indicates understanding. No dietary restrictions are
necessary after this injection is given. A subsequent rash is more likely to be seen 5 to
10 days after receiving the MMR vaccine, not the diphtheria, pertussis, and tetanus
vaccine. Diarrhea is not associated with this vaccine.
130.
A. Multiple bruises and burns on a toddler are signs child abuse. Therefore,
the nurse is responsible for reporting the case to Protective Services immediately to
protect the child from further harm. Scheduling a follow-up visit is inappropriate
because additional harm may come to the child if the nurse waits for further
assessment data. Although the nurse should notify the physician, the goal is to initiate
measures to protect the childs safety. Notifying the physician immediately does not
initiate the removal of the child from harm nor does it absolve the nurse from
responsibility. Multiple bruises and burns are not normal toddler injuries.

131.
B. The mother is using projection, the defense mechanism used when a person
attributes his or her own undesirable traits to another. Displacement is the transfer of
emotion onto an unrelated object, such as when the mother would kick a chair or
bang the door shut. Repression is the submerging of painful ideas into the
unconscious. Psychosis is a state of being out of touch with reality.
132.
A. Children with congenital heart disease are more prone to respiratory
infections. Bleeding tendencies, frequent vomiting, and diarrhea and seizure
disorders are not associated with congenital heart disease.
133.
D. The child is exhibiting classic signs of epiglottitis, always a pediatric
emergency. The physician must be notified immediately and the nurse must be
prepared for an emergency intubation or tracheostomy. Further assessment with
auscultating lungs and placing the child in a mist tent wastes valuable time. The
situation is a possible life-threatening emergency. Having the child lie down would
cause additional distress and may result in respiratory arrest. Throat examination
may result in laryngospasm that could be fatal.
134.
A. In females, the urethra is shorter than in males. This decreases the distance
for organisms to travel, thereby increasing the chance of the child developing a
urinary tract infection. Frequent emptying of the bladder would help to decrease
urinary tract infections by avoiding sphincter stress. Increased fluid intake enables
the bladder to be cleared more frequently, thus helping to prevent urinary tract
infections. The intake of acidic juices helps to keep the urine pH acidic and thus
decrease the chance of flora development.
135.
B. Compartment syndrome is an emergent situation and the physician needs
to be notified immediately so that interventions can be initiated to relieve the
increasing pressure and restore circulation. Acetaminophen (Tylenol) will be
ineffective since the pain is related to the increasing pressure and tissue ischemia. The
cast, not traction, is being used in this situation for immobilization, so releasing the
traction would be inappropriate. In this situation, specific action not continued
monitoring is indicated.
136.
D. The varicella zoster vaccine (VZV) is a live vaccine given after age 12
months. The first dose of hepatitis B vaccine is given at birth to 2 months, then at 1 to
4 months, and then again at 6 to 18 months. DtaP is routinely given at 2, 4, 6, and 15
to 18 months and a booster at 4 to 6 years.
137.
C. Because the 8-month-old is refining his gross motor skills, being able to sit
unsupported and also improving his fine motor skills, probably capable of making
hand-to-hand transfers, large blocks would be the most appropriate toy selection.
Push-pull toys would be more appropriate for the 10 to 12-month-old as he or she
begins to cruise the environment. Rattles and mobiles are more appropriate for
infants in the 1 to 3 month age range. Mobiles pose a danger to older infants because
of possible strangulation.
138.
B. During the preschool period, the child has mastered a sense of autonomy
and goes on to master a sense of initiative. During this period, the child commonly
experiences more fears than at any other time. One common fear is fear of the body
mutilation, especially associated with painful experiences. The preschool child uses
simple, not complex, reasoning, engages in associative, not competitive, play
(interactive and cooperative play with sharing), and is able to tolerate longer periods
of delayed gratification.

139.
A. Mild mental retardation refers to development disability involving an IQ 50
to 70. Typically, the child is not noted as being retarded, but exhibits slowness in
performing tasks, such as self-feeding, walking, and taking. Little or no speech,
marked motor delays, and gait disabilities would be seen in more severe forms mental
retardation.
140.
B. Down syndrome is characterized by the following a transverse palmar
crease (simian crease), separated sagittal suture, oblique palpebral fissures, small
nose, depressed nasal bridge, high-arched palate, excess and lax skin, wide spacing
and plantar crease between the second and big toes, hyperextensible and lax joints,
large protruding tongue, and muscle weakness.
141.
A. Because of the defect, the child will be unable to from the mouth adequately
around nipple, thereby requiring special devices to allow for feeding and sucking
gratification. Respiratory status may be compromised if the child is fed improperly or
during postoperative period, Locomotion would be a problem for the older infant
because of the use of restraints. GI functioning is not compromised in the child with a
cleft lip.
142.
B. Postoperatively children with cleft palate should be placed on their
abdomens to facilitate drainage. If the child is placed in the supine position, he or she
may aspirate. Using an infant seat does not facilitate drainage. Side-lying does not
facilitate drainage as well as the prone position.
143.
C. Projectile vomiting is a key symptom of pyloric stenosis. Regurgitation is
seen more commonly with GER. Steatorrhea occurs in malabsorption disorders such
as celiac disease. Currant jelly stools are characteristic of intussusception.
144.
D. GER is the backflow of gastric contents into the esophagus resulting from
relaxation or incompetence of the lower esophageal (cardiac) sphincter. No alteration
in the oral mucous membranes occurs with this disorder. Fluid volume deficit, risk for
aspiration, and altered nutrition are appropriate nursing diagnoses.
145.
A. Thickened feedings are used with GER to stop the vomiting. Therefore, the
nurse would monitor the childs vomiting to evaluate the effectiveness of using the
thickened feedings. No relationship exists between feedings and characteristics of
stools and uterine. If feedings are ineffective, this should be noted before there is any
change in the childs weight.
146.
C. Children with celiac disease cannot tolerate or digest gluten. Therefore,
because of its gluten content, wheat and wheat-containing products must be avoided.
Rice, milk, and chicken do not contain gluten and need not be avoided.
147.
C. Episodes of celiac crises are precipitated by infections, ingestion of gluten,
prolonged fasting, or exposure to anticholinergic drugs. Celiac crisis is typically
characterized by severe watery diarrhea. Respiratory distress is unlikely in a routine
upper respiratory infection. Irritability, rather than lethargy, is more likely. Because
of the fluid loss associated with the severe watery diarrhea, the childs weight is more
likely to be decreased.
148.
A. For the child with Hirschsprung disease, fever and explosive diarrhea
indicate enterocolitis, a life-threatening situation. Therefore, the physician should be
notified immediately. Generally, because of the intestinal obstruction and inadequate
propulsive intestinal movement, antidiarrheals are not used to treat Hirschsprung
disease. The child is acutely ill and requires intervention, with monitoring more

frequently than every 30 minutes. Hirschsprung disease typically presents with


chronic constipation.
149.
A. Failure to pass meconium within the first 24 hours after birth may be an
indication of Hirschsprung disease, a congenital anomaly resulting in mechanical
obstruction due to inadequate motility in an intestinal segment. Failure to pass
meconium is not associated with celiac disease, intussusception, or abdominal wall
defect.
150.
C. Because intussusception is not believed to have a familial tendency,
obtaining a family history would provide the least amount of information. Stool
inspection, pain pattern, and abdominal palpation would reveal possible indicators of
intussusception. Current, jelly-like stools containing blood and mucus are an
indication of intussusception. Acute, episodic abdominal pain is characteristics of
intussusception. A sausage-shaped mass may be palpated in the right upper quadrant.

MEDICAL-SURGICAL NURSING PRACTICE QUESTIONS WITH


RATIONALE
10:40 PM MEDICAL-SURGICAL NURSING REVIEWER 2 comments
1. Following surgery, Mario complains of mild incisional pain while performing deepbreathing and coughing exercises. The nurses best response would be:
A. Pain will become less each day.
B. This is a normal reaction after surgery.
C. With a pillow, apply pressure against the incision.
D. I will give you the pain medication the physician ordered.
Answer: (C) With a pillow, apply pressure against the incision.
Applying pressure against the incision with a pillow will help lessen the intra-abdominal
pressure created by coughing which causes tension on the incision that leads to pain.
2. The nurse needs to carefully assess the complaint of pain of the elderly because older
people
A. are expected to experience chronic pain
B. have a decreased pain threshold
C. experience reduced sensory perception
D. have altered mental function
Answer: (C) experience reduced sensory perception
Degenerative changes occur in the elderly. The response to pain in the elderly maybe
lessened because of reduced acuity of touch, alterations in neural pathways and
diminished processing of sensory data.
3. Mary received AtropineSO4 as a pre-medication 30 minutes ago and is now
complaining of dry mouth and her PR is higher, than before the medication was
administered. The nurses best
A. The patient is having an allergic reaction to the drug.
B. The patient needs a higher dose of this drug

C. This is normal side-effect of AtSO4


D. The patient is anxious about upcoming surgery
Answer: (C) This is normal side-effect of AtSO4
Atropine sulfate is a vagolytic drug that decreases oropharyngeal secretions and
increases the heart rate.
4. Anas postoperative vital signs are a blood pressure of 80/50 mm Hg, a pulse of 140,
and respirations of 32. Suspecting shock, which of the following orders would the nurse
question?
A. Put the client in modified Trendelenberg's position.
B. Administer oxygen at 100%.
C. Monitor urine output every hour.
D. Administer Demerol 50mg IM q4h
Answer: (D) Administer Demerol 50mg IM q4h
Administering Demerol, which is a narcotic analgesic, can depress respiratory and
cardiac function and thus not given to a patient in shock. What is needed is promotion
for adequate oxygenation and perfusion. All the other interventions can be expected to
be done by the nurse.
5. Mr. Pablo, diagnosed with Bladder Cancer, is scheduled for a cystectomy with the
creation of an ileal conduit in the morning. He is wringing his hands and pacing the
floor when the nurse enters his room. What is the best approach?
A. "Good evening, Mr. Pablo. Wasn't it a pleasant day, today?"
B. "Mr, Pablo, you must be so worried, I'll leave you alone with your thoughts.
C. Mr. Pablo, you'll wear out the hospital floors and yourself at this rate."
D. "Mr. Pablo, you appear anxious to me. How are you feeling about tomorrow's
surgery?"
Answer: (D) "Mr. Pablo, you appear anxious to me. How are you feeling about
tomorrow's surgery?"
The client is showing signs of anxiety reaction to a stressful event. Recognizing the
clients anxiety conveys acceptance of his behavior and will allow for verbalization of
feelings and concerns.
6. After surgery, Gina returns from the Post-anesthesia Care Unit (Recovery Room) with
a nasogastric tube in place following a gall bladder surgery. She continues to complain
of nausea. Which action would the nurse take?
A. Call the physician immediately.
B. Administer the prescribed antiemetic.
C. Check the patency of the nasogastric tube for any obstruction.
D. Change the patients position.
Answer: (C) Check the patency of the nasogastric tube for any obstruction.
Nausea is one of the common complaints of a patient after receiving general anesthesia.
But this complaint could be aggravated by gastric distention especially in a patient who

has undergone abdominal surgery. Insertion of the NGT helps relieve the problem.
Checking on the patency of the NGT for any obstruction will help the nurse determine
the cause of the problem and institute the necessary intervention.
7. Mr.
Perez
is in continuous pain from cancer that has metastasized to the bone. Pain medication
provides little relief and he refuses to move. The nurse should plan to:
A. Reassure him that the nurses will not hurt him
B. Let him perform his own activities of daily living
C. Handle him gently when assisting with required care
D. Complete A.M. care quickly as possible when necessary
Answer: (C) Handle him gently when assisting with required care
Patients with cancer and bone metastasis experience severe pain especially when
moving. Bone tumors weaken the bone to appoint at which normal activities and even
position changes can lead to fracture. During nursing care, the patient needs to be
supported and handled gently.
8. A client returns from the recovery room at 9AM alert and oriented, with an IV
infusing. His pulse is 82, blood pressure is 120/80, respirations are 20, and all are
within normal range. At 10 am and at 11 am, his vital signs are stable. At noon, however,
his pulse rate is 94, blood pressure is 116/74, and respirations are 24. What nursing
action is most appropriate?
A. Notify his physician.
B. Take his vital signs again in 15 minutes.
C. Take his vital signs again in an hour.
D. Place the patient in shock position.
Answer: (B) Take his vital signs again in 15 minutes.
Monitoring the clients vital signs following surgery gives the nurse a sound information
about the clients condition. Complications can occur during this period as a result of the
surgery or the anesthesia or both. Keeping close track of changes in the VS and
validating them will help the nurse initiate interventions to prevent complications from
occurring.
9. A 56 year old construction worker is brought to the hospital unconscious after falling
from a 2-story building. When assessing the client, the nurse would be most concerned
if the assessment revealed:
A. Reactive pupils
B. A depressed fontanel
C. Bleeding from ears
D. An elevated temperature
Answer: (C) Bleeding from ears

The nurse needs to perform a thorough assessment that could indicate alterations in
cerebral function, increased intracranial pressures, fractures and bleeding. Bleeding
from the ears occurs only with basal skull fractures that can easily contribute to
increased intracranial pressure and brain herniation
10. Which of the ff. statements by the client to the nurse indicates a risk factor for CAD?
A. I exercise every other day.
B. My father died of Myasthenia Gravis.
C. My cholesterol is 180.
D. I smoke 1 1/2 packs of cigarettes per day.
Answer: (D) I smoke 1 1/2 packs of cigarettes per day.
Smoking has been considered as one of the major modifiable risk factors for coronary
artery disease. Exercise and maintaining normal serum cholesterol levels help in its
prevention.
11. Mr. Braga was ordered Digoxin 0.25 mg. OD. Which is poor knowledge regarding
this drug?
A. It has positive inotropic and negative chronotropic effects
B. The positive inotropic effect will decrease urine output
C. Toxixity can occur more easily in the presence of hypokalemia, liver and renal
problems
D. Do not give the drug if the apical rate is less than 60 beats per minute.
Answer: (B) The positive inotropic effect will decrease urine output
Inotropic effect of drugs on the heart causes increase force of its contraction. This
increases cardiac output that improves renal perfusion resulting in an improved urine
output.
12. Valsalva maneuver can result in bradycardia. Which of the following activities will
not stimulate Valsalva's maneuver?
A. Use of stool softeners.
B. Enema administration
C. Gagging while toothbrushing.
D. Lifting heavy objects
Answer: (A) Use of stool softeners.
Straining or bearing down activities can cause vagal stimulation that leads to
bradycardia. Use of stool softeners promote easy bowel evacuation that prevents
straining or the valsalva maneuver.
13. The nurse is teaching the patient regarding his permanent artificial pacemaker.
Which information
given by the nurse shows her knowledge deficit about the artificial cardiac pacemaker?
A. take the pulse rate once a day, in the morning upon awakening
B. may be allowed to use electrical appliances

C. have regular follow up care


D. may engage in contact sports
Answer: (D) may engage in contact sports
The client should be advised by the nurse to avoid contact sports. This will prevent
trauma to the area of the pacemaker generator.
14. A patient with angina pectoris is being discharged home with nitroglycerine tablets.
Which of the
following instructions does the nurse include in the teaching?
A. When your chest pain begins, lie down, and place one tablet under your tongue. If
the pain continues, take another tablet in 5 minutes.
B. Place one tablet under your tongue. If the pain is not relieved in 15 minutes, go to the
hospital.
C. Continue your activity, and if the pain does not go away in 10 minutes, begin taking
the nitro tablets one every 5 minutes for 15 minutes, then go lie down.
D. Place one Nitroglycerine tablet under the tongue every five minutes for three doses.
Go to the hospital if the pain is unrelieved.
Answer: (D) Place one Nitroglycerine tablet under the tongue every five minutes for
three doses. Go to the hospital if the pain is unrelieved.
Angina pectoris is caused by myocardial ischemia related to decreased coronary blood
supply. Giving nitroglycerine will produce coronary vasodilation that improves the
coronary blood flow in 3 5 mins. If the chest pain is unrelieved, after three tablets,
there is a possibility of acute coronary occlusion that requires immediate medical
attention.
15. A client with chronic heart failure has been placed on a diet restricted to 2000mg. of
sodium per day. The client demonstrates adequate knowledge if behaviors are evident
such as not salting food and avoidance of which food?
A. Whole milk
B. Canned sardines
C. Plain nuts
D. Eggs
Answer: (B) Canned sardines
Canned foods are generally rich in sodium content as salt is used as the main
preservative.
16. A student nurse is assigned to a client who has a diagnosis of thrombophlebitis.
Which action by this team member is most appropriate?
A. Apply a heating pad to the involved site.
B. Elevate the client's legs 90 degrees.
C. Instruct the client about the need for bed rest.
D. Provide active range-of-motion exercises to both legs at least twice every shift.

Answer: (C) Instruct the client about the need for bed rest.
In a client with thrombophlebitis, bedrest will prevent the dislodgment of the clot in the
extremity which can lead to pulmonary embolism.
17. A client receiving heparin sodium asks the nurse how the drug works. Which of the
following points would the nurse include in the explanation to the client?
A. It dissolves existing thrombi.
B. It prevents conversion of factors that are needed in the formation of clots.
C. It inactivates thrombin that forms and dissolves existing thrombi.
D. It interferes with vitamin K absorption.
Answer: (B) It prevents conversion of factors that are needed in the formation of clots.
Heparin is an anticoagulant. It prevents the conversion of prothrombin to thrombin. It
does not dissolve a clot.
18. The nurse is conducting an education session for a group of smokers in a stop
smoking class.
Which finding would the nurse state as a common symptom of lung cancer? :
A. Dyspnea on exertion
B. Foamy, blood-tinged sputum
C. Wheezing sound on inspiration
D. Cough or change in a chronic cough
Answer: (D) Cough or change in a chronic cough
Cigarette smoke is a carcinogen that irritates and damages the respiratory epithelium.
The irritation causes the cough which initially maybe dry, persistent and unproductive.
As the tumor enlarges, obstruction of the airways occurs and the cough may become
productive due to infection.
19. Which is the most relevant knowledge about oxygen administration to a client with
COPD?
A. Oxygen at 1-2L/min is given to maintain the hypoxic stimulus for breathing.
B. Hypoxia stimulates the central chemoreceptors in the medulla that makes the client
breath.
C. Oxygen is administered best using a non-rebreathing mask
D. Blood gases are monitored using a pulse oximeter.
Answer: (A) Oxygen at 1-2L/min is given to maintain the hypoxic stimulus for
breathing.
COPD causes a chronic CO2 retention that renders the medulla insensitive to the CO2
stimulation for breathing. The hypoxic state of the client then becomes the stimulus for
breathing. Giving the clientoxygen in low concentrations will maintain the clients
hypoxic drive.

20. When suctioning mucus from a client's lungs, which nursing action would be least
appropriate?
A. Lubricate the catheter tip with sterile saline before insertion.
B. Use sterile technique with a two-gloved approach
C. Suction until the client indicates to stop or no longer than 20 second
D. Hyperoxygenate the client before and after suctioning
Answer: (C) Suction until the client indicates to stop or no longer than 20 second
One hazard encountered when suctioning a client is the development of hypoxia.
Suctioning sucks not only the secretions but also the gases found in the airways. This
can be prevented by suctioning the client for an average time of 5-10 seconds and not
more than 15 seconds and hyperoxygenating the client before and after suctioning.
21. Dr. Santos prescribes oral rifampin (Rimactane) and isoniazid (INH) for a client with
a positive Tuberculin skin test. When informing the client of this decision, the nurse
knows that the purpose of this choice of treatment is to
A. Cause less irritation to the gastrointestinal tract
B. Destroy resistant organisms and promote proper blood levels of the drugs
C. Gain a more rapid systemic effect
D. Delay resistance and increase the tuberculostatic effect
Answer: (D) Delay resistance and increase the tuberculostatic effect
Pulmonary TB is treated primarily with chemotherapeutic agents for 6-12 mons. A
prolonged treatment duration is necessary to ensure eradication of the organisms and to
prevent relapse. The increasing prevalence of drug resistance points to the need to begin
the treatment with drugs in combination. Using drugs in combination can delay the
drug resistance.
22. Mario undergoes a left thoracotomy and a partial pneumonectomy. Chest tubes are
inserted, and one-bottle water-seal drainage is instituted in the operating room. In the
postanesthesia care unit Mario is placed in Fowler's position on either his right
side or on his back to
A. Reduce incisional pain.
B. Facilitate ventilation of the left lung.
C. Equalize pressure in the pleural space.
D. Increase venous return
Answer: (B) Facilitate ventilation of the left lung.
Since only a partial pneumonectomy is done, there is a need to promote expansion of
this remaining Left lung by positioning the client on the opposite unoperated side.
23. A client with COPD is being prepared for discharge. The following are relevant
instructions to the client regarding the use of an oral inhaler EXCEPT
A. Breath in and out as fully as possible before placing the mouthpiece inside the mouth.
B. Inhale slowly through the mouth as the canister is pressed down

C. Hold his breath for about 10 seconds before exhaling


D. Slowly breath out through the mouth with pursed lips after inhaling the drug.
Answer: (D) Slowly breath out through the mouth with pursed lips after inhaling the
drug.
If the client breathes out through the mouth with pursed lips, this can easily force the
just inhaled drug out of the respiratory tract that will lessen its effectiveness.
24. A client is scheduled for a bronchoscopy. When teaching the client what to expect
afterward, the nurse's highest priority of information would be
A. Food and fluids will be withheld for at least 2 hours.
B. Warm saline gargles will be done q 2h.
C. Coughing and deep-breathing exercises will be done q2h.
D. Only ice chips and cold liquids will be allowed initially.
Answer: (A) Food and fluids will be withheld for at least 2 hours.
Prior to bronchoscopy, the doctors sprays the back of the throat with anesthetic to
minimize the gag reflex and thus facilitate the insertion of the bronchoscope. Giving the
client food and drink after the procedure without checking on the return of the gag
reflex can cause the client to aspirate. The gag reflex usually returns after two hours.
25. The nurse enters the room of a client with chronic obstructive pulmonary disease.
The client's nasal cannula oxygen is running at a rate of 6 L per minute, the skin color is
pink, and the respirations are 9 per minute and shallow. What is the nurses best initial
action?
A. Take heart rate and blood pressure.
B. Call the physician.
C. Lower the oxygen rate.
D. Position the client in a Fowler's position.
Answer: (C) Lower the oxygen rate.
The client with COPD is suffering from chronic CO2 retention. The hypoxic drive is his
chief stimulus for breathing. Giving O2 inhalation at a rate that is more than 2-3L/min
can make the client lose his hypoxic drive which can be assessed as decreasing RR.
26. The nurse is preparing her plan of care for her patient diagnosed with pneumonia.
Which is the most appropriate nursing diagnosis for this patient?
A. Fluid volume deficit
B. Decreased tissue perfusion.
C. Impaired gas exchange.
D. Risk for infection
Answer: (C) Impaired gas exchange.
Pneumonia, which is an infection, causes lobar consolidation thus impairing gas
exchange between the alveoli and the blood. Because the patient would require adequate
hydration, this makes him prone to fluid volume excess.

27. A nurse at the weight loss clinic assesses a client who has a large abdomen and a
rounded face. Which additional assessment finding would lead the nurse to suspect that
the client has Cushings syndrome rather than obesity?
A. large thighs and upper arms
B. pendulous abdomen and large hips
C. abdominal striae and ankle enlargement
D. posterior neck fat pad and thin extremities
Answer: (D) posterior neck fat pad and thin extremities
Buffalo hump is the accumulation of fat pads over the upper back and neck. Fat may
also accumulate on the face. There is truncal obesity but the extremities are thin. All
these are noted in a client with Cushings syndrome.
28. Which statement by the client indicates understanding of the possible side effects of
Prednisone therapy?
A. I should limit my potassium intake because hyperkalemia is a side-effect of this
drug.
B. I must take this medicine exactly as my doctor ordered it. I shouldnt skip doses.
C. This medicine will protect me from getting any colds or infection.
D. My incision will heal much faster because of this drug.
Answer: (B) I must take this medicine exactly as my doctor ordered it. I shouldnt skip
doses.
The possible side effects of steroid administration are hypokalemia, increase tendency to
infection and poor wound healing. Clients on the drug must follow strictly the doctors
order since skipping the drug can lower the drug level in the blood that can trigger acute
adrenal insufficiency or Addisonian Crisis
29. A client, who is suspected of having Pheochromocytoma, complains of sweating,
palpitation and headache. Which assessment is essential for the nurse to make first?
A. Pupil reaction
B. Hand grips
C. Blood pressure
D. Blood glucose
Answer: (C) Blood pressure
Pheochromocytoma is a tumor of the adrenal medulla that causes an increase secretion
of catecholamines that can elevate the blood pressure.
30. The nurse is attending a bridal shower for a friend when another guest, who
happens to be a diabetic, starts to tremble and complains of dizziness. The next best
action for the nurse to take is to:
A. Encourage the guest to eat some baked macaroni
B. Call the guests personal physician
C. Offer the guest a cup of coffee
D. Give the guest a glass of orange juice

Answer: (D) Give the guest a glass of orange juice


In diabetic patients, the nurse should watch out for signs of hypoglycemia manifested by
dizziness, tremors, weakness, pallor diaphoresis and tachycardia. When this occurs in a
conscious client, he should be given immediately carbohydrates in the form of fruit
juice, hard candy, honey or, if unconscious, glucagons or dextrose per IV.
31. An adult, who is newly diagnosed with Graves disease, asks the nurse, Why do I
need to take
Propanolol (Inderal)? Based on the nurses understanding of the medication and
Graves
disease, the best response would be:
A. The medication will limit thyroid hormone secretion.
B. The medication limit synthesis of the thyroid hormones.
C. The medication will block the cardiovascular symptoms of Graves disease.
D. The medication will increase the synthesis of thyroid hormones.
Answer: (C) The medication will block the cardiovascular symptoms of Graves
disease.
Propranolol (Inderal) is a beta-adrenergic blocker that controls the cardiovascular
manifestations brought about by increased secretion of the thyroid hormone in Graves
disease.
32. During the first 24 hours after thyroid surgery, the nurse should include in her care:
A. Checking the back and sides of the operative dressing
B. Supporting the head during mild range of motion exercise
C. Encouraging the client to ventilate her feelings about the surgery
D. Advising the client that she can resume her normal activities immediately
Answer: (A) Checking the back and sides of the operative dressing
Following surgery of the thyroid gland, bleeding is a potential complication. This can
best be assessed by checking the back and the sides of the operative dressing as the
blood may flow towards the side and back leaving the front dry and clear of drainage.
33. On discharge, the nurse teaches the patient to observe for signs of surgically induced
hypothyroidism. The nurse would know that the patient understands the teaching when
she states she should notify the MD if she develops:
A. Intolerance to heat
B. Dry skin and fatigue
C. Progressive weight gain
D. Insomnia and excitability
Answer: (C) Progressive weight gain
Hypothyroidism, a decrease in thyroid hormone production, is characterized by
hypometabolism that manifests itself with weight gain.

34. What is the best reason for the nurse in instructing the client to rotate injection sites
for insulin?
A. Lipodystrophy can result and is extremely painful
B. Poor rotation technique can cause superficial hemorrhaging
C. Lipodystrophic areas can result, causing erratic insulin absorption rates from these
D. Injection sites can never be reused
Answer: (C) Lipodystrophic areas can result, causing erratic insulin absorption rates
from these
Lipodystrophy is the development of fibrofatty masses at the injection site caused by
repeated use of an injection site. Injecting insulin into these scarred areas can cause the
insulin to be poorly absorbed and lead to erratic reactions.
35. Which of the following would be inappropriate to include in a diabetic teaching
plan?
A. Change position hourly to increase circulation
B. Inspect feet and legs daily for any changes
C. Keep legs elevated on 2 pillows while sleeping
D. Keep the insulin not in use in the refrigerator
Answer: (C) Keep legs elevated on 2 pillows while sleeping
The client with DM has decreased peripheral circulation caused by microangiopathy.
Keeping the legs elevated during sleep will further cause circulatory impairment.
36. Included in the plan of care for the immediate post-gastroscopy period will be:
A. Maintain NGT to intermittent suction
B. Assess gag reflex prior to administration of fluids
C. Assess for pain and medicate as ordered
D. Measure abdominal girth every 4 hours
Answer: (B) Assess gag reflex prior to administration of fluids
The client, after gastroscopy, has temporary impairment of the gag reflex due to the
anesthetic that has been sprayed into his throat prior to the procedure. Giving fluids and
food at this time can lead to aspiration.
36. Included in the plan of care for the immediate post-gastroscopy period will be:
A. Maintain NGT to intermittent suction
B. Assess gag reflex prior to administration of fluids
C. Assess for pain and medicate as ordered
D. Measure abdominal girth every 4 hours
Answer: (B) Assess gag reflex prior to administration of fluids
The client, after gastroscopy, has temporary impairment of the gag reflex due to the
anesthetic that has been sprayed into his throat prior to the procedure. Giving fluids and
food at this time can lead to aspiration.
37. Which description of pain would be most characteristic of a duodenal ulcer?

A. Gnawing, dull, aching, hungerlike pain in the epigastric area that is relieved by food
intake
B. RUQ pain that increases after meal
C. Sharp pain in the epigastric area that radiates to the right shoulder
D. A sensation of painful pressure in the midsternal area
Answer: (A) Gnawing, dull, aching, hungerlike pain in the epigastric area that is relieved
by food intake
Duodenal ulcer is related to an increase in the secretion of HCl. This can be buffered by
food intake thus the relief of the pain that is brought about by food intake.
38. The client underwent Billroth surgery for gastric ulcer. Post-operatively, the
drainage from his NGT is thick and the volume of secretions has dramatically reduced in
the last 2 hours and the client feels like vomiting. The most appropriate nursing action is
to:
A.
B.
C.
D.

Reposition the NGT by advancing it gently NSS


Notify the MD of your findings
Irrigate the NGT with 50 cc of sterile
Discontinue the low-intermittent suction

Answer: (B) Notify the MD of your findings


The clients feeling of vomiting and the reduction in the volume of NGT drainage that is
thick are signs of possible abdominal distention caused by obstruction of the NGT. This
should be reported immediately to the MD to prevent tension and rupture on the site of
anastomosis caused by gastric distention.
39. After Billroth II Surgery, the client developed dumping syndrome. Which of the
following should
the nurse exclude in the plan of care?
A. Sit upright for at least 30 minutes after meals
B. Take only sips of H2O between bites of solid food
C. Eat small meals every 2-3 hours
D. Reduce the amount of simple carbohydrate in the diet
Answer: (A) Sit upright for at least 30 minutes after meals
The dumping syndrome occurs within 30 mins after a meal due to rapid gastric
emptying, causing distention of the duodenum or jejunum produced by a bolus of food.
To delay the emptying, the client has to lie down after meals. Sitting up after meals will
promote the dumping syndrome.
40. The laboratory of a male patient with Peptic ulcer revealed an elevated titer of
Helicobacter pylori.
Which of the following statements indicate an understanding of this data?
A. Treatment will include Ranitidine and Antibiotics

B. No treatment is necessary at this time


C. This result indicates gastric cancer caused by the organism
D. Surgical treatment is necessary
Answer: (A) Treatment will include Ranitidine and Antibiotics
One of the causes of peptic ulcer is H. Pylori infection. It releases toxin that destroys the
gastric and duodenal mucosa which decreases the gastric epitheliums resistance to acid
digestion. Giving antibiotics will control the infection and Ranitidine, which is a
histamine-2 blocker, will reduce acid secretion that can lead to ulcer.
41. What instructions should the client be given before undergoing a paracentesis?
A. NPO 12 hours before procedure
B. Empty bladder before procedure
C. Strict bed rest following procedure
D. Empty bowel before procedure
Answer: (B) Empty bladder before procedure
Paracentesis involves the removal of ascitic fluid from the peritoneal cavity through a
puncture made below the umbilicus. The client needs to void before the procedure to
prevent accidental puncture of a distended bladder during the procedure.
42. The husband of a client asks the nurse about the protein-restricted diet ordered
because of advanced liver disease. What statement by the nurse would best explain the
purpose of the diet?
A. The liver cannot rid the body of ammonia that is made by the breakdown of protein
in the digestive system.
B. The liver heals better with a high carbohydrates diet rather than protein.
C. Most people have too much protein in their diets. The amount of this diet is better
for liver healing.
D. Because of portal hyperemesis, the blood flows around the liver and ammonia made
from protein collects in the brain causing hallucinations.
Answer: (A) The liver cannot rid the body of ammonia that is made by the breakdown
of protein in the digestive system.
The largest source of ammonia is the enzymatic and bacterial digestion of dietary and
blood proteins in the GI tract. A protein-restricted diet will therefore decrease ammonia
production.
43. Which of the drug of choice for pain controls the patient with acute pancreatitis?
A. Morphine
B. NSAIDS
C. Meperidine
D. Codeine
Answer: (C) Meperidine

Pain in acute pancreatitis is caused by irritation and edema of the inflamed pancreas as
well as spasm due to obstruction of the pancreatic ducts. Demerol is the drug of choice
because it is less likely to cause spasm of the Sphincter of Oddi unlike Morphine which
is spasmogenic.
44. Immediately after cholecystectomy, the nursing action that should assume the
highest priority is:
A. encouraging the client to take adequate deep breaths by mouth
B. encouraging the client to cough and deep breathe
C. changing the dressing at least BID
D. irrigate the T-tube frequently
Answer: (B) encouraging the client to cough and deep breathe
Cholecystectomy requires a subcostal incision. To minimize pain, clients have a
tendency to take shallow breaths which can lead to respiratory complications like
pneumonia and atelectasis. Deep breathing and coughing exercises can help prevent
such complications.
45. A Sengstaken-Blakemore tube is inserted in the effort to stop the bleeding
esophageal varices in a patient with complicated liver cirrhosis. Upon insertion of the
tube, the client complains of difficulty of breathing. The first action of the nurse is to:
A.
B.
C.
D.

Deflate the esophageal balloon


Monitor VS
Encourage him to take deep breaths
Notify the MD

Answer: (A) Deflate the esophageal balloon


When a client with a Sengstaken-Blakemore tube develops difficulty of breathing, it
means the tube is displaced and the inflated balloon is in the oropharynx causing airway
obstruction
46. The client presents with severe rectal bleeding, 16 diarrheal stools a day, severe
abdominal pain, tenesmus and dehydration. Because of these symptoms the nurse
should be alert for other problems associated with what disease?
A. Chrons disease
B. Ulcerative colitis
C. Diverticulitis
D. Peritonitis
Answer: (B) Ulcerative colitis
Ulcerative colitis is a chronic inflammatory condition producing edema and ulceration
affecting the entire colon. Ulcerations lead to sloughing that causes stools as many as
10-20 times a day that is filled with blood, pus and mucus. The other symptoms
mentioned accompany the problem.

47. A client is being evaluated for cancer of the colon. In preparing the client for barium
enema, the nurse should:
A. Give laxative the night before and a cleansing enema in the morning before the test
B. Render an oil retention enema and give laxative the night before
C. Instruct the client to swallow 6 radiopaque tablets the evening before the study
D. Place the client on CBR a day before the study
Answer: (A) Give laxative the night before and a cleansing enema in the morning before
the test
Barium enema is the radiologic visualization of the colon using a die. To obtain accurate
results in this procedure, the bowels must be emptied of fecal material thus the need for
laxative and enema.
48. The client has a good understanding of the means to reduce the chances of colon
cancer when
he states:
A. I will exercise daily.
B. I will include more red meat in my diet.
C. I will have an annual chest x-ray.
D. I will include more fresh fruits and vegetables in my diet.
Answer: (D) I will include more fresh fruits and vegetables in my diet.
Numerous aspects of diet and nutrition may contribute to the development of cancer. A
low-fiber diet, such as when fresh fruits and vegetables are minimal or lacking in the
diet, slows transport of materials through the gut which has been linked to colorectal
cancer.
49. Days after abdominal surgery, the clients wound dehisces. The safest nursing
intervention when
this occurs is to
A. Cover the wound with sterile, moist saline dressing
B. Approximate the wound edges with tapes
C. Irrigate the wound with sterile saline
D. Hold the abdominal contents in place with a sterile gloved hand
Answer: (A) Cover the wound with sterile, moist saline dressing
Dehiscence is the partial or complete separation of the surgical wound edges. When this
occurs, the client is placed in low Fowlers position and instructed to lie quietly. The
wound should be covered to protect it from exposure and the dressing must be sterile to
protect it from infection and moist to prevent the dressing from sticking to the wound
which can disturb the healing process.

50. An intravenous pyelogram reveals that Paulo, age 35, has a renal calculus. He is
believed to have a small stone that will pass spontaneously. To increase the chance of
the stone passing, the nurse would instruct the client to force fluids and to
A. Strain all urine.
B. Ambulate.
C. Remain on bed rest.
D. Ask for medications to relax him.
Answer: (B) Ambulate.
Free unattached stones in the urinary tract can be passed out with the urine by
ambulation which can mobilize the stone and by increased fluid intake which will flush
out the stone during urination.
51. A female client is admitted with a diagnosis of acute renal failure. She is awake, alert,
oriented, and complaining of severe back pain, nausea and vomiting and abdominal
cramps. Her vital signs are blood pressure 100/70 mm Hg, pulse 110, respirations 30,
and oral temperature 100.4F (38C). Her electrolytes are sodium 120 mEq/L,
potassium 5.2 mEq/L; her urinary output for the first 8 hours is 50 ml. The client is
displaying signs of which electrolyte imbalance?
A. Hyponatremia
B. Hyperkalemia
C. Hyperphosphatemia
D. Hypercalcemia
Answer: (A) Hyponatremia
The normal serum sodium level is 135 145 mEq/L. The clients serum sodium is below
normal. Hyponatremia also manifests itself with abdominal cramps and nausea and
vomiting
52. Assessing the laboratory findings, which result would the nurse most likely expect to
find in a
client with chronic renal failure?
A. BUN 10 to 30 mg/dl, potassium 4.0 mEq/L, creatinine 0.5 to 1.5 mg/dl
B. Decreased serum calcium, blood pH 7.2, potassium 6.5 mEq/L
C. BUN 15 mg/dl, increased serum calcium, creatinine l.0 mg/dl
D. BUN 35 to 40 mg/dl, potassium 3.5 mEq/L, pH 7.35, decreased serum calcium
Answer: (B) Decreased serum calcium, blood pH 7.2, potassium 6.5 mEq/L
Chronic renal failure is usually the end result of gradual tissue destruction and loss of
renal function. With the loss of renal function, the kidneys ability to regulate fluid and
electrolyte and acid base balance results. The serum Ca decreases as the kidneys fail to
excrete phosphate, potassium and hydrogen ions are retained.
53. Treatment with hemodialysis is ordered for a client and an external shunt is created.
Which nursing action would be of highest priority with regard to the external shunt?

A. Heparinize it daily.
B. Avoid taking blood pressure measurements or blood samples from the affected arm.
C. Change the Silastic tube daily.
D. Instruct the client not to use the affected arm.
Answer: (B) Avoid taking blood pressure measurements or blood samples from the
affected arm.
In the client with an external shunt, dont use the arm with the vascular access site to
take blood pressure readings, draw blood, insert IV lines, or give injections because
these procedures may rupture the shunt or occlude blood flow causing damage and
obstructions in the shunt.
54. Romeo Diaz, age 78, is admitted to the hospital with the diagnosis of benign
prostatic hyperplasia (BPH). He is scheduled for a transurethral resection of the
prostate (TURP). It would be inappropriate to include which of the following points in
the preoperative teaching?
A. TURP is the most common operation for BPH.
B. Explain the purpose and function of a two-way irrigation system.
C. Expect bloody urine, which will clear as healing takes place.
D. He will be pain free.
Answer: (D) He will be pain free.
Surgical interventions involve an experience of pain for the client which can come in
varying degrees. Telling the pain that he will be pain free is giving him false reassurance.
55. Roxy is admitted to the hospital with a possible diagnosis of appendicitis. On
physical examination, the nurse should be looking for tenderness on palpation at
McBurneys point, which is located in the
A. left lower quadrant
B. left upper quadrant
C. right lower quadrant
D. right upper quadrant
Answer: (C) right lower quadrant
To be exact, the appendix is anatomically located at the Mc Burneys point at the right
iliac area of the right lower quadrant.
56. Mr. Valdez has undergone surgical repair of his inguinal hernia. Discharge teaching
should include
A. telling him to avoid heavy lifting for 4 to 6 weeks
B. instructing him to have a soft bland diet for two weeks
C. telling him to resume his previous daily activities without limitations
D. recommending him to drink eight glasses of water daily
Answer: (A) telling him to avoid heavy lifting for 4 to 6 weeks

The client should avoid lifting heavy objects and any strenuous activity for 4-6 weeks
after surgery to prevent stress on the inguinal area. There is no special diet required.
The fluid intake of eight glasses a day is good advice but is not a priority in this case.
57. A 30-year-old homemaker fell asleep while smoking a cigarette. She sustained severe
burns of the face,neck, anterior chest, and both arms and hands. Using the rule of nines,
which is the best estimate of total body-surface area burned?
A. 18%
B. 22%
C. 31%
D. 40%
Answer: (C) 31%
Using the Rule of Nine in the estimation of total body surface burned, we allot the
following: 9% - head; 9% - each upper extremity; 18%- front chest and abdomen; 18% entire back; 18% - each lower extremity and 1% - perineum.
58. Nursing care planning is based on the knowledge that the first 24-48 hours postburn are characterized by:
A.
B.
C.
D.

An increase in the total volume of intracranial plasma


Excessive renal perfusion with diuresis
Fluid shift from interstitial space
Fluid shift from intravascular space to the interstitial space

Answer: (D) Fluid shift from intravascular space to the interstitial space
This period is the burn shock stage or the hypovolemic phase. Tissue injury causes
vasodilation that results in increase capillary permeability making fluids shift from the
intravascular to the interstitial space. This can lead to a decrease in circulating blood
volume or hypovolemia which decreases renal perfusion and urine output.
59. If a client has severe bums on the upper torso, which item would be a primary
concern?
A. Debriding and covering the wounds
B. Administering antibiotics
C. Frequently observing for hoarseness, stridor, and dyspnea
D. Establishing a patent IV line for fluid replacement
Answer: (C) Frequently observing for hoarseness, stridor, and dyspnea
Burns located in the upper torso, especially resulting from thermal injury related to fires
can lead to inhalation burns. This causes swelling of the respiratory mucosa and
blistering which can lead to airway obstruction manifested by hoarseness, noisy and
difficult breathing. Maintaining a patent airway is a primary concern.

60. Contractures are among the most serious long-term complications of severe burns.
If a burn is located on the upper torso, which nursing measure would be least effective
to help prevent contractures?
A. Changing the location of the bed or the TV set, or both, daily
B. Encouraging the client to chew gum and blow up balloons
C. Avoiding the use of a pillow for sleep, or placing the head in a position of
hyperextension
D. Helping the client to rest in the position of maximal comfort
Answer: (D) Helping the client to rest in the position of maximal comfort
Mobility and placing the burned areas in their functional position can help prevent
contracture deformities related to burns. Pain can immobilize a client as he seeks the
position where he finds less pain and provides maximal comfort. But this approach can
lead to contracture deformities and other complications.
61. An adult is receiving Total Parenteral Nutrition (TPN). Which of the following
assessment is essential?
A. evaluation of the peripheral IV site
B. confirmation that the tube is in the stomach
C. assess the bowel sound
D. fluid and electrolyte monitoring
Answer: (D) fluid and electrolyte monitoring
Total parenteral nutrition is a method of providing nutrients to the body by an IV route.
The admixture is made up of proteins, carbohydrates, fats, electrolytes, vitamins, trace
minerals and sterile water based on individual client needs. It is intended to improve the
clients nutritional status. Because of its composition, it is important to monitor the
clients fluid intake and output including electrolytes, blood glucose and weight.
62. Which drug would be least effective in lowering a client's serum potassium level?
A. Glucose and insulin
B. Polystyrene sulfonate (Kayexalate)
C. Calcium glucomite
D. Aluminum hydroxide
Answer: (D) Aluminum hydroxide
Aluminum hydroxide binds dietary phosphorus in the GI tract and helps treat
hyperphosphatemia. All the other medications mentioned help treat hyperkalemia and
its effects.
63. A nurse is directed to administer a hypotonic intravenous solution. Looking at the
following labeled solutions, she should choose
A. 0.45% NaCl
B. 0.9% NaCl
C. D5W
D. D5NSS

Answer: (A) 0.45% NaCl


Hypotonic solutions like 0.45% NaCl has a lower tonicity that the blood; 0.9% NaCl and
D5W are isotonic solutions with same tonicity as the blood; and D5NSS is hypertonic
with a higher tonicity thab the blood.
64. A patient is hemorrhaging from multiple trauma sites. The nurse expects that
compensatory mechanisms associated with hypovolemia would cause all of the following
symptoms EXCEPT
A. hypertension
B. oliguria
C. tachycardia
D. tachypnea
Answer: (A) hypertension
In hypovolemia, one of the compenasatory mechanisms is activation of the sympathetic
nervous system that increases the RR & PR and helps restore the BP to maintain tissue
perfusion but not cause a hypertension. The SNS stimulation constricts renal arterioles
that increases release of aldosterone, decreases glomerular filtration and increases
sodium & water reabsorption that leads to oliguria.
65. Maria Sison, 40 years old, single, was admitted to the hospital with a diagnosis of
Breast Cancer. She was scheduled for radical mastectomy. Nursing care during the
preoperative period should consist of
A. assuring Maria that she will be cured of cancer
B. assessing Maria's expectations and doubts
C. maintaining a cheerful and optimistic environment
D. keeping Maria's visitors to a minimum so she can have time for herself
Answer: (B) assessing Maria's expectations and doubts
Assessing the clients expectations and doubts will help lessen her fears and anxieties.
The nurse needs to encourage the client to verbalize and to listen and correctly provide
explanations when needed.
66. Maria refuses to acknowledge that her breast was removed. She believes that her
breast is intact under the dressing. The nurse should
A. call the MD to change the dressing so Kathy can see the incision
B. recognize that Kathy is experiencing denial, a normal stage of the grieving process
C. reinforce Kathys belief for several days until her body can adjust to stress of surgery.
D. remind Kathy that she needs to accept her diagnosis so that she can begin
rehabilitation exercises.
Answer: (B) recognize that Kathy is experiencing denial, a normal stage of the grieving
process

A person grieves to a loss of a significant object. The initial stage in the grieving process
is denial, then anger, followed by bargaining, depression and last acceptance. The nurse
should show acceptance of the patients feelings and encourage verbalization.
67. A chemotherapeutic agent 5FU is ordered as an adjunct measure to surgery. Which
of the ff. statements about chemotherapy is true?
A. it is a local treatment affecting only tumor cells
B. it affects both normal and tumor cells
C. it has been proven as a complete cure for cancer
D. it is often used as a palliative measure.
Answer: (B) it affects both normal and tumor cells
Chemotherapeutic agents are given to destroy the actively proliferating cancer cells. But
these agents cannot differentiate the abnormal actively proliferating cancer cells from
those that are actively proliferating normal cells like the cells of the bone marrow, thus
the effect of bone marrow depression.
68. Which is an incorrect statement pertaining to the following procedures for cancer
diagnostics?
A. Biopsy is the removal of suspicious tissue and the only definitive method to diagnose
cancer
B. Ultrasonography detects tissue density changes difficult to observe by X-ray via
sound waves.
C. CT scanning uses magnetic fields and radio frequencies to provide cross-sectional
view of tumor
D. Endoscopy provides direct view of a body cavity to detect abnormality.
Answer: (C) CT scanning uses magnetic fields and radio frequencies to provide crosssectional view of tumor
CT scan uses narrow beam x-ray to provide cross-sectional view. MRI uses magnetic
fields and radio frequencies to detect tumors.
69. A post-operative complication of mastectomy is lymphedema. This can be prevented
by
A. ensuring patency of wound drainage tube
B. placing the arm on the affected side in a dependent position
C. restricting movement of the affected arm
D. frequently elevating the arm of the affected side above the level of the heart.
Answer: (D) frequently elevating the arm of the affected side above the level of the heart.
Elevating the arm above the level of the heart promotes good venous return to the heart
and good lymphatic drainage thus preventing swelling.
70. Which statement by the client indicates to the nurse that the patient understands
precautions necessary during internal radiation therapy for cancer of the cervix?

A. I should get out of bed and walk around in my room.


B. My 7 year old twins should not come to visit me while Im receiving treatment.
C. I will try not to cough, because the force might make me expel the application.
D. I know that my primary nurse has to wear one of those badges like the people in the
x-ray department, but they are not necessary for anyone else who comes in here.
Answer: (B) My 7 year old twins should not come to visit me while Im receiving
treatment.
Children have cells that are normally actively dividing in the process of growth.
Radiation acts not only against the abnormally actively dividing cells of cancer but also
on the normally dividing cells thus affecting the growth and development of the child
and even causing cancer itself.
71. High uric acid levels may develop in clients who are receiving chemotherapy. This is
caused by:
A. The inability of the kidneys to excrete the drug metabolites
B. Rapid cell catabolism
C. Toxic effect of the antibiotic that are given concurrently
D. The altered blood ph from the acid medium of the drugs
Answer: (B) Rapid cell catabolism
One of the oncologic emergencies, the tumor lysis syndrome, is caused by the rapid
destruction of large number of tumor cells. . Intracellular contents are released,
including potassium and purines, into the bloodstream faster than the body can
eliminate them. The purines are converted in the liver to uric acid and released into the
blood causing hyperuricemia. They can precipitate in the kidneys and block the tubules
causing acute renal failure.
72. Which of the following interventions would be included in the care of plan in a client
with cervical
implant?
A. Frequent ambulation
B. Unlimited visitors
C. Low residue diet
D. Vaginal irrigation every shift
Answer: (C) Low residue diet
It is important for the nurse to remember that the implant be kept intact in the cervix
during therapy. Mobility and vaginal irrigations are not done. A low residue diet will
prevent bowel movement that could lead to dislodgement of the implant. Patient is also
strictly isolated to protect other people from the radiation emissions
73. Which nursing measure would avoid constriction on the affected arm immediately
after mastectomy?

A. Avoid BP measurement and constricting clothing on the affected arm


B. Active range of motion exercises of the arms once a day.
C. Discourage feeding, washing or combing with the affected arm
D. Place the affected arm in a dependent position, below the level of the heart
Answer: (A) Avoid BP measurement and constricting clothing on the affected arm
A BP cuff constricts the blood vessels where it is applied. BP measurements should be
done on the unaffected arm to ensure adequate circulation and venous and lymph
drainage in the affected arm
74. A client suffering from acute renal failure has an unexpected increase in urinary
output to 150ml/hr. The nurse assesses that the client has entered the second phase of
acute renal failure. Nursing actions throughout this phase include observation for signs
and symptoms of
A. Hypervolemia, hypokalemia, and hypernatremia.
B. Hypervolemia, hyperkalemia, and hypernatremia.
C. Hypovolemia, wide fluctuations in serum sodium and potassium levels.
D. Hypovolemia, no fluctuation in serum sodium and potassium levels.
Answer: (C) Hypovolemia, wide fluctuations in serum sodium and potassium levels.
The second phase of ARF is the diuretic phase or high output phase. The diuresis can
result in an output of up to 10L/day of dilute urine. Loss of fluids and electrolytes occur.
75. An adult has just been brought in by ambulance after a motor vehicle accident. When
assessing the client, the nurse would expect which of the following manifestations could
have resulted from sympathetic nervous system stimulation?
A. A rapid pulse and increased RR
B. Decreased physiologic functioning
C. Rigid posture and altered perceptual focus
D. Increased awareness and attention
Answer: (A) A rapid pulse and increased RR
The fight or flight reaction of the sympathetic nervous system occurs during stress like
in a motor vehicular accident. This is manifested by increased in cardiovascular function
and RR to provide the immediate needs of the body for survival.
76. Ms. Sy undergoes surgery and the abdominal aortic aneurysm is resected and
replaced with a graft. When she arrives in the RR she is still in shock. The nurse's
priority should be
A. placing her in a trendeleburg position
B. putting several warm blankets on her
C. monitoring her hourly urine output
D. assessing her VS especially her RR

Answer: (D) assessing her VS especially her RR


Shock is characterized by reduced tissue and organ perfusion and eventual organ
dysfunction and failure. Checking on the VS especially the RR, which detects need for
oxygenation, is a priority to help detect its progress and provide for prompt
management before the occurrence of complications.
77. A major goal for the client during the first 48 hours after a severe bum is to prevent
hypovolemic shock. The best indicator of adequate fluid balance during this period is
A. Elevated hematocrit levels.
B. Urine output of 30 to 50 ml/hr.
C. Change in level of consciousness.
D. Estimate of fluid loss through the burn eschar.
Answer: (B) Urine output of 30 to 50 ml/hr.
Hypovolemia is a decreased in circulatory volume. This causes a decrease in tissue
perfusion to the different organs of the body. Measuring the hourly urine output is the
most quantifiable way of measuring tissue perfusion to the organs. Normal renal
perfusion should produce 1ml/kg of BW/min. An output of 30-50 ml/hr is considered
adequate and indicates good fluid balance.
78. A thoracentesis is performed on a chest-injured client, and no fluid or air is found.
Blood and fluids is administered intravenously (IV), but the client's vital signs do not
improve. A central venous pressure line is inserted, and the initial reading is 20 cm
H^O. The most likely cause of these findings is which of the following?
A. Spontaneous pneumothorax
B. Ruptured diaphragm
C. Hemothorax
D. Pericardial tamponade
Answer: (D) Pericardial tamponade
Pericardial tamponade occurs when there is presence of fluid accumulation in the
pericardial space that compresses on the ventricles causing a decrease in ventricular
filling and stretching during diastole with a decrease in cardiac output. . This leads to
right atrial and venous congestion manifested by a CVP reading above normal.
79. Intervention for a pt. who has swallowed a Muriatic Acid includes all of the following
except
A. administering an irritant that will stimulate vomiting
B. aspirating secretions from the pharynx if respirations are affected
C. neutralizing the chemical
D. washing the esophagus with large volumes of water via gastric lavage
Answer: (A) administering an irritant that will stimulate vomiting
Swallowing of corrosive substances causes severe irritation and tissue destruction of the
mucous membrane of the GI tract. Measures are taken to immediately remove the toxin

or reduce its absorption. For corrosive poison ingestion, such as in muriatic acid where
burn or perforation of the mucosa may occur, gastric emptying procedure is
immediately instituted, This includes gastric lavage and the administration of activated
charcoal to absorb the poison. Administering an irritant with the concomitant vomiting
to remove the swallowed poison will further cause irritation and damage to the mucosal
lining of the digestive tract. Vomiting is only indicated when non-corrosive poison is
swallowed.
80. Which initial nursing assessment finding would best indicate that a client has been
successfully resuscitated after a cardio-respiratory arrest?
A. Skin warm and dry
B. Pupils equal and react to light
C. Palpable carotid pulse
D. Positive Babinski's reflex
Answer: (C) Palpable carotid pulse
Presence of a palpable carotid pulse indicates the return of cardiac function which,
together with the return of breathing, is the primary goal of CPR. Pulsations in arteries
indicates blood flowing in the blood vessels with each cardiac contraction. Signs of
effective tissue perfusion will be noted after.
81. Chemical burn of the eye are treated with
A. local anesthetics and antibacterial drops for 24 36 hrs.
B. hot compresses applied at 15-minute intervals
C. Flushing of the lids, conjunctiva and cornea with tap or preferably sterile water
D. cleansing the conjunctiva with a small cotton-tipped applicator
Answer: (C) Flushing of the lids, conjunctiva and cornea with tap or preferably sterile
water
Prompt treatment of ocular chemical burns is important to prevent further damage.
Immediate tap-water eye irrigation should be started on site even before transporting
the patient to the nearest hospital facility. In the hospital, copious irrigation with
normal saline, instillation of local anesthetic and antibiotic is done.
82. The Heimlich maneuver (abdominal thrust), for acute airway obstruction, attempts
to:
A. Force air out of the lungs
B. Increase systemic circulation
C. Induce emptying of the stomach
D. Put pressure on the apex of the heart
Answer: (A) Force air out of the lungs
The Heimlich maneuver is used to assist a person choking on a foreign object. The
pressure from the thrusts lifts the diaphragm, forces air out of the lungs and creates an
artificial cough that expels the aspirated material.

83. John, 16 years old, is brought to the ER after a vehicular accident. He is pronounced
dead on arrival. When his parents arrive at the hospital, the nurse should:
A. ask them to stay in the waiting area until she can spend time alone with them
B. speak to both parents together and encourage them to support each other and
express their emotions freely
C. Speak to one parent at a time so that each can ventilate feelings of loss without
upsetting the other
D. ask the MD to medicate the parents so they can stay calm to deal with their sons
death.
Answer: (B) speak to both parents together and encourage them to support each other
and express their emotions freely
Sudden death of a family member creates a state of shock on the family. They go into a
stage of denial and anger in their grieving. Assisting them with information they need to
know, answering their questions and listening to them will provide the needed support
for them to move on and be of support to one another.
84. An emergency treatment for an acute asthmatic attack is Adrenaline 1:1000 given
hypodermically. This is given to:
A. increase BP
B. decrease mucosal swelling
C. relax the bronchial smooth muscle
D. decrease bronchial secretions
Answer: (C) relax the bronchial smooth muscle
Acute asthmatic attack is characterized by severe bronchospasm which can be relieved
by the immediate administration of bronchodilators. Adrenaline or Epinephrine is an
adrenergic agent that causes bronchial dilation by relaxing the bronchial smooth
muscles.
85. A nurse is performing CPR on an adult patient. When performing chest
compressions, the nurse understands the correct hand placement is located over the
A. upper half of the sternum
B. upper third of the sternum
C. lower half of the sternum
D. lower third of the sternum
Answer: (C) lower half of the sternum
The exact and safe location to do cardiac compression is the lower half of the sternum.
Doing it at the lower third of the sternum may cause gastric compression which can lead
to a possible aspiration.
86. The nurse is performing an eye examination on an elderly client. The client states
My vision is blurred, and I dont easily see clearly when I get into a dark room. The
nurse best response is:

A. You should be grateful you are not blind.


B. As one ages, visual changes are noted as part of degenerative changes. This is
normal.
C. You should rest your eyes frequently.
D. You maybe able to improve you vision if you move slowly.
Answer: (B) As one ages, visual changes are noted as part of degenerative changes. This
is normal.
Aging causes less elasticity of the lens affecting accommodation leading to blurred
vision. The muscles of the iris increase in stiffness and the pupils dilate slowly and less
completely so that it takes the older person to adjust when going to and from light and
dark environment and needs brighter light for close vision.
87. Which of the following activities is not encouraged in a patient after an eye surgery?
A. sneezing, coughing and blowing the nose
B. straining to have a bowel movement
C. wearing tight shirt collars
D. sexual intercourse
Answer: (D) sexual intercourse
To reduce increases in IOP, teach the client and family about activity restrictions. Sexual
intercourse can cause a sudden rise in IOP.
88. Which of the following indicates poor practice in communicating with a hearingimpaired client?
A. Use appropriate hand motions
B. Keep hands and other objects away from your mouth when talking to the client
C. Speak clearly in a loud voice or shout to be heard
D. Converse in a quiet room with minimal distractions
Answer: (C) Speak clearly in a loud voice or shout to be heard
Shouting raises the frequency of the sound and often makes understanding the spoken
words difficult. It is enough for the nurse to speak clearly and slowly.
89. A client is to undergo lumbar puncture. Which is least important information about
LP?
A. Specimens obtained should be labeled in their proper sequence.
B. It may be used to inject air, dye or drugs into the spinal canal.
C. Assess movements and sensation in the lower extremities after the
D. Force fluids before and after the procedure.
Answer: (D) Force fluids before and after the procedure.
LP involves the removal of some amount of spinal fluid. To facilitate CSF production,
the client is instructed to increase fluid intake to 3L, unless contraindicated, for 24 to 48
hrs after the procedure.

90. A client diagnosed with cerebral thrombosis is scheduled for cerebral angiography.
Nursing care of the client includes the following EXCEPT
A. Inform the client that a warm, flushed feeling and a salty taste may be
B. Maintain pressure dressing over the site of puncture and check for
C. Check pulse, color and temperature of the extremity distal to the site of
D. Kept the extremity used as puncture site flexed to prevent bleeding.
Answer: (D) Kept the extremity used as puncture site flexed to prevent bleeding.
Angiography involves the threading of a catheter through an artery which can cause
trauma to the endothelial lining of the blood vessel. The platelets are attracted to the
area causing thrombi formation. This is further enhanced by the slowing of blood flow
caused by flexion of the affected extremity. The affected extremity must be kept straight
and immobilized during the duration of the bedrest after the procedure. Ice bag can be
applied intermittently to the puncture site.
91. Which is considered as the earliest sign of increased ICP that the nurse should
closely observed for?
A. abnormal respiratory pattern
B. rising systolic and widening pulse pressure
C. contralateral hemiparesis and ipsilateral dilation of the pupils
D. progression from restlessness to confusion and disorientation to lethargy
Answer: (D) progression from restlessness to confusion and disorientation to lethargy
The first major effect of increasing ICP is a decrease in cerebral perfusion causing
hypoxia that produces a progressive alteration in the LOC. This is initially manifested by
restlessness.
92. Which is irrelevant in the pharmacologic management of a client with CVA?
A. Osmotic diuretics and corticosteroids are given to decrease cerebral edema
B. Anticonvulsants are given to prevent seizures
C. Thrombolytics are most useful within three hours of an occlusive CVA
D. Aspirin is used in the acute management of a completed stroke.
Answer: (D) Aspirin is used in the acute management of a completed stroke.
The primary goal in the management of CVA is to improve cerebral tissue perfusion.
Aspirin is a platelet deaggregator used in the prevention of recurrent or embolic stroke
but is not used in the acute management of a completed stroke as it may lead to
bleeding.
93. What would be the MOST therapeutic nursing action when a clients expressive
aphasia is severe?
A. Anticipate the client wishes so she will not need to talk

B. Communicate by means of questions that can be answered by the client shaking the
head
C. Keep us a steady flow rank to minimize silence
D. Encourage the client to speak at every possible opportunity.
Answer: (D) Encourage the client to speak at every possible opportunity.
Expressive or motor aphasia is a result of damage in the Brocas area of the frontal lobe.
It is amotor speech problem in which the client generally understands what is said but is
unable to communicate verbally. The patient can best he helped therefore by
encouraging him to communicate and reinforce this behavior positively.
94. A client with head injury is confused, drowsy and has unequal pupils. Which of the
following nursing diagnosis is most important at this time?
A. altered level of cognitive function
B. high risk for injury
C. altered cerebral tissue perfusion
D. sensory perceptual alteration
Answer: (C) altered cerebral tissue perfusion
The observations made by the nurse clearly indicate a problem of decrease cerebral
perfusion. Restoring cerebral perfusion is most important to maintain cerebral
functioning and prevent further brain damage.
95. Which nursing diagnosis is of the highest priority when caring for a client with
myasthenia gravis?
A. Pain
B. High risk for injury related to muscle weakness
C. Ineffective coping related to illness
D. Ineffective airway clearance related to muscle weakness
Answer: (D) Ineffective airway clearance related to muscle weakness
Myasthenia gravis causes a failure in the transmission of nerve impulses at the
neuromuscular junction which may be due to a weakening or decrease in acetylcholine
receptor sites. This leads to sporadic, progressive weakness or abnormal fatigability of
striated muscles that eventually causes loss of function. The respiratory muscles can
become weak with decreased tidal volume and vital capacity making breathing and
clearing the airway through coughing difficult. The respiratory muscle weakness may be
severe enough to require and emergency airway and mechanical ventilation.
96. The client has clear drainage from the nose and ears after a head injury. How can the
nurse determine if the drainage is CSF?
A. Measure the ph of the fluid
B. Measure the specific gravity of the fluid
C. Test for glucose
D. Test for chlorides

Answer: (C) Test for glucose


The CSF contains a large amount of glucose which can be detected by using glucostix. A
positive result with the drainage indicate CSF leakage.
97. The nurse includes the important measures for stump care in the teaching plan for a
client with an amputation. Which measure would be excluded from the teaching plan?
A. Wash, dry, and inspect the stump daily.
B. Treat superficial abrasions and blisters promptly.
C. Apply a "shrinker" bandage with tighter arms around the proximal end of the affected
limb.
D. Toughen the stump by pushing it against a progressively harder substance (e.g.,
pillow on a foot-stool).
Answer: (C) Apply a "shrinker" bandage with tighter arms around the proximal end of
the affected limb.
The shrinker bandage is applied to prevent swelling of the stump. It should be applied
with the distal end with the tighter arms. Applying the tighter arms at the proximal end
will impair circulation and cause swelling by reducing venous flow.
98. A 70-year-old female comes to the clinic for a routine checkup. She is 5 feet 4 inches
tall and weighs 180 pounds. Her major complaint is pain in her joints. She is retired and
has had to give up her volunteer work because of her discomfort. She was told her
diagnosis was osteoarthritis about 5 years ago. Which would be excluded from the
clinical pathway for this client?
A. Decrease the calorie count of her daily diet.
B. Take warm baths when arising.
C. Slide items across the floor rather than lift them.
D. Place items so that it is necessary to bend or stretch to reach them.
Answer: (D) Place items so that it is necessary to bend or stretch to reach them.
Patients with osteoarthritis have decreased mobility caused by joint pain. Over-reaching
and stretching to get an object are to be avoided as this can cause more pain and can
even lead to falls. The nurse should see to it therefore that objects are within easy reach
of the patient.
99. A client is admitted from the emergency department with severe-pain and edema in
the right foot. His diagnosis is gouty arthritis. When developing a plan of care, which
action would have the highest priority?
A. Apply hot compresses to the affected joints.
B. Stress the importance of maintaining good posture to prevent deformities.
C. Administer salicylates to minimize the inflammatory reaction.
D. Ensure an intake of at least 3000 ml of fluid per day.
Answer: (D) Ensure an intake of at least 3000 ml of fluid per day.

Gouty arthritis is a metabolic disease marked by urate deposits that cause painful
arthritic joints. The patient should be urged to increase his fluid intake to prevent the
development of urinary uric acid stones.
100. A client had a laminectomy and spinal fusion yesterday. Which statement is to be
excluded from your plan of care?
A. Before log rolling, place a pillow under the client's head and a pillow between the
client's legs.
B. Before log rolling, remove the pillow from under the client's head and use no pillows
between the client's legs.
C. Keep the knees slightly flexed while the client is lying in a semi-Fowler's position in
bed.
D. Keep a pillow under the client's head as needed for comfort.
Answer: (B) Before log rolling, remove the pillow from under the client's head and use
no pillows between the client's legs.
Following a laminectomy and spinal fusion, it is important that the back of the patient
be maintained in straight alignment and to support the entire vertebral column to
promote complete healing.
101. The nurse is assisting in planning care for a client with a diagnosis of immune
deficiency. The nurse would incorporate which of the ff. as a priority in the plan of care?
A. providing emotional support to decrease fear
B. protecting the client from infection
C. encouraging discussion about lifestyle changes
D. identifying factors that decreased the immune function
Answer: (B) protecting the client from infection
Immunodeficiency is an absent or depressed immune response that increases
susceptibility to infection. So it is the nurses primary responsibility to protect the
patient from infection.
102. Joy, an obese 32 year old, is admitted to the hospital after an automobile accident.
She has a fractured hip and is brought to the OR for surgery.
After surgery Joy is to receive a piggy-back of Clindamycin phosphate (Cleocin) 300 mg
in 50 ml of D5W. The piggyback is to infuse in 20 minutes. The drop factor of the IV set
is 10 gtt/ml. The nurse should set the piggyback to flow at:
A. 25 gtt/min
B. 30 gtt/min
C. 35 gtt/min
D. 45 gtt/min
Answer: (A) 25 gtt/min

To get the correct flow rate: multiply the amount to be infused (50 ml) by the drop
factor (10) and divide the result by the amount of time in minutes (20)
103. The day after her surgery Joy asks the nurse how she might lose weight. Before
answering her question, the nurse should bear in mind that long-term weight loss best
occurs when:
A. Fats are controlled in the diet
B. Eating habits are altered
C. Carbohydrates are regulated
D. Exercise is part of the program
Answer: (B) Eating habits are altered
For weight reduction to occur and be maintained, a new dietary program, with a balance
of foods from the basic four food groups, must be established and continued
104. The nurse teaches Joy, an obese client, the value of aerobic exercises in her weight
reduction program. The nurse would know that this teaching was effective when Joy
says that exercise will:
A. Increase her lean body mass
B. Lower her metabolic rate
C. Decrease her appetite
D. Raise her heart rate
Answer: (A) Increase her lean body mass
Increased exercise builds skeletal muscle mass and reduces excess fatty tissue.
105. The physician orders non-weight bearing with crutches for Joy, who had surgery
for a fractured hip. The most important activity to facilitate walking with crutches before
ambulation begun is:
A. Exercising the triceps, finger flexors, and elbow extensors
B. Sitting up at the edge of the bed to help strengthen back muscles
C. Doing isometric exercises on the unaffected leg
D. Using the trapeze frequently for pull-ups to strengthen the biceps muscles
Answer: (A) Exercising the triceps, finger flexors, and elbow extensors
These sets of muscles are used when walking with crutches and therefore need
strengthening prior to ambulation.
106. The nurse recognizes that a client understood the demonstration of crutch walking
when she places her weight on:
A. The palms and axillary regions
B. Both feet placed wide apart
C. The palms of her hands
D. Her axillary regions

Answer: (C) The palms of her hands


The palms should bear the clients weight to avoid damage to the nerves in the axilla
(brachial plexus)
107. Joey is a 46 year-old radio technician who is admitted because of mild chest pain.
He is 5 feet, 8 inches tall and weighs 190 pounds. He is diagnosed with a myocardial
infarct. Morphine sulfate, Diazepam (Valium) and Lidocaine are prescribed.
The physician orders 8 mg of Morphine Sulfate to be given IV. The vial on hand is
labeled 1 ml/ 10 mg. The nurse should administer:
A. 8 minims
B. 10 minims
C. 12 minims
D. 15 minims
Answer: (C) 12 minims
Using ratio and proportion 8 mg/10 mg = X minims/15 minims 10 X= 120 X = 12
minims The nurse will administer 12 minims intravenously equivalent to 8mg Morphine
Sulfate
108. Joey asks the nurse why he is receiving the injection of Morphine after he was
hospitalized for severe anginal pain. The nurse replies that it:
A. Will help prevent erratic heart beats
B. Relieves pain and decreases level of anxiety
C. Decreases anxiety
D. Dilates coronary blood vessels
Answer: (B) Relieves pain and decreases level of anxiety
Morphine is a specific central nervous system depressant used to relieve the pain
associated with myocardial infarction. It also decreases anxiety and apprehension and
prevents cardiogenic shock by decreasing myocardial oxygen demand.
109. Oxygen 3L/min by nasal cannula is prescribed for Joey who is admitted to the
hospital for chest pain. The nurse institutes safety precautions in the room because
oxygen:
A. Converts to an alternate form of matter
B. Has unstable properties
C. Supports combustion
D. Is flammable
Answer: (C) Supports combustion
The nurse should know that Oxygen is necessary to produce fire, thus precautionary
measures are important regarding its use.

110. Myra is ordered laboratory tests after she is admitted to the hospital for angina. The
isoenzyme test that is the most reliable early indicator of myocardial insult is:
A. SGPT
B. LDH
C. CK-MB
D. AST
Answer: (C) CK-MB
The cardiac marker, Creatinine phosphokinase (CPK) isoenzyme levels, especially the
MB sub-unit which is cardio-specific, begin to rise in 3-6 hours, peak in 12-18 hours and
are elevated 48 hours after the occurrence of the infarct. They are therefore most
reliable in assisting with early diagnosis. The cardiac markers elevate as a result of
myocardial tissue damage.
111. An early finding in the EKG of a client with an infarcted mycardium would be:
A. Disappearance of Q waves
B. Elevated ST segments
C. Absence of P wave
D. Flattened T waves
Answer: (B) Elevated ST segments
This is a typical early finding after a myocardial infarct because of the altered
contractility of the heart. The other choices are not typical of MI.
112. Jose, who had a myocardial infarction 2 days earlier, has been complaining to the
nurse about issues related to his hospital stay. The best initial nursing response would
be to:
A. Allow him to release his feelings and then leave him alone to allow him to regain his
composure
B. Refocus the conversation on his fears, frustrations and anger about his condition
C. Explain how his being upset dangerously disturbs his need for rest
D. Attempt to explain the purpose of different hospital routines
Answer: (B) Refocus the conversation on his fears, frustrations and anger about his
condition
This provides the opportunity for the client to verbalize feelings underlying behavior
and helpful in relieving anxiety. Anxiety can be a stressor which can activate the
sympathoadrenal response causing the release of catecholamines that can increase
cardiac contractility and workload that can further increase myocardial oxygen demand.
113. Twenty four hours after admission for an Acute MI, Joses temperature is noted at
39.3 C. The nurse monitors him for other adaptations related to the pyrexia, including:
A. Shortness of breath

B. Chest pain
C. Elevated blood pressure
D. Increased pulse rate
Answer: (D) Increased pulse rate
Fever causes an increase in the bodys metabolism, which results in an increase in
oxygen consumption and demand. This need for oxygen increases the heart rate, which
is reflected in the increased pulse rate. Increased BP, chest pain and shortness of breath
are not typically noted in fever.
114. Jose, who is admitted to the hospital for chest pain, asks the nurse, Is it still
possible for me to have another heart attack if I watch my diet religiously and avoid
stress? The most appropriate initial response would be for the nurse to:
A. Suggest he discuss his feelings of vulnerability with his physician.
B. Tell him that he certainly needs to be especially careful about his diet and lifestyle.
C. Avoid giving him direct information and help him explore his feelings
D. Recognize that he is frightened and suggest he talk with the psychiatrist or counselor.
Answer: (C) Avoid giving him direct information and help him explore his feelings
To help the patient verbalize and explore his feelings, the nurse must reflect and analyze
the feelings that are implied in the clients question. The focus should be on collecting
data to minister to the clients psychosocial needs.
115. Ana, 55 years old, is admitted to the hospital to rule out pernicious anemia. A
Schilling test is ordered for Ana. The nurse recognizes that the primary purpose of the
Schilling test is to determine the clients ability to:
A. Store vitamin B12
B. Digest vitamin B12
C. Absorb vitamin B12
D. Produce vitamin B12
Answer: (C) Absorb vitamin B12
Pernicious anemia is caused by the inability to absorb vitamin B12 in the stomach due to
a lack of intrinsic factor in the gastric juices. In the Schilling test, radioactive vitamin
B12 is administered and its absorption and excretion can be ascertained through the
urine.
116. Ana is diagnosed to have Pernicious anemia. The physician orders 0.2 mg of
Cyanocobalamin (Vitamin B12) IM. Available is a vial of the drug labeled 1 ml= 100 mcg.
The nurse should administer:
A. 0.5 ml
B. 1.0 ml
C. 1.5 ml
D. 2.0 ml

Answer: (D) 2.0 ml


First convert milligrams to micrograms and then use ratio and proportion (0.2 mg= 200
mcg) 200 mcg : 100 mcg= X ml : ml 100 X= 200 X = 2 ml. Inject 2 ml. to give 0.2 mg of
Cyanocobalamin.
117. Health teachings to be given to a client with Pernicious Anemia regarding her
therapeutic regimen concerning Vit. B12 will include:
A. Oral tablets of Vitamin B12 will control her symptoms
B. IM injections are required for daily control
C. IM injections once a month will maintain control
D. Weekly Z-track injections provide needed control
Answer: (C) IM injections once a month will maintain control
Deep IM injections bypass B12 absorption defect in the stomach due to lack of intrinsic
factor, the transport carrier component of gastric juices. A monthly dose is usually
sufficient since it is stored in active body tissues such as the liver, kidney, heart,
muscles, blood and bone marrow
118. The nurse knows that a client with Pernicious Anemia understands the teaching
regarding the vitamin B12 injections when she states that she must take it:
A. When she feels fatigued
B. During exacerbations of anemia
C. Until her symptoms subside
D. For the rest of her life
Answer: (D) For the rest of her life
Since the intrinsic factor does not return to gastric secretions even with therapy, B12
injections will be required for the remainder of the clients life.
119. Arthur Cruz, a 45 year old artist, has recently had an abdominoperineal resection
and colostomy. Mr. Cruz accuses the nurse of being uncomfortable during a dressing
change, because his wound looks terrible. The nurse recognizes that the client is using
the defense mechanism known as:
A. Reaction Formation
B. Sublimation
C. Intellectualization
D. Projection
Answer: (D) Projection
Projection is the attribution of unacceptable feelings and emotions to others which may
indicate the patients nonacceptance of his condition.
120. When preparing to teach a client with colostomy how to irrigate his colostomy, the
nurse should plan to perform the procedure:

A. When the client would have normally had a bowel movement


B. After the client accepts he had a bowel movement
C. Before breakfast and morning care
D. At least 2 hours before visitors arrive
Answer: (A) When the client would have normally had a bowel movement
Irrigation should be performed at the time the client normally defecated before the
colostomy to maintain continuity in lifestyle and usual bowel function/habit.
121. When observing an ostomate do a return demonstration of the colostomy irrigation,
the nurse notes that he needs more teaching if he:
A. Stops the flow of fluid when he feels uncomfortable
B. Lubricates the tip of the catheter before inserting it into the stoma
C. Hangs the bag on a clothes hook on the bathroom door during fluid insertion
D. Discontinues the insertion of fluid after only 500 ml of fluid has been instilled
Answer: (C) Hangs the bag on a clothes hook on the bathroom door during fluid
insertion
The irrigation bag should be hung 12-18 inches above the level of the stoma; a clothes
hook is too high which can create increase pressure and sudden intestinal distention and
cause abdominal discomfort to the patient.
122. When doing colostomy irrigation at home, a client with colostomy should be
instructed to report to his physician :
A. Abdominal cramps during fluid inflow
B. Difficulty in inserting the irrigating tube
C. Passage of flatus during expulsion of feces
D. Inability to complete the procedure in half an hour
Answer: (B) Difficulty in inserting the irrigating tube
Difficulty of inserting the irrigating tube indicates stenosis of the stoma and should be
reported to the physician. Abdominal cramps and passage of flatus can be expected
during colostomy irrigations. The procedure may take longer than half an hour.
123. A client with colostomy refuses to allow his wife to see the incision or stoma and
ignores most of his dietary instructions. The nurse on assessing this data, can assume
that the client is experiencing:
A. A reaction formation to his recent altered body image.
B. A difficult time accepting reality and is in a state of denial.
C. Impotency due to the surgery and needs sexual counseling
D. Suicide thoughts and should be seen by psychiatrist
Answer: (B) A difficult time accepting reality and is in a state of denial.

As long as no one else confirms the presence of the stoma and the client does not need to
adhere to a prescribed regimen, the clients denial is supported
124. The nurse would know that dietary teaching had been effective for a client with
colostomy when he states that he will eat:
A. Food low in fiber so that there is less stool
B. Everything he ate before the operation but will avoid those foods that cause gas
C. Bland foods so that his intestines do not become irritated
D. Soft foods that are more easily digested and absorbed by the large intestines
Answer: (B) Everything he ate before the operation but will avoid those foods that cause
gas
There is no special diets for clients with colostomy. These clients can eat a regular diet.
Only gas-forming foods that cause distention and discomfort should be avoided.
125. Eddie, 40 years old, is brought to the emergency room after the crash of his private
plane. He has suffered multiple crushing wounds of the chest, abdomen and legs. It is
feared his leg may have to be amputated.
When Eddie arrives in the emergency room, the assessment that assume the greatest
priority are:
A. Level of consciousness and pupil size
B. Abdominal contusions and other wounds
C. Pain, Respiratory rate and blood pressure
D. Quality of respirations and presence of pulsesQuality of respirations and presence of
pulses
Answer: (D) Quality of respirations and presence of pulsesQuality of respirations and
presence of pulses
Respiratory and cardiovascular functions are essential for oxygenation. These are top
priorities to trauma management. Basic life functions must be maintained or
reestablished
126. Eddie, a plane crash victim, undergoes endotracheal intubation and positive
pressure ventilation. The most immediate nursing intervention for him at this time
would be to:
A. Facilitate his verbal communication
B. Maintain sterility of the ventilation system
C. Assess his response to the equipment
D. Prepare him for emergency surgery
Answer: (C) Assess his response to the equipment
It is a primary nursing responsibility to evaluate effect of interventions done to the
client. Nothing is achieved if the equipment is working and the client is not responding

127. A chest tube with water seal drainage is inserted to a client following a multiple
chest injury. A few hours later, the clients chest tube seems to be obstructed. The most
appropriate nursing action would be to
A. Prepare for chest tube removal
B. Milk the tube toward the collection container as ordered
C. Arrange for a stat Chest x-ray film.
D. Clam the tube immediately
Answer: (B) Milk the tube toward the collection container as ordered
This assists in moving blood, fluid or air, which may be obstructing drainage, toward the
collection chamber
128. The observation that indicates a desired response to thoracostomy drainage of a
client with chest injury is:
A. Increased breath sounds
B. Constant bubbling in the drainage chamber
C. Crepitus detected on palpation of chest
D. Increased respiratory rate
Answer: (A) Increased breath sounds
The chest tube normalizes intrathoracic pressure and restores negative intra-pleural
pressure, drains fluid and air from the pleural space, and improves pulmonary function
129. In the evaluation of a clients response to fluid replacement therapy, the
observation that indicates adequate tissue perfusion to vital organs is:
A. Urinary output is 30 ml in an hour
B. Central venous pressure reading of 2 cm H2O
C. Pulse rates of 120 and 110 in a 15 minute period
D. Blood pressure readings of 50/30 and 70/40 within 30 minutes
Answer: (A) Urinary output is 30 ml in an hour
A rate of 30 ml/hr is considered adequate for perfusion of kidney, heart and brain.
130. A client with multiple injury following a vehicular accident is transferred to the
critical care unit. He begins to complain of increased abdominal pain in the left upper
quadrant. A ruptured spleen is diagnosed and he is scheduled for emergency
splenectomy. In preparing the client for surgery, the nurse should emphasize in his
teaching plan the:
A. Complete safety of the procedure
B. Expectation of postoperative bleeding
C. Risk of the procedure with his other injuries
D. Presence of abdominal drains for several days after surgery
Answer: (D) Presence of abdominal drains for several days after surgery

Drains are usually inserted into the splenic bed to facilitate removal of fluid in the area
that could lead to abscess formation.
131. To promote continued improvement in the respiratory status of a client following
chest tube removal after a chest surgery for multiple rib fracture, the nurse should:
A. Encourage bed rest with active and passive range of motion exercises
B. Encourage frequent coughing and deep breathing
C. Turn him from side to side at least every 2 hours
D. Continue observing for dyspnea and crepitus
Answer: (B) Encourage frequent coughing and deep breathing
This nursing action prevents atelectasis and collection of respiratory secretions and
promotes adequate ventilation and gas exchange.
132. A client undergoes below the knee amputation following a vehicular accident. Three
days postoperatively, the client is refusing to eat, talk or perform any rehabilitative
activities. The best initial nursing approach would be to:
A. Give him explanations of why there is a need to quickly increase his activity
B. Emphasize repeatedly that with as prosthesis, he will be able to return to his normal
lifestyle
C. Appear cheerful and non-critical regardless of his response to attempts at
intervention
D. Accept and acknowledge that his withdrawal is an initially normal and necessary part
of grieving
Answer: (D) Accept and acknowledge that his withdrawal is an initially normal and
necessary part of grieving
The withdrawal provides time for the client to assimilate what has occurred and
integrate the change in the body image. Acceptance of the clients behavior is an
important factor in the nurses intervention.
133. The key factor in accurately assessing how body image changes will be dealt with by
the client is the:
A. Extent of body change present
B. Suddenness of the change
C. Obviousness of the change
D. Clients perception of the change
Answer: (D) Clients perception of the change
It is not reality, but the clients feeling about the change that is the most important
determinant of the ability to cope. The client should be encouraged to his feelings.
134. Larry is diagnosed as having myelocytic leukemia and is admitted to the hospital
for chemotherapy. Larry discusses his recent diagnosis of leukemia by referring to

statistical facts and figures. The nurse recognizes that Larry is using the defense
mechanism known as:
A. Reaction formation
B. Sublimation
C. Intellectualization
D. Projection
Answer: (C) Intellectualization
People use defense mechanisms to cope with stressful events. Intellectualization is the
use of reasoning and thought processes to avoid the emotional upsets.
135. The laboratory results of the client with leukemia indicate bone marrow depression.
The nurse should encourage the client to:
A. Increase his activity level and ambulate frequently
B. Sleep with the head of his bed slightly elevated
C. Drink citrus juices frequently for nourishment
D. Use a soft toothbrush and electric razor
Answer: (D) Use a soft toothbrush and electric razor
Suppression of red bone marrow increases bleeding susceptibility associated with
thrombocytopenia, decreased platelets. Anemia and leucopenia are the two other
problems noted with bone marrow depression.
136. Dennis receives a blood transfusion and develops flank pain, chills, fever and
hematuria. The nurse recognizes that Dennis is probably experiencing:
A. An anaphylactic transfusion reaction
B. An allergic transfusion reaction
C. A hemolytic transfusion reaction
D. A pyrogenic transfusion reaction
Answer: (C) A hemolytic transfusion reaction
This results from a recipients antibodies that are incompatible with transfused RBCs;
also called type II hypersensitivity; these signs result from RBC hemolysis,
agglutination, and capillary plugging that can damage renal function, thus the flank pain
and hematuria and the other manifestations.
137. A client jokes about his leukemia even though he is becoming sicker and weaker.
The nurses most therapeutic response would be:
A. Your laugher is a cover for your fear.
B. He who laughs on the outside, cries on the inside.
C. Why are you always laughing?
D. Does it help you to joke about your illness?
Answer: (D) Does it help you to joke about your illness?

This non-judgmentally on the part of the nurse points out the clients behavior.
138. In dealing with a dying client who is in the denial stage of grief, the best nursing
approach is to:
A. Agree with and encourage the clients denial
B. Reassure the client that everything will be okay
C. Allow the denial but be available to discuss death
D. Leave the client alone to discuss the loss
Answer: (C) Allow the denial but be available to discuss death
This does not take away the clients only way of coping, and it permits future movement
through the grieving process when the client is ready. Dying clients move through the
different stages of grieving and the nurse must be ready to intervene in all these stages.
139. During and 8 hour shift, Mario drinks two 6 oz. cups of tea and vomits 125 ml of
fluid. During this 8 hour period, his fluid balance would be:
A. +55 ml
B. +137 ml
C. +235 ml
D. +485 ml
Answer: (C) +235 ml
The clients intake was 360 ml (6oz x 30 ml) and loss was 125 ml of fluid; loss is
subtracted from intake
140. Mr. Ong is admitted to the hospital with a diagnosis of Left-sided CHF. In the
assessment, the nurse should expect to find:
A. Crushing chest pain
B. Dyspnea on exertion
C. Extensive peripheral edema
D. Jugular vein distention
Answer: (B) Dyspnea on exertion
Pulmonary congestion and edema occur because of fluid extravasation from the
pulmonary capillary bed, resulting in difficult breathing. Left-sided heart failure creates
a backward effect on the pulmonary system that leads to pulmonary congestion.
141. The physician orders on a client with CHF a cardiac glycoside, a vasodilator, and
furosemide (Lasix). The nurse understands Lasix exerts is effects in the:
A. Distal tubule
B. Collecting duct
C. Glomerulus of the nephron
D. Ascending limb of the loop of Henle

Answer: (D) Ascending limb of the loop of Henle


This is the site of action of Lasix being a potent loop diuretic.
142. Mr. Ong weighs 210 lbs on admission to the hospital. After 2 days of diuretic
therapy he weighs 205.5 lbs. The nurse could estimate that the amount of fluid he has
lost is:
A. 0.5 L
B. 1.0 L
C. 2.0 L
D. 3.5 L
Answer: (C) 2.0 L
One liter of fluid weighs approximately 2.2 lbs. Therefore a 4.5 lbs weight loss equals
approximately 2 Liters.
143. Mr. Ong, a client with CHF, has been receiving a cardiac glycoside, a diuretic, and a
vasodilator drug. His apical pulse rate is 44 and he is on bed rest. The nurse concludes
that his pulse rate is most likely the result of the:
A. Diuretic
B. Vasodilator
C. Bed-rest regimen
D. Cardiac glycoside
Answer: (D) Cardiac glycoside
A cardiac glycoside such as digitalis increases force of cardiac contraction, decreases the
conduction speed of impulses within the myocardium and slows the heart rate.
144. The diet ordered for a client with CHF permits him to have a 190 g of
carbohydrates, 90 g of fat and 100 g of protein. The nurse understands that this diet
contains approximately:
A. 2200 calories
B. 2000 calories
C. 2800 calories
D. 1600 calories
Answer: (B) 2000 calories
There are 9 calories in each gram of fat and 4 calories in each gram of carbohydrate and
protein
145. After the acute phase of congestive heart failure, the nurse should expect the dietary
management of the client to include the restriction of:
A. Magnesium
B. Sodium
C. Potassium

D. Calcium
Answer: (B) Sodium
Restriction of sodium reduces the amount of water retention that reduces the cardiac
workload
146. Jude develops GI bleeding and is admitted to the hospital. An important etiologic
clue for the nurse to explore while taking his history would be:
A. The medications he has been taking
B. Any recent foreign travel
C. His usual dietary pattern
D. His working patterns
Answer: (A) The medications he has been taking
Some medications, such as aspirin and prednisone, irritate the stomach lining and may
cause bleeding with prolonged use
147. The meal pattern that would probably be most appropriate for a client recovering
from GI bleeding is:
A. Three large meals large enough to supply adequate energy.
B. Regular meals and snacks to limit gastric discomfort
C. Limited food and fluid intake when he has pain
D. A flexible plan according to his appetite
Answer: (B) Regular meals and snacks to limit gastric discomfort
Presence of food in the stomach at regular intervals interacts with HCl limiting acid
mucosal irritation. Mucosal irritation can lead to bleeding.
148. A client with a history of recurrent GI bleeding is admitted to the hospital for a
gastrectomy. Following surgery, the client has a nasogastric tube to low continuous
suction. He begins to hyperventilate. The nurse should be aware that this pattern will
alter his arterial blood gases by:
A. Increasing HCO3
B. Decreasing PCO2
C. Decreasing pH
D. Decreasing PO2
Answer: (B) Decreasing PCO2
Hyperventilation results in the increased elimination of carbon dioxide from the blood
that can lead to respiratory alkalosis.
149. Routine postoperative IV fluids are designed to supply hydration and electrolyte
and only limited energy. Because 1 L of a 5% dextrose solution contains 50 g of sugar, 3
L per day would apply approximately:

A. 400 Kilocalories
B. 600 Kilocalories
C. 800 Kilocalories
D. 1000 Kilocalories
Answer: (B) 600 Kilocalories
Carbohydrates provide 4 kcal/ gram; therefore 3L x 50 g/L x 4 kcal/g = 600 kcal; only
about a third of the basal energy need.
150. Thrombus formation is a danger for all postoperative clients. The nurse should act
independently to prevent this complication by:
A. Encouraging adequate fluids
B. Applying elastic stockings
C. Massaging gently the legs with lotion
D. Performing active-assistive leg exercises
Answer: (D) Performing active-assistive leg exercises
Inactivity causes venous stasis, hypercoagulability, and external pressure against the
veins, all of which lead to thrombus formation. Early ambulation or exercise of the lower
extremities reduces the occurrence of this phenomenon
151. An unconscious client is admitted to the ICU, IV fluids are started and a Foley
catheter is inserted. With an indwelling catheter, urinary infection is a potential danger.
The nurse can best plan to avoid this problem by:
A. Emptying the drainage bag frequently
B. Collecting a weekly urine specimen
C. Maintaining the ordered hydration
D. Assessing urine specific gravity
Answer: (C) Maintaining the ordered hydration
Promoting hydration, maintains urine production at a higher rate, which flushes the
bladder and prevents urinary stasis and possible infection
152. The nurse performs full range of motion on a bedridden clients extremities. When
putting his ankle through range of motion, the nurse must perform:
A. Flexion, extension and left and right rotation
B. Abduction, flexion, adduction and extension
C. Pronation, supination, rotation, and extension
D. Dorsiflexion, plantar flexion, eversion and inversion
Answer: (D) Dorsiflexion, plantar flexion, eversion and inversion
These movements include all possible range of motion for the ankle joint
153. A client has been in a coma for 2 months. The nurse understands that to prevent
the effects of shearing force on the skin, the head of the bed should be at an angle of:

A. 30 degrees
B. 45 degrees
C. 60 degrees
D. 90 degrees
Answer: (A) 30 degrees
Shearing force occurs when 2 surfaces move against each other; when the bed is at an
angle greater than 30 degrees, the torso tends to slide and causes this phenomenon.
Shearing forces are good contributory factors of pressure sores.
154. Rene, age 62, is scheduled for a TURP after being diagnosed with a Benign Prostatic
Hyperplasia (BPH). As part of the preoperative teaching, the nurse should tell the client
that after surgery:
A. Urinary control may be permanently lost to some degree
B. Urinary drainage will be dependent on a urethral catheter for 24 hours
C. Frequency and burning on urination will last while the cystotomy tube is in place
D. His ability to perform sexually will be permanently impaired
Answer: (B) Urinary drainage will be dependent on a urethral catheter for 24 hours
An indwelling urethral catheter is used, because surgical trauma can cause urinary
retention leading to further complications such as bleeding.
155. The transurethral resection of the prostate is performed on a client with BPH.
Following surgery, nursing care should include:
A. Changing the abdominal dressing
B. Maintaining patency of the cystotomy tube
C. Maintaining patency of a three-way Foley catheter for cystoclysis
D. Observing for hemorrhage and wound infection
Answer: (C) Maintaining patency of a three-way Foley catheter for cystoclysis
Patency of the catheter promotes bladder decompression, which prevents distention and
bleeding. Continuous flow of fluid through the bladder limits clot formation and
promotes hemostasis
156. In the early postoperative period following a transurethral surgery, the most
common complication the nurse should observe for is:
A. Sepsis
B. Hemorrhage
C. Leakage around the catheter
D. Urinary retention with overflow
Answer: (B) Hemorrhage
After transurethral surgery, hemorrhage is common because of venous oozing and
bleeding from many small arteries in the prostatic bed.

157. Following prostate surgery, the retention catheter is secured to the clients leg
causing slight traction of the inflatable balloon against the prostatic fossa. This is done
to:
A. Limit discomfort
B. Provide hemostasis
C. Reduce bladder spasms
D. Promote urinary drainage
Answer: (B) Provide hemostasis
The pressure of the balloon against the small blood vessels of the prostate creates a
tampon-like effect that causes them to constrict thereby preventing bleeding.
158. Twenty-four hours after TURP surgery, the client tells the nurse he has lower
abdominal discomfort. The nurse notes that the catheter drainage has stopped. The
nurses initial action should be to:
A. Irrigate the catheter with saline
B. Milk the catheter tubing
C. Remove the catheter
D. Notify the physician
Answer: (B) Milk the catheter tubing
Milking the tubing will usually dislodge the plug and will not harm the client. A
physicians order is not necessary for a nurse to check catheter patency.
159. The nurse would know that a post-TURP client understood his discharge teaching
when he says I should:
A. Get out of bed into a chair for several hours daily
B. Call the physician if my urinary stream decreases
C. Attempt to void every 3 hours when Im awake
D. Avoid vigorous exercise for 6 months after surgery
Answer: (B) Call the physician if my urinary stream decreases
Urethral mucosa in the prostatic area is destroyed during surgery and strictures my
form with healing that causes partial or even complete ueinary obstruction.
160. Lucy is admitted to the surgical unit for a subtotal thyroidectomy. She is diagnosed
with Graves Disease. When assessing Lucy, the nurse would expect to find:
A. Lethargy, weight gain, and forgetfulness
B. Weight loss, protruding eyeballs, and lethargy
C. Weight loss, exopthalmos and restlessness
D. Constipation, dry skin, and weight gain
Answer: (C) Weight loss, exopthalmos and restlessness

Classic signs associated with hyperthyroidism are weight loss and restlessness because
of increased basal metabolic rate. Exopthalmos is due to peribulbar edema.
161. Lucy undergoes Subtotal Thyroidectomy for Graves Disease. In planning for the
clients return from the OR, the nurse would consider that in a subtotal thyroidectomy:
A. The entire thyroid gland is removed
B. A small part of the gland is left intact
C. One parathyroid gland is also removed
D. A portion of the thyroid and four parathyroids are removed
Answer: (B) A small part of the gland is left intact
Remaining thyroid tissue may provide enough hormone for normal function. Total
thyroidectomy is generally done in clients with Thyroid Ca.
162. Before a post- thyroidectomy client returns to her room from the OR, the nurse
plans to set up emergency equipment, which should include:
A. A crash cart with bed board
B. A tracheostomy set and oxygen
C. An airway and rebreathing mask
D. Two ampules of sodium bicarbonate
Answer: (B) A tracheostomy set and oxygen
Acute respiratory obstruction in the post-operative period can result from edema,
subcutaneous bleeding that presses on the trachea, nerve damage, or tetany.
163. When a post-thyroidectomy client returns from surgery the nurse assesses her for
unilateral injury of the laryngeal nerve every 30 to 60 minutes by:
A. Observing for signs of tetany
B. Checking her throat for swelling
C. Asking her to state her name out loud
D. Palpating the side of her neck for blood seepage
Answer: (C) Asking her to state her name out loud
If the recurrent laryngeal nerve is damaged during surgery, the client will be hoarse and
have difficulty speaking.
164. On a post-thyroidectomy clients discharge, the nurse teaches her to observe for
signs of surgically induced hypothyroidism. The nurse would know that the client
understands the teaching when she states she should notify the physician if she
develops:
A. Intolerance to heat
B. Dry skin and fatigue
C. Progressive weight loss
D. Insomnia and excitability

Answer: (B) Dry skin and fatigue


Dry skin is most likely caused by decreased glandular function and fatigue caused by
decreased metabolic rate. Body functions and metabolism are decreased in
hypothyroidism.
165. A clients exopthalmos continues inspite of thyroidectomy for Graves Disease. The
nurse teaches her how to reduce discomfort and prevent corneal ulceration. The nurse
recognizes that the client understands the teaching when she says: I should:
A. Elevate the head of my bed at night
B. Avoid moving my extra-ocular muscles
C. Avoid using a sleeping mask at night
D. Avoid excessive blinking
Answer: (C) Avoid using a sleeping mask at night
The mask may irritate or scratch the eye if the client turns and lies on it during the
night.
166. Clara is a 37-year old cook. She is admitted for treatment of partial and fullthickness burns of her entire right lower extremity and the anterior portion of her right
upper extremity. Her respiratory status is compromised, and she is in pain and anxious.
Performing an immediate appraisal, using the rule of nines, the nurse estimates the
percent of Claras body surface that is burned is:
A. 4.5%
B. 9%
C. 18 %
D. 22.5%
Answer: (D) 22.5%
The entire right lower extremity is 18% the anterior portion of the right upper extremity
is 4.5% giving a total of 22.5%
167. The nurse applies mafenide acetate (Sulfamylon cream) to Clara, who has second
and third degree burns on the right upper and lower extremities, as ordered by the
physician. This medication will:
A. Inhibit bacterial growth
B. Relieve pain from the burn
C. Prevent scar tissue formation
D. Provide chemical debridement
Answer: (A) Inhibit bacterial growth
Sulfamylon is effective against a wide variety of gram positive and gram negative
organisms including anaerobes

168. Forty-eight hours after a burn injury, the physician orders for the client 2 liters of
IV fluid to be administered q12 h. The drop factor of the tubing is 10 gtt/ml. The nurse
should set the flow to provide:
A. 18 gtt/min
B. 28 gtt/min
C. 32 gtt/min
D. 36 gtt/min
Answer: (B) 28 gtt/min
This is the correct flow rate; multiply the amount to be infused (2000 ml) by the drop
factor (10) and divide the result by the amount of time in minutes (12 hours x 60
minutes)
169. Clara, a burn client, receives a temporary heterograft (pig skin) on some of her
burns. These grafts will:
A. Debride necrotic epithelium
B. Be sutured in place for better adherence
C. Relieve pain and promote rapid epithelialization
D. Frequently be used concurrently with topical antimicrobials.
Answer: (C) Relieve pain and promote rapid epithelialization
The graft covers nerve endings, which reduces pain and provides a framework for
granulation that promotes effective healing.
170. A client with burns on the chest has periodic episodes of dyspnea. The position that
would provide for the greatest respiratory capacity would be the:
A. Semi-fowlers position
B. Sims position
C. Orthopneic position
D. Supine position
Answer: (C) Orthopneic position
The orthopneic position lowers the diaphragm and provides for maximal thoracic
expansion
171. Jane, a 20- year old college student is admiited to the hospital with a tentative
diagnosis of myasthenia gravis. She is scheduled to have a series of diagnostic studies
for myasthenia gravis, including a Tensilon test. In preparing her for this procedure, the
nurse explains that her response to the medication will confirm the diagnosis if Tensilon
produces:
A. Brief exaggeration of symptoms
B. Prolonged symptomatic improvement
C. Rapid but brief symptomatic improvement
D. Symptomatic improvement of just the ptosis

Answer: (C) Rapid but brief symptomatic improvement


Tensilon acts systemically to increase muscle strength; with a peak effect in 30 seconds,
It lasts several minutes.
172. The initial nursing goal for a client with myasthenia gravis during the diagnostic
phase of her hospitalization would be to:
A. Develop a teaching plan
B. Facilitate psychologic adjustment
C. Maintain the present muscle strength
D. Prepare for the appearance of myasthenic crisis
Answer: (C) Maintain the present muscle strength
Until diagnosis is confirmed, primary goal should be to maintain adequate activity and
prevent muscle atrophy
173. The most significant initial nursing observations that need to be made about a
client with myasthenia include:
A. Ability to chew and speak distinctly
B. Degree of anxiety about her diagnosis
C. Ability to smile an to close her eyelids
D. Respiratory exchange and ability to swallow
Answer: (D) Respiratory exchange and ability to swallow
Muscle weakness can lead to respiratory failure that will require emergency intervention
and inability to swallow may lead to aspiration
174. Helen is diagnosed with myasthenia gravis and pyridostigmine bromide (Mestinon)
therapy is started. The Mestinon dosage is frequently changed during the first week.
While the dosage is being adjusted, the nurses priority intervention is to:
A. Administer the medication exactly on time
B. Administer the medication with food or mild
C. Evaluate the clients muscle strength hourly after medication
D. Evaluate the clients emotional side effects between doses
Answer: (C) Evaluate the clients muscle strength hourly after medication
Peak response occurs 1 hour after administration and lasts up to 8 hours; the response
will influence dosage levels.
175. Helen, a client with myasthenia gravis, begins to experience increased difficulty in
swallowing. To prevent aspiration of food, the nursing action that would be most
effective would be to:
A. Change her diet order from soft foods to clear liquids
B. Place an emergency tracheostomy set in her room
C. Assess her respiratory status before and after meals

D. Coordinate her meal schedule with the peak effect of her medication, Mestinon
Answer: (D) Coordinate her meal schedule with the peak effect of her medication,
Mestinon
Dysphagia should be minimized during peak effect of Mestinon, thereby decreasing the
probability of aspiration. Mestinon can increase her muscle strength including her
ability to swallow.

PSYCHIATRIC NURSING PRACTICE QUESTIONS WITH


RATIONALE
10:53 PM PSYCHIATRIC NURSING No comments
1.
Marco approached Nurse Trish asking for advice on how to deal with his alcohol
addiction. Nurse Trish should tell the client that the only effective treatment for
alcoholism is:
a.
Psychotherapy
b.
Alcoholics anonymous (A.A.)
c.
Total abstinence
d.
Aversion Therapy
2.
Nurse Hazel is caring for a male client who experience false sensory perceptions
with no basis in reality. This perception is known as:
a.
Hallucinations
b.
Delusions
c.
Loose associations
d.
Neologisms
3.
Nurse Monet is caring for a female client who has suicidal tendency. When
accompanying the client to the restroom, Nurse Monet should
a.
Give her privacy
b.
Allow her to urinate
c.
Open the window and allow her to get some fresh air
d.
Observe her
4.
Nurse Maureen is developing a plan of care for a female client with anorexia
nervosa. Which action should the nurse include in the plan?
a.
Provide privacy during meals
b.
Set-up a strict eating plan for the client
c.
Encourage client to exercise to reduce anxiety
d.
Restrict visits with the family
5.
A client is experiencing anxiety attack. The most appropriate nursing intervention
should include?
a.
Turning on the television
b.
Leaving the client alone
c.
Staying with the client and speaking in short sentences
d.
Ask the client to play with other clients
6.
A female client is admitted with a diagnosis of delusions of GRANDEUR. This
diagnosis reflects a belief that one is:

a.
b.
c.
d.

Being Killed
Highly famous and important
Responsible for evil world
Connected to client unrelated to oneself
7.
A 20 year old client was diagnosed with dependent personality disorder. Which
behavior is not most likely to be evidence of ineffective individual coping?
a.
Recurrent self-destructive behavior
b.
Avoiding relationship
c.
Showing interest in solitary activities
d.
Inability to make choices and decision without advise
8.
A male client is diagnosed with schizotypal personality disorder. Which signs
would this client exhibit during social situation?
a.
Paranoid thoughts
b.
Emotional affect
c.
Independence need
d.
Aggressive behavior
9.
Nurse Claire is caring for a client diagnosed with bulimia. The most appropriate
initial goal for a client diagnosed with bulimia is?
a.
Encourage to avoid foods
b.
Identify anxiety causing situations
c.
Eat only three meals a day
d.
Avoid shopping plenty of groceries
10. Nurse Tony was caring for a 41 year old female client. Which behavior by the client
indicates adult cognitive development?
a.
Generates new levels of awareness
b.
Assumes responsibility for her actions
c.
Has maximum ability to solve problems and learn new skills
d.
Her perception are based on reality
11. A neuromuscular blocking agent is administered to a client before ECT therapy.
The Nurse should carefully observe the client for?
a.
Respiratory difficulties
b.
Nausea and vomiting
c.
Dizziness
d.
Seizures
12. A 75 year old client is admitted to the hospital with the diagnosis of dementia of
the Alzheimers type and depression. The symptom that is unrelated to depression
would be?
a.
Apathetic response to the environment
b.
I dont know answer to questions
c.
Shallow of labile effect
d.
Neglect of personal hygiene
13. Nurse Trish is working in a mental health facility; the nurse priority nursing
intervention for a newly admitted client with bulimia nervosa would be to?
a.
Teach client to measure I & O
b.
Involve client in planning daily meal
c.
Observe client during meals
d.
Monitor client continuously

14. Nurse Patricia is aware that the major health complication associated with
intractable anorexia nervosa would be?
a.
Cardiac dysrhythmias resulting to cardiac arrest
b.
Glucose intolerance resulting in protracted hypoglycemia
c.
Endocrine imbalance causing cold amenorrhea
d.
Decreased metabolism causing cold intolerance
15. Nurse Anna can minimize agitation in a disturbed client by?
a.
Increasing stimulation
b.
limiting unnecessary interaction
c.
increasing appropriate sensory perception
d.
ensuring constant client and staff contact
16. A 39 year old mother with obsessive-compulsive disorder has become immobilized
by her elaborate hand washing and walking rituals. Nurse Trish recognizes that the basis
of O.C. disorder is often:
a.
Problems with being too conscientious
b.
Problems with anger and remorse
c.
Feelings of guilt and inadequacy
d.
Feeling of unworthiness and hopelessness
17. Mario is complaining to other clients about not being allowed by staff to keep food
in his room. Which of the following interventions would be most appropriate?
a.
Allowing a snack to be kept in his room
b.
Reprimanding the client
c.
Ignoring the clients behavior
d.
Setting limits on the behavior
18. Conney with borderline personality disorder who is to be discharge soon threatens
to do something to herself if discharged. Which of the following actions by the nurse
would be most important?
a.
Ask a family member to stay with the client at home temporarily
b.
Discuss the meaning of the clients statement with her
c.
Request an immediate extension for the client
d.
Ignore the clients statement because its a sign of manipulation
19. Joey a client with antisocial personality disorder belches loudly. A staff member
asks Joey, Do you know why people find you repulsive? this statement most likely
would elicit which of the following client reaction?
a.
Depensiveness
b.
Embarrassment
c.
Shame
d.
Remorsefulness
20. Which of the following approaches would be most appropriate to use with a client
suffering from narcissistic personality disorder when discrepancies exist between what
the client states and what actually exist?
a.
Rationalization
b.
Supportive confrontation
c.
Limit setting
d.
Consistency

21. Cely is experiencing alcohol withdrawal exhibits tremors, diaphoresis and


hyperactivity. Blood pressure is 190/87 mmhg and pulse is 92 bpm. Which of the
medications would the nurse expect to administer?
a.
Naloxone (Narcan)
b.
Benzlropine (Cogentin)
c.
Lorazepam (Ativan)
d.
Haloperidol (Haldol)
22. Which of the following foods would the nurse Trish eliminate from the diet of a
client in alcohol withdrawal?
a.
Milk
b.
Orange Juice
c.
Soda
d.
Regular Coffee
23. Which of the following would Nurse Hazel expect to assess for a client who is
exhibiting late signs of heroin withdrawal?
a.
Yawning & diaphoresis
b.
Restlessness & Irritability
c.
Constipation & steatorrhea
d.
Vomiting and Diarrhea
24. To establish open and trusting relationship with a female client who has been
hospitalized with severe anxiety, the nurse in charge should?
a.
Encourage the staff to have frequent interaction with the client
b.
Share an activity with the client
c.
Give client feedback about behavior
d.
Respect clients need for personal space
25. Nurse Monette recognizes that the focus of environmental (MILIEU) therapy is to:
a.
Manipulate the environment to bring about positive changes in behavior
b.
Allow the clients freedom to determine whether or not they will be involved in
activities
c.
Role play life events to meet individual needs
d.
Use natural remedies rather than drugs to control behavior
26. Nurse Trish would expect a child with a diagnosis of reactive attachment disorder
to:
a.
Have more positive relation with the father than the mother
b.
Cling to mother & cry on separation
c.
Be able to develop only superficial relation with the others
d.
Have been physically abuse
27. When teaching parents about childhood depression Nurse Trina should say?
a.
It may appear acting out behavior
b.
Does not respond to conventional treatment
c.
Is short in duration & resolves easily
d.
Looks almost identical to adult depression
28. Nurse Perry is aware that language development in autistic child resembles:
a.
Scanning speech
b.
Speech lag
c.
Shuttering
d.
Echolalia

29. A 60 year old female client who lives alone tells the nurse at the community health
center I really dont need anyone to talk to. The TV is my best friend. The nurse
recognizes that the client is using the defense mechanism known as?
a.
Displacement
b.
Projection
c.
Sublimation
d.
Denial
30. When working with a male client suffering phobia about black cats, Nurse Trish
should anticipate that a problem for this client would be?
a.
Anxiety when discussing phobia
b.
Anger toward the feared object
c.
Denying that the phobia exist
d.
Distortion of reality when completing daily routines
31. Linda is pacing the floor and appears extremely anxious. The duty nurse
approaches in an attempt to alleviate Lindas anxiety. The most therapeutic question by
the nurse would be?
a.
Would you like to watch TV?
b.
Would you like me to talk with you?
c.
Are you feeling upset now?
d.
Ignore the client
32. Nurse Penny is aware that the symptoms that distinguish post traumatic stress
disorder from other anxiety disorder would be:
a.
Avoidance of situation & certain activities that resemble the stress
b.
Depression and a blunted affect when discussing the traumatic situation
c.
Lack of interest in family & others
d.
Re-experiencing the trauma in dreams or flashback
33. Nurse Benjie is communicating with a male client with substance-induced
persisting dementia; the client cannot remember facts and fills in the gaps with
imaginary information. Nurse Benjie is aware that this is typical of?
a.
Flight of ideas
b.
Associative looseness
c.
Confabulation
d.
Concretism
34. Nurse Joey is aware that the signs & symptoms that would be most specific for
diagnosis anorexia are?
a.
Excessive weight loss, amenorrhea & abdominal distension
b.
Slow pulse, 10% weight loss & alopecia
c.
Compulsive behavior, excessive fears & nausea
d.
Excessive activity, memory lapses & an increased pulse
35. A characteristic that would suggest to Nurse Anne that an adolescent may have
bulimia would be:
a.
Frequent regurgitation & re-swallowing of food
b.
Previous history of gastritis
c.
Badly stained teeth
d.
Positive body image
36. Nurse Monette is aware that extremely depressed clients seem to do best in
settings where they have:

a.
b.
c.
d.

Multiple stimuli
Routine Activities
Minimal decision making
Varied Activities
37. To further assess a clients suicidal potential. Nurse Katrina should be especially
alert to the client expression of:
a.
Frustration & fear of death
b.
Anger & resentment
c.
Anxiety & loneliness
d.
Helplessness & hopelessness
38.
A nursing care plan for a male client with bipolar I disorder should include:
a.
Providing a structured environment
b.
Designing activities that will require the client to maintain contact with reality
c.
Engaging the client in conversing about current affairs
d.
Touching the client provide assurance
39. When planning care for a female client using ritualistic behavior, Nurse Gina must
recognize that the ritual:
a.
Helps the client focus on the inability to deal with reality
b.
Helps the client control the anxiety
c.
Is under the clients conscious control
d.
Is used by the client primarily for secondary gains
40. A 32 year old male graduate student, who has become increasingly withdrawn and
neglectful of his work and personal hygiene, is brought to the psychiatric hospital by his
parents. After detailed assessment, a diagnosis of schizophrenia is made. It is unlikely
that the client will demonstrate:
a.
Low self esteem
b.
Concrete thinking
c.
Effective self boundaries
d.
Weak ego
41. A 23 year old client has been admitted with a diagnosis of schizophrenia says to
the nurse Yes, its march, March is little woman. Thats literal you know. These
statement illustrate:
a.
Neologisms
b.
Echolalia
c.
Flight of ideas
d.
Loosening of association
42. A long term goal for a paranoid male client who has unjustifiably accused his wife
of having many extramarital affairs would be to help the client develop:
a.
Insight into his behavior
b.
Better self control
c.
Feeling of self worth
d.
Faith in his wife
43. A male client who is experiencing disordered thinking about food being poisoned
is admitted to the mental health unit. The nurse uses which communication technique
to encourage the client to eat dinner?
a.
Focusing on self-disclosure of own food preference
b.
Using open ended question and silence

c.
d.

Offering opinion about the need to eat


Verbalizing reasons that the client may not choose to eat
44. Nurse Nina is assigned to care for a client diagnosed with Catatonic Stupor. When
Nurse Nina enters the clients room, the client is found lying on the bed with a body
pulled into a fetal position. Nurse Nina should?
a.
Ask the client direct questions to encourage talking
b.
Rake the client into the dayroom to be with other clients
c.
Sit beside the client in silence and occasionally ask open-ended question
d.
Leave the client alone and continue with providing care to the other clients
45. Nurse Tina is caring for a client with delirium and states that look at the spiders
on the wall. What should the nurse respond to the client?
a.
Youre having hallucination, there are no spiders in this room at all
b.
I can see the spiders on the wall, but they are not going to hurt you
c.
Would you like me to kill the spiders
d.
I know you are frightened, but I do not see spiders on the wall
46. Nurse Jonel is providing information to a community group about violence in the
family. Which statement by a group member would indicate a need to provide additional
information?
a.
Abuse occurs more in low-income families
b.
Abuser Are often jealous or self-centered
c.
Abuser use fear and intimidation
d.
Abuser usually have poor self-esteem
47. During electroconvulsive therapy (ECT) the client receives oxygen by mask via
positive pressure ventilation. The nurse assisting with this procedure knows that
positive pressure ventilation is necessary because?
a.
Anesthesia is administered during the procedure
b.
Decrease oxygen to the brain increases confusion and disorientation
c.
Grand mal seizure activity depresses respirations
d.
Muscle relaxations given to prevent injury during seizure activity depress
respirations.
48. When planning the discharge of a client with chronic anxiety, Nurse Chris
evaluates achievement of the discharge maintenance goals. Which goal would be most
appropriately having been included in the plan of care requiring evaluation?
a.
The client eliminates all anxiety from daily situations
b.
The client ignores feelings of anxiety
c.
The client identifies anxiety producing situations
d.
The client maintains contact with a crisis counselor
49. Nurse Tina is caring for a client with depression who has not responded to
antidepressant medication. The nurse anticipates that what treatment procedure may be
prescribed?
a.
Neuroleptic medication
b.
Short term seclusion
c.
Psychosurgery
d.
Electroconvulsive therapy
50. Mario is admitted to the emergency room with drug-included anxiety related to
over ingestion of prescribed antipsychotic medication. The most important piece of
information the nurse in charge should obtain initially is the:

a.
b.
c.
d.

Length of time on the med.


Name of the ingested medication & the amount ingested
Reason for the suicide attempt
Name of the nearest relative & their phone number

ANSWERS AND RATIONALE


1. C. Total abstinence is the only effective treatment for alcoholism.
2. A. Hallucinations are visual, auditory, gustatory, tactile or olfactory perceptions that have
no basis in reality.
3. D. The Nurse has a responsibility to observe continuously the acutely suicidal client. The
Nurse should watch for clues, such as communicating suicidal thoughts, and messages;
hoarding medications and talking about death.
4. B. Establishing a consistent eating plan and monitoring clients weight are important to
this disorder.
5. C. Appropriate nursing interventions for an anxiety attack include using short sentences,
staying with the client, decreasing stimuli, remaining calm and medicating as needed.
6. B. Delusion of grandeur is a false belief that one is highly famous and important.
7. D. Individual with dependent personality disorder typically shows indecisiveness
submissiveness and clinging behavior so that others will make decisions with them.
8. A. Clients with schizotypal personality disorder experience excessive social anxiety that
can lead to paranoid thoughts.
9. B. Bulimia disorder generally is a maladaptive coping response to stress and underlying
issues. The client should identify anxiety causing situation that stimulate the bulimic
behavior and then learn new ways of coping with the anxiety.
10. A. An adult age 31 to 45 generates new level of awareness.
11. A. Neuromuscular Blocker, such as SUCCINYLCHOLINE (Anectine) produces
respiratory depression because it inhibits contractions of respiratory muscles.
12. C. With depression, there is little or no emotional involvement therefore little alteration
in affect.
13. D. These clients often hide food or force vomiting; therefore they must be carefully
monitored.
14. A. These clients have severely depleted levels of sodium and potassium because of their
starvation diet and energy expenditure, these electrolytes are necessary for cardiac
functioning.
15. B. Limiting unnecessary interaction will decrease stimulation and agitation.
16. C. Ritualistic behavior seen in this disorder is aimed at controlling guilt and inadequacy
by maintaining an absolute set pattern of behavior.
17. D. The nurse needs to set limits in the clients manipulative behavior to help the client
control dysfunctional behavior. A consistent approach by the staff is necessary to
decrease manipulation.
18. B. Any suicidal statement must be assessed by the nurse. The nurse should discuss the
clients statement with her to determine its meaning in terms of suicide.
19. A. When the staff member ask the client if he wonders why others find him repulsive,
the client is likely to feel defensive because the question is belittling. The natural
tendency is to counterattack the threat to self image.

20. B. The nurse would specifically use supportive confrontation with the client to point out
discrepancies between what the client states and what actually exists to increase
responsibility for self.
21. C. The nurse would most likely administer benzodiazepine, such as lorazepan (ativan) to
the client who is experiencing symptom: The clients experiences symptoms of
withdrawal because of the rebound phenomenon when the sedation of the CNS from
alcohol begins to decrease.
22. D. Regular coffee contains caffeine which acts as psychomotor stimulants and leads to
feelings of anxiety and agitation. Serving coffee top the client may add to tremors or
wakefulness.
23. D. Vomiting and diarrhea are usually the late signs of heroin withdrawal, along with
muscle spasm, fever, nausea, repetitive, abdominal cramps and backache.
24. D. Moving to a clients personal space increases the feeling of threat, which increases
anxiety.
25. A. Environmental (MILIEU) therapy aims at having everything in the clients
surrounding area toward helping the client.
26. C. Children who have experienced attachment difficulties with primary caregiver are not
able to trust others and therefore relate superficially
27. A. Children have difficulty verbally expressing their feelings, acting out behavior, such
as temper tantrums, may indicate underlying depression.
28. D. The autistic child repeat sounds or words spoken by others.
29. D. The client statement is an example of the use of denial, a defense that blocks problem
by unconscious refusing to admit they exist.
30. A. Discussion of the feared object triggers an emotional response to the object.
31. B. The nurse presence may provide the client with support & feeling of control.
32. D. Experiencing the actual trauma in dreams or flashback is the major symptom that
distinguishes post traumatic stress disorder from other anxiety disorder.
33. C. Confabulation or the filling in of memory gaps with imaginary facts is a defense
mechanism used by people experiencing memory deficits.
34. A. These are the major signs of anorexia nervosa. Weight loss is excessive (15% of
expected weight).
35. C. Dental enamel erosion occurs from repeated self-induced vomiting.
36. B. Depression usually is both emotional & physical. A simple daily routine is the best,
least stressful and least anxiety producing.
37. D. The expression of these feeling may indicate that this client is unable to continue the
struggle of life.
38. A. Structure tends to decrease agitation and anxiety and to increase the clients feeling
of security.
39. B. The rituals used by a client with obsessive compulsive disorder help control the
anxiety level by maintaining a set pattern of action.
40. C. A person with this disorder would not have adequate self-boundaries.
41. D. Loose associations are thoughts that are presented without the logical connections
usually necessary for the listening to interpret the message.
42. C. Helping the client to develop feeling of self worth would reduce the clients need to
use pathologic defenses.
43. B. Open ended questions and silence are strategies used to encourage clients to discuss
their problem in descriptive manner.

44. C. Clients who are withdrawn may be immobile and mute, and require consistent,
repeated interventions. Communication with withdrawn clients requires much patience
from the nurse. The nurse facilitates communication with the client by sitting in silence,
asking open-ended question and pausing to provide opportunities for the client to
respond.
45. D. When hallucination is present, the nurse should reinforce reality with the client.
46. A. Personal characteristics of abuser include low self-esteem, immaturity, dependence,
insecurity and jealousy.
47. D. A short acting skeletal muscle relaxant such as succinylcholine (Anectine) is
administered during this procedure to prevent injuries during seizure.
48. C. Recognizing situations that produce anxiety allows the client to prepare to cope with
anxiety or avoid specific stimulus.
49. D. Electroconvulsive therapy is an effective treatment for depression that has not
responded to medication.
50. B. In an emergency, lives saving facts are obtained first. The name and the amount of
medication ingested are of outmost important in treating this potentially life threatening
situation.

Das könnte Ihnen auch gefallen